0% found this document useful (0 votes)
2 views

AMT_Final

The document outlines the final examination paper for the Advanced Mathematical Techniques course at NED University for the Mechanical Engineering batch of 2018-2019. It includes instructions for students, a detailed question pattern covering various mathematical topics, and emphasizes the importance of presentation and explanation in answers. The exam consists of five questions, each with sub-parts, focusing on error analysis, numerical methods, integrals, complex variables, and mathematical modeling.

Uploaded by

Talha Bin Wasi
Copyright
© © All Rights Reserved
We take content rights seriously. If you suspect this is your content, claim it here.
Available Formats
Download as PDF, TXT or read online on Scribd
0% found this document useful (0 votes)
2 views

AMT_Final

The document outlines the final examination paper for the Advanced Mathematical Techniques course at NED University for the Mechanical Engineering batch of 2018-2019. It includes instructions for students, a detailed question pattern covering various mathematical topics, and emphasizes the importance of presentation and explanation in answers. The exam consists of five questions, each with sub-parts, focusing on error analysis, numerical methods, integrals, complex variables, and mathematical modeling.

Uploaded by

Talha Bin Wasi
Copyright
© © All Rights Reserved
We take content rights seriously. If you suspect this is your content, claim it here.
Available Formats
Download as PDF, TXT or read online on Scribd
You are on page 1/ 369

NED UNIVERSITY OF ENGINEERING & TECHNOLOGY, KARACHI

FINAL YEAR (MECHANICAL ENGINEERING)


SEMESTER EXAMINATION, 2021
BATCH 2018-2019
PAPER PATTREN

Dated: Xth August, 2022


Time: 3 Hours Max. Marks: 60
MT-441-ADVANCED MATHEMATICAL TECHNIQUES

Instructions: 1): Attempt all questions.


2): Neat, clean, clear and excellent presentation with good explanation (physical/mechanical) and
good final remark on your results shall be bring more marks.
3): All questions carry equal marks.

Q#1: (a): CH#1(From Error Analysis Part only, Applied) (12)


(Difference Operators is not included) (CLO-2)

(b): CH#2(From Lagranges, Newton’s, Least square methods Part only, Applied)
(Hermite’s and Cubic Spline methods are not included)

Q#2: (a): CH#3(From Numerical Differentiation, Applied) (12)


(b): CH#3(From Numerical Integration, Applied) (CLO-2)

Q#3: (a): CH#4(From Numerical Differential Equation, Applied) (12)


(b): CH#4(From Difference Equations, Applied) (CLO-2)

Q#4: (a): CH#5(From Improper Integrals) (12)


(b): CH#5(From Elliptic Integrals) (CLO-1)

Q#5: (a): CH#6(From Complex Variables) (12)


(b): CH#6(From Complex Variables) (CLO-1)
(b): CH#6(From Contour Integration)

Note:-
(i): Definition, Derivation and theory are not included in Exam.

(ii): However, all questions from Exercises are all included in Exam, whether their nature is
definition/derivation/theoretical type.

(ii): Further, we can ask you about Scientific/Geometrical/Mechanical/Physical/Philosophical/


Historical/Sociological/Economical discussion on any methods/Definition/Equations/Results/Conclusions.

WISH YOU GREAT SUCCESS


NED UNIVERSITY OF ENGINEERING & TECHNOLOGY, KARACHI
FINAL YEAR (MECHANICAL ENGINEERING)
SEMESTER EXAMINATION, 2021
BATCH 2018-2019
SAMPLE PAPER-2022

Dated: Xth August, 2022


Time: 3 Hours Max. Marks: 60
MT-441-ADVANCED MATHEMATICAL TECHNIQUES

Instructions: 1): Attempt all questions.


2): Neat, clean, clear and excellent presentation with good explanation (physical/mechanical) and
good final remark on your results shall be bring more marks.
3): All questions carry equal marks.

Q#1 (a): The voltage across a resistor is measured with error , and the resistance is measured (06)
with an error . Show that the error in calculating the power generated in the (CLO-2)
resistor is . If can be measured to an accuracy of and to an
accuracy of . What is the approximately possible Percentage Error in .

(b): The pressure drop that occurs when water flows through an orifice meter is measured using a (06)
differential pressure transmitter. The output current is converted to voltage drop by a resister. (CLO-2)
The objective is to correlate the flow rate with the voltage drop . The
following data were collected from an experiment:

3 5 6 9
293 508 585 764

(i): Derive a polynomial passing through these data points using Newton’s Divided interpolation method
(ii): Write Polynomial in simplest form.
(iii): Use this Polynomial to estimate at flow rate.
(iv): Interpret and conclude your results.

Q#2 (a): A slider in a machine moves along a fixed straight rod. Its distance along the rod is (06)
given below for various values of the time Find the velocity of the slider and (CLO-2)
its acceleration when

0 0.1 0.2 0.3 0.4 0.5 0.6

30.13 31.62 32.87 33.64 39.95 33.81 33.24

(i): Write some interesting/scientific/philosophical discussion on you method you have applied.
(ii): List the best conclusions you drawn from your findings.

(b): The speed of an electric train at various times are given in following data: (06)

0 0.5 1 1.5 2.0 2.5 3.0 3.25 3.5 (CLO-2)

0 13 33 39.5 40 40 36 15 10

(i): Provide firstly the rational/logical/geometrical/historical analysis of your method(s).


(ii): Find the total distance covered in 3.5 hours.
(iii): Explain your method and give scientific conclusions of your results.

PTO
Q#3: (a): The current in an circuit is given by (06)

{ } (CLO-2)

where is the voltage at time is the resistance, and is the inductance. At time
Compute at using the fourth-order RK-Method. Take
and

b): Discrete Mathematical Modeling of Electrical System leads to the following Nonhomogeneous (06)
Difference Equation of the form: (CLO-2)

(i): Solve the difference equations in the form


(ii): Can you relate this Discrete system to some Continuous Electrical/Dynamical system?
(iii): Write final conclusions on your General Solution.

Q4: (a): Test the convergence of the following improper integral and give explanation (06)
on your final results. (CLO-1)
i): ∫ . ii): ∫ .

(b): The study of some mechanical system leads to the following integral: (06)
1): Express this integral into Elliptic integral (if possible) (CLO-1)
2): Give some scientific explanation (may be physical/mechanical) and concluding remarks.


( )√( )( )

Q#5 (a): Show that does not exist even though this function approaches the same limit (03)
along every straight line through the origin. (CLO-1)

(b): Prove that the function is harmonic. Also find the conjugate harmonic (04)
function “v” and express the corresponding analytic function in term (CLO-1)
of z. .

(c): Evaluate the following Contour integral: ∫ ( )( )


∫ ( )( )
(05)
(CLO-1)

WISH YOU GREAT SUCCESS


NED UNIVERSITY OF ENGINEERING & TECHNOLOGY, KARACHI
FINAL YEAR (MECHANICAL ENGINEERING)
SEMESTER EXAMINATION, 2021
BATCH 2017-2018
LAST YEAR PAPER 2021

Dated: 26th August, 2021


Time: 3 Hours Max. Marks: 60
MT-441-ADVANCED MATHEMATICAL TECHNIQUES

Instructions: 1): Attempt all questions.


2): Neat, clean, clear and excellent presentation with good explanation (physical/mechanical) and
good final remark on your results shall be bring more marks.
3): All questions carry equal marks.

Q#1 (a): The study of some natural phenomena yield the following Definite Integral: (06)
∫ (CLO-1)
(i): Prove that, this integral can be express in term of the Elliptic Integral as:

{ (√ ) (√ )}

(ii): Give some interesting explanation may be physical/mechanical and concluding remarks
on this equivalency.
(b): The Mathematics/geometry of the certain Applied Mathematical Problems provide the (06)
following Improper integrals: (i): ∫ (ii): ∫ (CLO-1)

(i): Test the Convergence/Divergence of the above Improper integrals.


(ii): Write some suitable comments on your results.

Q#2 (a): Show that the following function is continuous at (06)

{ √ (CLO-1)

(b): The application of Contour Integral is to evaluate of certain type of Definite Integrals. (06)
In view of this definition/application either solve the following integral by some previous (CLO-1)
methods that you know OR by Residue Theorem:

Q#3 (a): The Mathematical modeling of four measurable quantities leads to the following relation: (06)
(CLO-2)
Find an upper limit to the Relative Error in the measurement of
OR
Integration provides a means to compute how much mass enters or leaves a reactor over a specified
time period, as in ∫
where the initial and final times, respectively. This formula makes intuitive sense if you
recall the analogy between integration and summation. Thus, the integral represents the summation
of the product of flow times concentration to give the total mass entering or leaving from .
If the flow rate is constant, Q can be moved outside the integral:

Use numerical integration to evaluate this equation for the data listed below. Note that .

PTO
(b): The following ordinary differential equation(ODE) is used to describe a Population of Fish: (06)
(CLO-2)
Where and are +ve constants; is known as the intrinsic growth constant. It measures the
differences between the birth and death rates per population unit in the absence of overcrowding.
The ratio is called saturation constant. Take and . Use initial condition,
Find , using Modified Euler’s Formula for step size . Compare your
results with the analytical solution of the above differential equation:

Q#4 (a): The Chemical Oxygen Demand(COD) is found to increase downstream a river. The data (06)
collected from the experiment are tabulated below: (CLO-2)

0 1 2 5
2 3 12 147

(i): Design a Mathematical Model through Polynomial using Lagrange interpolation method,
(ii): Write Polynomial in simplest form.
(iii): Use this Polynomial to estimate COD at 3km downstream.
(iv): Interpret and conclude your results.

(b): The following table gives the results of the measurement of Train resistance; is the Velocity (06)
in miles per hours, is the Resistance in pounds per ton: (CLO-2)

10 20 30 40 50
8 10 15 21 30

(i): Use the method of Least square to fit the a curve of the form to the above data.
(ii): Predict at via above relation.
(iii): How do you see this Exponential relation to fitting this data(It is good!).
(iv): Give some suitable remarks on your estimation.

Q#5 (a): The Discrete Mathematical Modeling of Forced Vibrating System of particular oscillation (06)
Phenomena leads to the following Nonhomogeneous Difference Equation of the form: (CLO-2)

(i): Solve the difference equations in the form


(ii): Can you relate this Discrete system to some Continuous Dynamical system?
(iii): Write final conclusions on your General Solution.

(b): A rod is rotating in a plan. The following table given the angle in radian through which the (06)
rod has turned for various values of the time in second. (CLO-2)
0 0.2 0.4 0.6 0.8 1.0 1.2
0 0.12 0.49 1.12 2.02 3.20 4.67

(i): Calculate the Angular Velocity and Angular Acceleration at

(ii): Give final accumulating remarks and interpretation of your findings.

WISH YOU GREAT SUCCESS EVERYWHERE


IMPORTANT FORMULAE OF THE AMT MT-441
CH#1: Error Analysis & Difference Operators:-
∆𝑽 𝑬 |𝑽𝑻 −𝑽𝑨 | | 𝑽𝑻 −𝑽𝑨 |
𝑬𝒂 = ∆𝑽 = | 𝑽𝑻 − 𝑽𝑨 |(Absolute) , 𝑬𝒓 = 𝑽 = 𝑽 𝒂 = (Relative) , 𝑬𝒑 = 𝑬𝒓 × 𝟏𝟎𝟎 = × 𝟏𝟎𝟎 (Percentage )
𝑻 𝑻 𝑽𝑻 𝑽𝑻

𝟏 𝟏
𝑬𝒂 ≤ 𝟐 × 𝟏𝟎−𝒏 = 𝟎. 𝟓 × 𝟏𝟎−𝒏 (Theorem -1) , 𝑬𝒓 ≤ 𝑲×𝟏𝟎𝒏−𝟏 (𝒏 ≠ 𝟏): (Theorem -2)
𝝏𝒖 𝝏𝒖 ∆𝒙𝒊
𝒖 = 𝒇(𝒙𝟏 , 𝒙𝟐 , 𝒙𝟑 , … , 𝒙𝒏 ) , ∆𝒖 = ∑𝒏𝒊=𝟏 𝝏𝒙 ∆𝒙𝒊 , 𝑷𝒓 = 𝑬𝒓 × 𝟏𝟎𝟎 = ∑𝒏𝒊=𝟏 𝝏𝒙 × 𝟏𝟎𝟎 (General Error Formula)
𝒊 𝒊 𝒖

CH#2: Interpolation & Curve Fitting:-


1): Interpolation
(𝒙−𝒙 )(𝒙−𝒙 )… (𝒙−𝒙 )(𝒙−𝒙 )…(𝒙−𝒙 )
𝒚(𝒙) = 𝒇(𝒙) ≅ 𝑷(𝒙) = ∑𝒏𝒊=𝟎 𝑳𝒊 (𝒙)𝒚𝒊 , 𝑳𝒊 (𝒙): (𝒙𝒊 −𝒙𝟎𝟎)(𝒙𝒊 −𝒙𝟏𝟏)…(𝒙𝒊 −𝒙𝒊−𝟏
𝒊−𝟏 𝒊+𝟏 𝒏
)(𝒙𝒊 −𝒙𝒊+𝟏 )…(𝒙𝒊 −𝒙𝒏 ) (Lagrange Formula)

𝒚(𝒙) = 𝑷(𝒙) = ∑𝒏𝒊=𝟎 𝑨𝒊 (𝒙)𝒚𝒊 + ∑𝒏𝒊=𝟎 𝑩𝒊 (𝒙)𝒚′𝒊 , 𝑨𝒊 (𝒙) = [𝟏 − 𝟐𝑳′ 𝒊 (𝒙𝒊 )(𝒙 − 𝒙𝒊 )][𝑳𝒊 (𝒙)]𝟐 , 𝑩𝒊 (𝒙) = (𝒙 − 𝒙𝒊 )[𝑳𝒊 (𝒙)]𝟐 (Hermite)
𝒑(𝒑−𝟏) 𝒑(𝒑−𝟏)(𝒑−𝟐) 𝒑(𝒑−𝟏)(𝒑−𝟐)…(𝒑−𝒏+𝟏) 𝒙−𝒙𝟎
𝑷(𝒙) = 𝒚𝟎 + 𝒑𝜟𝒚𝟎 + 𝚫𝟐 𝒚𝟎 + 𝚫𝟑 𝒚𝟎 +. . . + 𝜟𝒏 𝒚𝟎 where 𝒑 = (Newton’s Forward)
𝟐! 𝟑! 𝒏! 𝒉
𝒑(𝒑+𝟏) 𝒑(𝒑+𝟏)(𝒑+𝟐) 𝒑(𝒑+𝟏)(𝒑+𝟐)…(𝒑+𝒏−𝟏) 𝒙−𝒙𝒏
𝑷(𝒙) = 𝒚𝒏 + 𝒑𝛁𝒚𝒏 + 𝛁 𝟐 𝒚𝒏 + 𝛁 𝟑 𝒚𝒏 +. . . + 𝛁 𝒏 𝒚𝒏 where 𝒑 = (Newton’s Backward)
𝟐! 𝟑! 𝒏! 𝒉

𝒚(𝒙) = 𝑷(𝒙) = 𝒇(𝒙𝟎 ) + (𝒙 − 𝒙𝟎 )𝒇[𝒙𝟎 , 𝒙𝟏 ] + (𝒙 − 𝒙𝟎 )(𝒙 − 𝒙𝟏 )𝒇[𝒙𝟎 , 𝒙𝟏 , 𝒙𝟐 ] + (𝒙 − 𝒙𝟎 )(𝒙 − 𝒙𝟏 )(𝒙 − 𝒙𝟐 )𝒇[𝒙𝟎 , 𝒙𝟏 , 𝒙𝟐 , 𝒙𝟑 ]

+ ⋯ … . +(𝒙 − 𝒙𝟎 )(𝒙 − 𝒙𝟏 ) … (𝒙 − 𝒙𝒏−𝟏 )𝒇[𝒙𝟎 , 𝒙𝟏 , … , 𝒙𝒏 ] (Newton’s Divided Difference Fotrmula)

𝟏 𝟏 𝒉𝟐 𝟏 𝒉𝟐
𝒚(𝒙) = 𝑺(𝒙) = )𝟑
[(𝒙𝒊 − 𝒙 𝑴𝒊−𝟏 + (𝒙 − 𝒙𝒊−𝟏 𝑴𝒊 ] + (𝒙𝒊 − 𝒙) [𝒚𝒊−𝟏 − )𝟑 𝑴𝒊−𝟏 ] + (𝒙 − 𝒙𝒊−𝟏 ) [𝒚𝒊 − 𝑴𝒊 ] (Cubic Spline
𝟔𝒉 𝒉 𝟔 𝒉 𝟔 Formula)
𝟔
𝑴𝒊−𝟏 + 𝟒𝑴𝒊 + 𝑴𝒊+𝟏 = 𝒉𝟐 [𝒚𝒊−𝟏 − 𝟐𝒚𝒊 + 𝒚𝒊+𝟏 ], 𝐢 = 𝟏, 𝟐, 𝟑, … . . , 𝐧 − 𝟏 and 𝑴𝟎 = 𝑴𝒏 = 𝟎

2: Least Square Methods:-

1): Let 𝒚 = 𝒂𝒙 + 𝒃 be the best fit straight line to the data. Then the normal equations are:

𝒂 ∑ 𝒙𝒊 + 𝒏𝒃 = ∑ 𝒚𝒊 , 𝒂 ∑ 𝒙𝟐𝒊 + 𝒃 ∑ 𝒙𝒊 = ∑ 𝒙𝒊 𝒚𝒊 (Least Square Formulae for Lines)

2): Let 𝒚 = 𝒂𝒙𝟐 + 𝒃𝒙 + 𝒄 be the best fit parabola. Then the normal equations are:

𝒂 ∑ 𝒙𝟐𝒊 + 𝒃 ∑ 𝒙𝒊 + 𝒏𝒄 = ∑ 𝒚𝒊 , 𝒂 ∑ 𝒙𝟑𝒊 + 𝒃 ∑ 𝒙𝟐𝒊 + 𝒄 ∑ 𝒙𝒊 = ∑ 𝒙𝒊 𝒚𝒊 , a∑ 𝒙𝟒𝒊 + 𝒃 ∑ 𝒙𝟑𝒊 + 𝒄 ∑ 𝒙𝟐𝒊 = ∑ 𝒙𝟐𝒊 𝒚𝒊 (Least Square Formulae for Parabola)

CH#3: Numerical Differentiation & Integration Formulae:-

1): Numerical Differentiation Formulae:-


𝒅𝒚 𝟏 𝟐𝒑−𝟏 𝟐 𝟑𝒑𝟐 −𝟔𝒑+𝟐 𝟑 𝟒𝒑𝟑 −𝟏𝟖𝒑𝟐 +𝟐𝟐𝒑−𝟔 𝟒 𝟓𝒑𝟒 −𝟒𝟎𝒑𝟑 +𝟏𝟎𝟓𝒑𝟐 −𝟏𝟎𝟎𝒑+𝟐𝟒 𝟓
⇒ 𝒅𝒙 = 𝒉
[ ∆ 𝒚𝒐 + 𝟐
∆ 𝒚𝒐 + 𝟔
∆ 𝒚𝒐 + 𝟐𝟒
∆ 𝒚𝒐 + 𝟏𝟐𝟎
∆ 𝒚𝒐 + ⋯ ] (F1X)

𝐝𝟐 𝐲 𝟏 𝟔𝐩𝟐 −𝟏𝟖𝐩+𝟏𝟏 𝟐𝒑𝟑 −𝟏𝟐𝒑𝟐 +𝟐𝟏𝒑−𝟏𝟎


⇒ 𝐝𝐱 𝟐 = 𝐡𝟐
[∆𝟐 𝐲𝐨 + (𝐩 − 𝟏)∆𝟑 𝐲𝐨 + 𝟏𝟐
∆𝟒 𝐲𝐨 + 𝟏𝟐
∆𝟓 𝒚𝒐 + ⋯ ] (F2X)

𝒅𝒚 𝟏 𝟏 𝟏 𝟏 𝟏 𝟏
⇒ 𝒅𝒙| = 𝒉
[∆𝒚𝟎 − 𝟐 ∆𝟐 𝒚𝟎 + 𝟑 ∆𝟑 𝒚𝟎 − 𝟒 ∆𝟐 𝒚𝟎 + 𝟓 ∆𝟓 𝒚𝟎 − 𝟔 ∆𝟔 𝒚𝟎 + ⋯ ] (F1X0)
𝒙=𝒙𝟎

𝒅𝟐 𝒚 𝟏 𝟏𝟏 𝟒 𝟓
⇒ |
𝒅𝒙𝟐 𝒙=𝒙𝟎
=
𝒉𝟐
[∆𝟐 𝒚𝒐 – ∆𝟑 𝒚𝒐 +
𝟏𝟐
∆ 𝒚𝒐 − ∆𝟓 𝒚𝒐 … ] (F2X0)
𝟔

𝒅𝒚 𝟏 (∆𝒚𝒐 + ∆ 𝒚−𝟏 ) 𝟑𝒑𝟐 −𝟏 ∆𝟑 𝒚−𝟏 + ∆𝟑 𝒚−𝟐 (𝟒𝒑𝟑 −𝟐𝒑)


𝒅𝒙
= 𝒉 [( 𝟐
) + 𝒑∆𝟐 𝒚−𝟏 + 𝟑!
( 𝟐
) + 𝟒!
∆𝟒 𝒚−𝟐 + ⋯ . ] (C1X)

𝒅𝟐 𝒚 𝟏 ∆𝟑 𝒚−𝟏 + ∆𝟑 𝒚−𝟐 (𝟏𝟐𝒑𝟐 −𝟐)


𝒅𝒙𝟐
= 𝒉𝟐 [∆𝟐 𝒚−𝟏 + 𝒑 ( 𝟐
) + 𝟒!
∆𝟒 𝒚−𝟐 + ⋯ . ] (C2X)

𝒅𝒚 𝟏 (∆𝒚𝒐 + ∆ 𝒚−𝟏 ) 𝟏 ∆𝟑 𝒚−𝟏 + ∆𝟑 𝒚−𝟐


⇒ 𝒅𝒙| = 𝒉 [( 𝟐
) − 𝟔( 𝟐
)+ ⋯ . ] (C1X0)
𝒙=𝒙𝒐

𝒅𝟐 𝒚 𝟏 𝟏
⇒ | = [∆𝟐 𝒚−𝟏 − ∆𝟒 𝒚−𝟐 + ⋯ . ] (C2X0)
𝒅𝒙𝟐 𝒙=𝒙𝒐 𝒉𝟐 𝟏𝟐

2: Numerical Integration Formulae:-


𝒃 𝒉
∫𝒂 𝒇(𝒙) = 𝟐 [𝒇(𝒙𝟎 ) + 𝟐𝒇(𝒙𝟏 ) + 𝟐𝒇(𝒙𝟐 ) + 𝟐𝒇(𝒙𝟑 ) + ⋯ + 𝟐𝒇(𝒙𝒏−𝟏 ) + 𝒇(𝒙𝒏 )] (Trapezoidal Formula)

𝒃 𝒉
∫𝒂 𝒇(𝒙) = 𝟑 [𝒇(𝒙𝟎 ) + 𝟒𝒇(𝒙𝟏 ) + 𝟐𝒇(𝒙𝟐 ) + 𝟒𝒇(𝒙𝟑 ) + 𝟐𝒇(𝒙𝟒 ) + 𝟒𝒇(𝒙𝟓 ) + ⋯ + 𝟐𝒇(𝒙𝟐𝒏−𝟐 ) + 𝟒𝒇(𝒙𝟐𝒏−𝟏 ) + 𝒇(𝒙𝟐𝒏 )] (Simpson 1/3rd Formula)

𝒃 𝟑𝒉
∫𝒂 𝒇(𝒙) = [𝒇(𝒙𝟎 ) + 𝟑𝒇(𝒙𝟏 ) + 𝟑𝒇(𝒙𝟐 ) + 𝟐𝒇(𝒙𝟑 ) + 𝟑𝒇(𝒙𝟒 ) + 𝟑𝒇(𝒙𝟓 ) + 𝟐𝒇(𝒙𝟔 ) + ⋯ + 𝟑𝒇(𝒙𝟐𝒏−𝟐 ) + 𝟑𝒇(𝒙𝒏−𝟏 ) + 𝒇(𝒙𝒏 )](Simpson 3/8th)
𝟖

CH#4: Numerical Differentiation Equations & Difference Equations Formulae:-

1): Numerical Differentiation Equations Formulae:-


𝒉
𝒚𝒏+𝟏 = 𝒚𝒏 + 𝒉𝒇(𝒙𝒏 , 𝒚𝒏 ), 𝒏 = 𝟎, 𝟏, 𝟐, … (Euler Method) 𝒚𝒏+𝟏 = 𝒚𝒏 + 𝟐 [𝒇(𝒙𝒏 , 𝒚𝒏 ) + 𝒇(𝒙𝒏+𝟏 , 𝒚𝒏+𝟏 )], 𝒏 = 𝟎, 𝟏, 𝟐, ….(Modified Euler’s)

𝟏
𝒚𝒏+𝟏 = 𝒚𝒏 + 𝟔
(𝒌𝟏 + 𝟐𝒌𝟐 + 𝟐𝒌𝟑 + 𝒌𝟒 ), 𝒏 = 𝟎, 𝟏, 𝟐, (RK-Methods of 4th order)

𝒉 𝒌𝟏 𝒉 𝒌𝟐
where, 𝒌𝟏 = 𝒉𝒇(𝒙𝒏 , 𝒚𝒏 ), 𝒌𝟐 = 𝒉𝒇(𝒙𝒏 + , 𝒚𝒏 + ) , 𝒌𝟑 = 𝒉𝒇(𝒙𝒏 + , 𝒚𝒏 + ) , 𝒌𝟒 = 𝒉𝒇(𝒙𝒏 + 𝒉, 𝒚𝒏 + 𝒌𝟑 )
𝟐 𝟐 𝟐 𝟐

2):Difference Equations:-
𝟏 𝟏 𝒆∓𝒊𝒂 +𝒃
𝑪𝒐𝒔𝒃𝒌 = 𝑹𝒆(𝒆𝒊𝒃𝒌 ), 𝑺𝒊𝒏𝒃𝒌 = 𝑰𝒎(𝒆𝒊𝒃𝒌 ) , 𝒆±𝒊𝒂 +𝒃
= 𝒆±𝒊𝒂 +𝒃 × 𝒆∓𝒊𝒂 +𝒃,

𝒌(𝒏) = 𝒌(𝒌 − 𝒉)(𝒌 − 𝟐𝒉)(𝒌 − 𝟑𝒉) … . [𝒌 − (𝒏 − 𝟏)𝒉] (Factorial Formula) (i): 𝒌(𝟏) = 𝒌 (i): 𝒌(𝟐) = 𝒌(𝒌 − 𝟏) = 𝒌𝟐 − 𝒌
(i): ∆𝒌(𝒏) = 𝒏𝒌(𝒏−𝟏) (Derivative Formula) (ii): ∆𝒏 𝒌(𝒏) = 𝒏! (Power Formula) (iii): ∆𝒏+𝟏 𝒌(𝒏) = 𝟎 (Zero Formula)
𝟏 𝟏
1): (𝟏 − 𝒙)−𝟏 = 𝟏−𝒙 = 𝟏 + 𝒙 + 𝒙𝟐 + 𝒙𝟑 + ⋯ … 2): (𝟏 + 𝒙)−𝟏 = 𝟏+𝒙 = 𝟏 − 𝒙 + 𝒙𝟐 − 𝒙𝟑 + ⋯ …

CH#5: The Improper & Elliptic Integrals:-

1): The Improper Integrals

1): The Exponential and P-integrals:-



1): The integral: ∫𝒂 𝒆−𝒕𝒙 𝒅𝒙 , where 𝒕 is a constant converges if 𝒕 > 𝟎 and diverges if 𝒕 ≤ 𝟎.
∞ 𝒅𝒙
2): The integral: ∫𝒂 𝒙𝒑
where 𝒑 is a constant and 𝒂 > 𝟎, converges if 𝒑 > 𝟏 and diverges if p≤ 𝟏.

2): The Elliptic Integrals of 1st, 2nd & 3rd Kind:-


𝛟 𝒅𝜽 𝛟  𝒅𝜽
(i): 𝑭(𝒌, 𝝓) = ∫𝟎 (ii): 𝑺(𝒌, 𝝓) = ∫𝟎 √𝟏 − 𝒌𝟐 𝒔𝒊𝒏𝟐  𝒅𝜽 , (iii): 𝐓(𝒌, 𝒏, ) =∫𝟎 , 𝟎 < 𝒌 < 𝟏.
√𝟏−𝒌𝟐 𝒔𝒊𝒏𝟐  (𝟏+𝒏 𝐬𝐢𝐧𝟐 )√𝟏−𝒌𝟐 𝐬𝐢𝐧𝟐 

CH#6: Complex Variables:-

𝝎 = 𝒖(𝒙, 𝒚) + 𝒊𝒗(𝒙, 𝒚) = 𝒖 + 𝒊𝒗 , 𝐥𝐢𝐦 𝒇(𝒛) = 𝒇(𝒛𝒐 )[𝐢. 𝐞. 𝐥𝐢𝐦𝐢𝐭 = 𝐯𝐚𝐥𝐮𝐞 𝐨𝐟 𝐭𝐡𝐞 𝐟𝐮𝐧𝐜𝐭𝐢𝐨𝐧](Definition of Continuity)
𝒛→𝒛𝒐
𝝏𝒖 𝝏𝒗 𝝏𝒖 𝝏𝒗 𝒅𝝎 𝒅𝒇(𝒛) 𝝏𝒖 𝝏𝒗
= , =− ⇒ 𝒖𝒙 = 𝒗𝒚 , 𝒖𝒚 = −𝒗𝒙 (CR-Equations) , = = 𝒇′ (𝒛) = +𝒊 = 𝒖𝒙 + 𝒊𝒗𝒙 (Derivative Formula)
𝝏𝒙 𝝏𝒚 𝝏𝒚 𝝏𝒙 𝒅𝒛 𝒅𝒛 𝝏𝒙 𝝏𝒙
𝝏𝒖 𝟏 𝝏𝒗 𝝏𝒖 𝝏𝒗 𝒅𝝎 𝒅𝒇(𝒛) 𝝏𝒖 𝝏𝒗
𝝎 = 𝒇(𝒛) = 𝒖(𝒓, 𝜽) + 𝒊𝒗(𝒓, 𝜽) = 𝒖 + 𝒊𝒗 , 𝝏𝒓 = 𝒓 𝝏𝜽 , 𝝏𝜽 = −𝒓 𝝏𝒓 (CR-Eq. in Polar), 𝒅𝒛 = 𝒅𝒛
= 𝒇′ (𝒛) = 𝒆−𝒊𝜽 ( 𝝏𝒓 + 𝒊 𝝏𝒓) = 𝒆−𝒊𝜽 (𝒖𝒓 + 𝒊𝒗𝒓 )

𝝏𝒖 𝝏𝒖
𝒗 = ∫ 𝒅𝒗 = − ∫ 𝒅𝒙 + ∫ 𝒅𝒚 + 𝒄
𝝏𝒚 𝝏𝒙
𝒚−𝒄𝒐𝒏𝒔𝒕𝒂𝒏𝒕 𝑫𝒆𝒍𝒆𝒕𝒆 𝒙−𝒕𝒆𝒓𝒎𝒔

1): |𝒛 − 𝒂| = 𝒓, represents a circle with center (𝒂, 𝟎) and radius is 𝒓, in complex plane.

Residue at Simple Pole:-


Res (𝒛 = 𝒂) = R(𝒛 = 𝒂) = 𝐥𝐢𝐦(𝒛 − 𝒂)𝒇(𝒛) (At Simple Pole)
𝒛→𝒂

Residue at pole of order n:-


𝟏 𝒅𝒏−𝟏
Res (𝒛 = 𝒂) = R(𝒛 = 𝒂) = (𝒏−𝟏)!
𝐥𝐢𝐦 𝒅𝒛𝒏−𝟏
[(𝒛 − 𝒂)𝒏 𝒇(𝒛)]
𝒛→𝒂

Residue at double pole or pole of order 2:-


𝒅
Res (𝒛 = 𝒂) = R(𝒛 = 𝒂) = 𝐥𝐢𝐦 𝒅𝒛[(𝒛 − 𝒂)𝟐 𝒇(𝒛)]
𝒛→𝒂

Residue theorem:-

∫𝒄 𝒇(𝒛) 𝒅𝒛 = 𝟐𝝅𝒊[𝑹𝟏 + 𝑹𝟐 + ⋯ + 𝑹𝒏 ] OR ∫𝒄 𝒇(𝒛) 𝒅𝒛 = 𝟐𝝅𝒊 (Sum of the residues within 𝑪)

Contour Integration Type-1:- Contour Integration Type-2:- Contour Integration Type-3:-


𝟐𝝅 ∞ 𝑷(𝒙) ∞ 𝑷(𝒙) ∞ 𝑷(𝒙)
∫𝟎 𝒇(𝐬𝐢𝐧 𝜽 , 𝐜𝐨𝐬 𝜽) 𝒅𝜽 ∫−∞ 𝑸(𝒙) 𝒅𝒙 ∫−∞ 𝑸(𝒙) 𝒔𝒊𝒏 𝒎𝒙𝒅𝒙 or ∫−∞ 𝑸(𝒙) 𝐜𝐨𝐬 𝒎𝒙 𝒅𝒙
Advanced Mathematical Techniques Chapter # 1 Error Analysis & Difference Operators

CHAPTER # 1
ERROR ANALYSIS
&
DIFFERENCE OPERATORS

Dr. Jamil Book Series 2 Page 1


Advanced Mathematical Techniques Chapter # 1 Error Analysis & Difference Operators

CHAPTER # 1
PART-I: ERROR ANALYSIS
1.1 Nature or types of errors:-

The problems in applied mathematics, engineering and sciences ultimately converge to the
problems for finding the numerical results or numbers satisfying some kind of equations
under prescribed data. The checking procedures are adopted to eliminate all mistakes
whether these are human, due to the computer or technical, due to the calculating device.
Even when all such mistakes have been eliminated from a computation, the solution is not
generally exact on account of inherent errors of various types. The error in numerical
computation can enter in three different ways:
1): Inherent or propagation error,
2): Truncation errors,
3): Round-off errors.
1.1.1 Inherent Errors (Propagation or inherited errors):-

These errors are


1): due to the supplied data which take part in our numerical calculation throughout.
2): due to the approximate nature of the applied formula used in the solution.
3): and it is caused by the use of previous points calculated by the computer which already
have errors.
1.1.2 Truncation Error:-
The truncation error may occur as the result of replacing an infinite process by a finite one.
For example, such error appears when a definite integral is computed by Simpson’s rule or
when a differential equation is solved by some difference method. This error is not machine
– caused but is due to the method.
Example #1:-
The infinite series expansion of in term of Maclaurin’s series is given by

( )

If we approximate by the finite sum

( )

Then , the remainder after terms (from Maclaurin’s theorem), constitute the
truncation error given by

( ) ( )

Dr. Jamil Book Series 2 Page 2


Advanced Mathematical Techniques Chapter # 1 Error Analysis & Difference Operators

Example #2:-
Let ( ) be a continuously differentiable function in the domain. Then by definition
of derivative, we have

( ) ( )
( )

( ( ))
( )

So, if we write
( ) ( )
( )

( ) ( )

We commit an error, arising through the replacement of the infinitesimal process of


calculating the differential coefficient by the finite process of calculating simply the
difference quotient (average): Such an error is also a truncation error . We write:

( ) ( )

1.1.3 Round-off Errors:-


These error are unavoidable in most of the calculations since some of the quantities in the
calculations will be non-terminating decimals and for practical reasons only certain
number of figures will be carried in a calculation. These are due to the fact that in
computational work we have to deal with approximations.
Example:-
Consider and round-off it up to 3 decimal places, we get then
The difference that is the number is called
Round-off error. This is occurred due to Rounding-off a number.
1.2 Round-off Rules:-
To round-off a number to significant digits, discard all digits to the right of the
digit, and if this discarded number is:
1): less than half a unit in the place, leave the digit unaltered.
2): greater than half a unit in the place, increase the digit by unity.
3): exactly half a unit in the place, increase the digit by unity if it is odd, otherwise
leave it unchanged.
The number thus rounded-off is said to be correct to significant figures.

Dr. Jamil Book Series 2 Page 3


Advanced Mathematical Techniques Chapter # 1 Error Analysis & Difference Operators

1.2.1 Rounding off a numerical value to specified numberof decimal places


Numerical Value Rounded-off up to Round-off value

Significant Figures
Numerical Value Significant Figures

Rule-1:- Non-zero digits are significant:

Dr. Jamil Book Series 2 Page 4


Advanced Mathematical Techniques Chapter # 1 Error Analysis & Difference Operators

Rule-2:- Zeros between non-zero digits are significant:

Rule-3:- If zeros are not between non-zero digits, then they are not significant:

Rule-4:- Final zeros are significant:

Rule-5:- Initial zeros are not significant:

Miscellaneous Examples:-

1.3.1 Significant Errors:-


This is also a computational error and enters in a computation due to the loss of significant
digits. This type of error usually occurs in two cases:
1): When almost two equal numbers be subtracted.
2): When a number is divided by a very small number.

Dr. Jamil Book Series 2 Page 5


Advanced Mathematical Techniques Chapter # 1 Error Analysis & Difference Operators

Example # 1:-
Consider almost two equal numbers of eight significant figures as and
, then

Now it is clear that has only two significant figures namely and .
Therefore losses six significant digits though and are both of eight significant
digits. If this value of be used in further computation the subsequent result must be
affected by a serious amount of errors.
Example #2:-
As another example, we consider a function

( )

Now we consider a number , such that is very small with


respect to dividend . Now

( )
( )

If we take i.e., if has an error in the place, then

( )
( )

From above two calculations, it is clear that the error in computed value of ( ) occur in
the significant digit corresponding to the error in at the place. This error ccurs
due to the loss of significant figures.

1.3.2 Remarks:-
In modern arithmetical computation, the following rules should be followed.
1): For addition:-
a): The numbers which are least number of correct significant figures should be kept
unchanged.
b): The rest numbers should be rounded-off to one or two decimal places relative to those
which are least number of correct significant figures.
c): Then the number so obtained be added and then rounded-off the added number by
retaining as many significant figures as there are in the least accurate number among the
given numbers.

Dr. Jamil Book Series 2 Page 6


Advanced Mathematical Techniques Chapter # 1 Error Analysis & Difference Operators

2): For subtraction:-


When the subtraction of two approximate number is required, both the number should be
rounded-off to same decimal places at first, then after the numbers should be subtracted.
3): For multiplication:-
In this case the rules are same as was followed for addition.
Example # 1:-
Find the Sum of the approximate numbers:

Solution:-
The least accurate numbers are Rounding-off the remaining numbers
to two decimal places, we have

Example # 2:-
Subtract the approximate number from .
Solution:-
We first round-off the number correct to two decimal places, then

Example # 3:-
Find the Product of
Solution:-
We have:

× ×

1.3.3 Absolute, Relative and Percentage Error


1): Absolute Error :-
The absolute error is defined as the difference between its true value and its
approximate value i.e.

| |

2): The Relative Error :-


The relative error of a number is known as the absolute error divided by its true value i.e.

| |

Dr. Jamil Book Series 2 Page 7


Advanced Mathematical Techniques Chapter # 1 Error Analysis & Difference Operators

3): The Percentage Error :-


The percentage error is defined as the relative error multiplied by i.e

| |

Example:-
If , then its 3-Decimal Places Round-off number is

( )

Similarly, the 3-Significant Digits Round-off number of is

( )

This Example yields that in general the 3 Decimal Places Round-off number is NOT equal
to 3 Significant Digits Round-off number.

Theorem - 1:-
If is a true number and its -Decimal Places Rounding-off number is (
| |) the Absolute Error is given by

Theorem - 2:-
If is a true number and its -Significant Digits Rounding-off number is (
| | ) the Relative Error is given by

( )

where is the First Significant Digit in the number (either in true or approximate
number ).
Note:-
1): ( ) 2): ( ) 3): ( ) 4): ( )

Example # 1:-
Let a number be rounded to correct to decimal places, then
absolute error is

Dr. Jamil Book Series 2 Page 8


Advanced Mathematical Techniques Chapter # 1 Error Analysis & Difference Operators

Verification:-
From theorem -1, we have (here ):-

Hence theorem – 1 is verified. Now

From theorem – 2, we have (here ):-

Hence theorem – 2 is verified.


Example # 2:-
Write down the approximate representation of correct to four significant figures and
then find:
1): Absolute error
2): Relative error
3): Percentage error
Solution:-
The four significant figures representation of is

) | | | |

Verification:-
From theorem – 1, we have (here ): -

Dr. Jamil Book Series 2 Page 9


Advanced Mathematical Techniques Chapter # 1 Error Analysis & Difference Operators

Hence theorem -1 is verified


From theorem – 2, we have (here )

Hence theorem -2 is verified.


Example # 3:-
Find the number of Significant Figures in Approximate number if:
) given its absolute error as .

) given its relative error as .

) given its absolute error as .

Solution:-
a): Here

Then by theorem – 1, is correct to two decimal places that is is


correct to -significant figures.
b):We have

Thus, by theorem -1, the Approximate number is correct to three decimal places is
. Thus, the number of significant figures in is 4.
c): Here

Thus, by theorem – 1 the number is correct to two decimal places. Hence the number of
significant figures is two.
Example # 4:-
If and be the absolute error in and , find the

Dr. Jamil Book Series 2 Page 10


Advanced Mathematical Techniques Chapter # 1 Error Analysis & Difference Operators

relative error in computation of


Solution:-
Here

The result has an absolute error.

( )

Therefore, relative error in is


( )

which is the required error in

Example # 5:-
If ( ) , find the Percentage Error in at , if the error in is
Solution:-
We have

( )

At and , we have

( ( ) )

and

( ) ( )

Thus, percentage error in is (note that we are interested in only quantity of error.

| | | |

Example # 6:-
If ( ) , find the relative and percentage error in ( ) for , if error
in .
Solution:-
The error in computation of ( ) is given by

Dr. Jamil Book Series 2 Page 11


Advanced Mathematical Techniques Chapter # 1 Error Analysis & Difference Operators

( ) ( )
( )

( ) ( )
At , the absolute error is computation ( )

( ) ( ( ) )

and

( ) ( ) ( ) at

Therefore, percentage error is


( )
| |
( )

Example #7:-
The exponential function ( ) can be expand as

( )

Find , the number of terms, such that their sum yields the value of correct to 8 decimal
places at .
Solution:-
The error term (i.e., remainder term): is given by

At and for maximum absolute error, we put θ = 1, then

Then relative error in when , is given by

For an decimal accuracy at , we must have

Dr. Jamil Book Series 2 Page 12


Advanced Mathematical Techniques Chapter # 1 Error Analysis & Difference Operators

The least value of the that satisfy above equation is Thus, we need to take
terms of the exponential series in order that its sum is correct to decimal places.

Total Differential Coefficient:-


If ( ), then its total Differential coefficient is defined as:

With the help of total differential coefficient, we can calculate change, error, accuracy and
increment etc. For example in the language of Error analysis, we can write it as:

Now, we will discuss it applications in Error analysis in the following.

1.3.4 A General Error Formula


Let

( )

Be a function of several variables ( ) and let the error in each be .


Then the error in is given by total differential coefficient as

The formula for the relative error follows immediately, we get

Dr. Jamil Book Series 2 Page 13


Advanced Mathematical Techniques Chapter # 1 Error Analysis & Difference Operators

Example # 8:-

Find the Percentage Error in , where and

Solution:-
We have given

and

and we have to determine


From general formula of error, we have

where,

Then

In general, the error and may be or , and hence, we take the absolute
values of the terms on right side and using
(| | | | | | | | | | | || || |).
This gives

| | | |

| | | | | |

At and we have

( )

Since
( )( )

Then the relative error is given by

Dr. Jamil Book Series 2 Page 14


Advanced Mathematical Techniques Chapter # 1 Error Analysis & Difference Operators

Example # 9:-

If the kinetic energy , find the Maximum Error and Percentage Error in as
changes from 49 to 49.5 and changes from to .
Solution:-
We have given
and

and we have to determine


From general formula of error, we have

( ) ( )

In general, the error and may be or , and hence, we take the absolute
values of the terms on right side and using
(| | | | | | | | | || |).
This gives

| | |( ) ( ) |

|( ) | |( ) |=|( )| | | |( )|| |

( )
|( )| | | |( )( )||( )|

and
( )( )

Then the Percentage error is given by

Example # 10:-
The power required to propel a ship of length with a velocity is given by
where is constant. Find the Percentage Error in Power if percentage error in
velocity is and in length is .

Solution:-

Dr. Jamil Book Series 2 Page 15


Advanced Mathematical Techniques Chapter # 1 Error Analysis & Difference Operators

We have given
, and

In order to calculate Percentage Error in Power that is first we calculate

Absolute Error in as

For Relative in we have

Then the Percentage error is given by

( ) ( )

( ) ( )

( ) ( )

Example # 11:-
If , find the Maximum Percentage Error in when error in
respectively are equal to .

Solution:-
We have given
and .

and we have to determine


From general formula of error, we have

where,

Then

Dr. Jamil Book Series 2 Page 16


Advanced Mathematical Techniques Chapter # 1 Error Analysis & Difference Operators

In general, the error and may be or , and hence, we take the absolute
values of the terms on right side and using
(| | | | | | | | | | | || || |).
This gives

| | | |

| | | | | |

At and we have

( )( )( ) ( )( )( ) ( )( )( )
* +( ) * +( )* +( )

( )( ) ( )( ) ( )( )

and
( )( )( )

Then the Relative error is given by

and Percentage error is given by

Example # 12:-
Find the possible Percentage Error in computing the parallel resistance of these resistance
from the formula . If are each in the error by plus .

Solution:-
We have given

and we have to determine


From general formula of error, we have

Dr. Jamil Book Series 2 Page 17


Advanced Mathematical Techniques Chapter # 1 Error Analysis & Difference Operators

Since

( ) ( )

Similarly, we get

Then

( ) ( ) ( ) ( )

( ) ( ) ( ) ( )

( ) ( )( )

( ) ( )

Example # 13:-
The voltage across a resistor is measured with error , and the resistance is measured
with an error . Show that the error in calculating the power generated in the
resistor is ( ). If can be measured to an accuracy of and to an accuracy
of . What is the approximately possible Percentage Error in .
Solution:-
We have given

Dr. Jamil Book Series 2 Page 18


Advanced Mathematical Techniques Chapter # 1 Error Analysis & Difference Operators

, we have to show that: ( ) and we have to


determine when

From general error formula, we have

Since we have

( )

This prove the result. The above expression can be written as

( )

( )

Question # 14(HW):-
The Deflection at the centre of a road of length and the diameter of supported at its
ends and loaded at the centre with a weight varies as
( ).What is the Percentage
Error in the deflection corresponding to the percentage error in and of and
respectively.
Answer:-

Question # 15(HW):-
The diameter and altitude of a can in the shape of a right circular cylinder are measured as
and respectively. The possible error in each measurement is . Find
approximately the Maximum Percentage Error in the computed value for the Volume and
the Lateral Surface. (Note:- )
Answer:-
.

Dr. Jamil Book Series 2 Page 19


Advanced Mathematical Techniques Chapter # 1 Error Analysis & Difference Operators

1.4 Numerical stability & ill-condition


Sensitivity of the solution w.r.t. errors in the data
1.4.1 Well-conditioned or stability:-
A problem is well- conditioned if small errors in the data produce small errors in the
solution.
1.4.2 Ill-conditioned or instability:-
A problem is well- conditioned if small errors in the data produce large errors in the
solution.
1.4.3 The condition:-
The term condition is often used to describe the sensitivity of a function ( ) to small
changes in the argument . It is measured by the maximum relative change in ( ) due
to unit relative change/error in the argument.
Let ∆ ( ) be the increment/error in the ( ) due to small increment in . The
condition of ( ) at , may be expressed as:

( )
| |

( ) ( )
| |

( )
| ( )
|

Since

( ) ( )
( )
( )

( )
| |
( )

A large value of the condition indicates that ( ) is highly sensitive near , and it is termed
ill-condition.
Note that a function may be ill-condition near a particular point but well-conditioned at
other points.
Example #1:-
The function ( ) , is ill-conditioned near and well-conditioned at .

At :-
Since ( ) then

Dr. Jamil Book Series 2 Page 20


Advanced Mathematical Techniques Chapter # 1 Error Analysis & Difference Operators

( )
| | | | | |
( )

| | | | | |
( ) ( )

Which grows unbounded as . Hence ( ) is ill-conditioned at .

At :-

We have

| | | | | |
( ) ( )

Which is finite as . Hence ( ) is well-conditioned at

Example #2:-
Check whether ( ) and ( ) are well-conditioned or ill-conditioned at
and

Solution:-
1): For ( ) :-
We have ( ) , then
( )
| | | |
( )

| |

At :-
At , form therefore apply L’hopital theorem, we get

| | | |

| | | |

Hence ( ) is well-conditioned at .
At :-

| | | |

Hence ( ) is well-conditioned at

Dr. Jamil Book Series 2 Page 21


Advanced Mathematical Techniques Chapter # 1 Error Analysis & Difference Operators

2): For ( ) :-
We have ( ) , then
( )
| |
( )
( )
| | | |

| |

At :-
| |

Hence ( ) is well-conditioned at
At :-

| |

Hence ( ) is ill-conditioned at .

1.5 Numerical algorithms & Flow charts


1.5.1 Algorithms:-

An algorithm for solving a problem is a finite sequence of simple instruction which, when
followed yields the solution of the problem.

1.5.2 Flow charts:-


Graphical or diagrammatic representation of an algorithm is a flow chart, which its
representation in a programming language is a program.

1.5.3 Flowchart Keys:-

1. Start/End

2. Process or
task

3.
Decision
s

Dr. Jamil Book Series 2 Page 22


Advanced Mathematical Techniques Chapter # 1 Error Analysis & Difference Operators

Input/
4.
Output

5. Sequence

Example #1:-
Given a real number , to compute the absolute value | |.
Solution:-
Algorithm: Absolute Value Computation:-
Input :
Output : | |
Method :
1. Test if true, then , terminate.
Otherwise
2.

“Flow chart:- Absolute value computation”

Start

Input 𝒙

NO YES
𝒙 𝟎

𝑬 𝒙 𝑬 𝒙

Print 𝑬

End

Dr. Jamil Book Series 2 Page 23


Advanced Mathematical Techniques Chapter # 1 Error Analysis & Difference Operators

Example # 2:-
“Input Output flow chart”

Start

Input
Temp

NO YES
Temp 𝟑𝟐

Print “below Print “above


freezing” freezing”

End

Dr. Jamil Book Series 2 Page 24


Advanced Mathematical Techniques Chapter # 1 Error Analysis & Difference Operators

Example #3:-
“Order Processing”

Start

Receive order
via e-mail

Copy and paste e-mail


data into database.

Shipping YES
involved

Print invoice
NO and UPS lable.

Send e-mail to confirm


shipping.

Assemble Package and ship

End

Dr. Jamil Book Series 2 Page 25


Advanced Mathematical Techniques Chapter # 1 Error Analysis & Difference Operators

CHAPTER # 1
PART-II: DIFFERENCE OPERTORS
1.6.1 Introduction:-
Let us consider a function ( ) , defined in . Let us consider the consecutive
values of differing by equal length i.e.

OR

Let the corresponding values of the functions respective are:


( )

( ) ( )

( ) ( )

( ) ( )

( ) ( ( ) )

( ) ( )

Then the values of are called the arguments or nodes and the values of the function
( ) corresponding to the arguments are called entries. Let the following data, we
have given:

“Table of values”
. .
. .
. .

( ) ( ) ( ) ( ) . ( ) . ( ) ( ) ( )
. .
. .

Dr. Jamil Book Series 2 Page 26


Advanced Mathematical Techniques Chapter # 1 Error Analysis & Difference Operators

1.6.2: Difference Operators


1): Forward difference Operator :-
The first order forward difference operator is denoted by and defined by:

(What about )
The second order forward difference operator is defined by:

In the same way, the order forward difference operator is defined as follows:

In general,

Forward difference table”

Dr. Jamil Book Series 2 Page 27


Advanced Mathematical Techniques Chapter # 1 Error Analysis & Difference Operators

Important Note:-
In forward difference table the upper diagonal is conserved/constant with respect to initial
value
Example #1:-
Construct forward difference table for the following data:

( )

Solution:-
“Forward difference table”

We have from above table:


, , , , , and .

Example #2:-
Express and , in terms of the value of the function .
Solution:-
We have
( ) ( )
( )
and

Dr. Jamil Book Series 2 Page 28


Advanced Mathematical Techniques Chapter # 1 Error Analysis & Difference Operators

( ) ( )

( ) ( ) ( ) ( )

( )

Now, From ( ) and ( ) , we arrive at the following results:

( ) ( ) ( ) ( )

where

( )
( )

2): Backward difference operator :-


The first order backward difference operator is denoted by and defined by:

(What about )

The second order backward difference operator is defined by:

In the same way order backward difference operator is defined by:

In general,

Dr. Jamil Book Series 2 Page 29


Advanced Mathematical Techniques Chapter # 1 Error Analysis & Difference Operators

“Backward Difference table”

Important note:-
In forward difference table the lower diagonal is conserved/constant with respect to final
value .
Example #1:-
Construct backward difference table for the following data:

( )
Solution:-
“Backward difference table”

We have (Since here): , , , , ,

Dr. Jamil Book Series 2 Page 30


Advanced Mathematical Techniques Chapter # 1 Error Analysis & Difference Operators

Example#2 (H.W):-
Construct:
a): Forward difference table for:

( )
b): Backward difference table for:

( )

3): Central Difference operator :-


The first order central difference operator is denoted by and defined by:
=
=
=

=
Similarly, higher order central differences are defined as follows:
= -

= -
. .
=

Central Difference Table

Dr. Jamil Book Series 2 Page 31


Advanced Mathematical Techniques Chapter # 1 Error Analysis & Difference Operators

4): Shift operator E (or Translation or Displacement):-


The shift operator E is defined by

( ) ( ) ( ) ( )

Similarly, higher order shift operator is defined by

( ) ( ) ( ) ( )

( ) ( )

( ) ( )
In general

( ) ( )

We have
( ) ( ) ( )
, ,…, =

And
, and so on.
The inverse operator is defined as:

( ) ( )

and

( ) ( )

5): Average Operator :-


The average operator is defined as

( ) [ ( ) ( )]

Similarly,

( ) [ ( ) ( )]

6): Differential Operator :-


The differential operator is defined as
( )
( ) ( )

Dr. Jamil Book Series 2 Page 32


Advanced Mathematical Techniques Chapter # 1 Error Analysis & Difference Operators

( )
( ) ( )

( )
( ) ( )

( )
( ) ( )

7): Unit Operator 1:-


The unit operator is such that

( ) ( )

8): Zero Operator 0:-


The zero operator is such that

( )

1.6.3 Properties of Operators:-


1): Linear Property:-
The operators , , , , , and are all linear i.e., they satisfy:

( ) ( ) ( ) ( )

Examples:-
Differential and integral operators are linear. Since
( ) ( )
1): ( ) ( ) +

) ∫ ( ) ( ) ∫ ( ) + ∫ ( )

1): Distributive over addition property:-


These operators satisfy distributive over addition property, that is

( ) ( ) ( ) ( )

Dr. Jamil Book Series 2 Page 33


Advanced Mathematical Techniques Chapter # 1 Error Analysis & Difference Operators

1.6.4 “Relation among the operators”


1): Relation between and :-

By the definition of forward difference operator, we have:

( ) ( ) ( )

( ) ( ) ( )

( ) ( ) ( )

Thus

Here is unit operator.

2): Relation between and :-

By the definition backward difference operator,

We have

( ) ( ) ( )

( ) ( ) ( )

( ) ( ) ( )

Thus,

( ) ∫

( )

( )

Where is the unit operator.

3): Relation between and :-


By the definition of central difference operator, we have

( ) ( ) ( )

( ) ( ) ( )

( ) ( ) ( )

Thus

Dr. Jamil Book Series 2 Page 34


Advanced Mathematical Techniques Chapter # 1 Error Analysis & Difference Operators

Now

( )

and

( )

Hence

4): Relation between and :-


By the definition of average operator, we have

( ) [ ( ) (( )]

( ) [ ( ) ( )]

( ) [ ] ( )

Thus

[ ]

5): Relation between and :-


By the definition of differential operator, we have
( )
( ) ( )

( )
( ) ( )

And so on.
Using Taylor’s series expression, we have

( ) ( ) ( ) ( ) ( )

( ) ( ) ( ) ( )

( ) ( )
( ) * + ( )

( ) ( )

Dr. Jamil Book Series 2 Page 35


Advanced Mathematical Techniques Chapter # 1 Error Analysis & Difference Operators

Thus

Again

( )
Now consider,

( )

Since ( )

* +

Example# 1:-

Prove that:

( ) ( )

Proof:
We know that:
( )
Hence

( )

Now

[ ][ ]

( )

( )

( )

Hence

( ) ( )

Dr. Jamil Book Series 2 Page 36


Advanced Mathematical Techniques Chapter # 1 Error Analysis & Difference Operators

Example # 2:-
Show that the operators and are Commutative.
Proof:-
By the definition of operators and , we have

[ ] ( )

( ) * ( ) ( )+

While

[ ] [ ] ( )

Equating (1) and (2) , we have

Thus

Therefore, the operators and are commutative.

Example # 3:-
Prove that:

) ( )

) √

Proof (1):-
By the definition of operators, we have

[ ][ ]

Therefore

1+( ) ( ) 1+ ( )

Dr. Jamil Book Series 2 Page 37


Advanced Mathematical Techniques Chapter # 1 Error Analysis & Difference Operators

( ) ( )

Now, consider

1+ ( )

( )

( ) ( ) ( )

Equating (1) and (2) , we get

( )

Proof (2):-

By definition of operators, we have

( ) ( )

Proof (3):-

By definition of operator, we have

( ) √ ( )
( )

( )√

√( )
( )

( )( )

Dr. Jamil Book Series 2 Page 38


Advanced Mathematical Techniques Chapter # 1 Error Analysis & Difference Operators

Hence

Proof 4):-
We have

( )( )=

( ) * +

Proof (5):-
We have

( )( )=

( ) ( )

( ) ( )

Example # 4:-
Find:
)
)
Proof (a):-
By definition of forward operator, we have
( )

[ ]

Proof (b):-
By definition of forward operator, we have

( )

Since * +

[ ]
( )

[ ]

Dr. Jamil Book Series 2 Page 39


Advanced Mathematical Techniques Chapter # 1 Error Analysis & Difference Operators

Example # 5:-
Evaluate (taking 1 as the interval of differencing):

) [ ]

) [ ]

Solution (a):-

By the definition of forward operator, we have

( ) ( )
[ ] [ ]
( )( )

[ ] [ ]

[ ] [ ] [ ]

[ ] [ ] [ ]

Since interval of differencing is mean’s .

[ ] [ ]
( ) ( ) ( )( )

Solution (b):-
By the definition of forward operator, we have

[ ]
( )
Then

[ ]
( )( ) ( )

[ ]
( )( ) ( )

( )
[ ]
( )( )

Similarly, we can get

( )
[ ]
( )( ) ( )

Dr. Jamil Book Series 2 Page 40


Advanced Mathematical Techniques Chapter # 1 Error Analysis & Difference Operators

Example # 6:-
Evaluate:

( )

Solution:-
Let be the interval of differencing, then

( ) ( )

( ) ( )

( ) ( )

( ) ( )

( )

( ) ( ) ( )

( )

( )

Note:-
If , then

( )

Example # 7:-
Prove that: ( ) ( ) ( ) ( ) ( )
Proof:-
By the definition of forward operator, we have

( ) ( ) ( )

( ) ( ) ( ) ( ) ( ) ( ) ( )

( ) ( ) ( ) ( ) ( ) ( ) ( )

Dr. Jamil Book Series 2 Page 41


Advanced Mathematical Techniques Chapter # 1 Error Analysis & Difference Operators

( ) ( ) ( ) ( ) ( )

( ) ( ) ( ) ( ) ( )

Hence

( ) ( ) ( ) ( ) ( )

Example # 8:-

Given . Find

Solution:-

We have
( )

[ ( ) ( ) ( ) ]

Example # 9:-

Given Find

Solution:-

We have

( )

[ ( ) ( ) ( ) ( ) ]

[ ]

Substituting values, we get

Dr. Jamil Book Series 2 Page 42


Advanced Mathematical Techniques Chapter # 1 Error Analysis & Difference Operators

Hence

Example # 10:-
Prove that:
) ( ) ( ) ( ) ( ) ( ) ( )

( ) ( ) ( ) ( ) ( )
) [ ]
( ) ( ) ( )
Proof (a):-
By the definition of forward operator, we have

( ) ( ) ( ) ( ) ( ) ( )

( ) ( ) ( ) ( ) ( ) ( ) ( ) ( ) ( ) ( )

( ) ( ) ( ) ( ) ( ) ( ) ( ) ( )

Hence

( ) ( ) ( ) ( ) ( ) ( )

Proof (b):-

By the definition of forward operator, we have

( ) ( ) ( )
[ ]
( ) ( ) ( )

( ) ( ) ( ) ( ) ( )
[ ]
( ) ( ) ( )

( ) ( ) ( ) ( ) ( ) ( ) ( ) ( ) ( )
[ ]
( ) ( ) ( )

( ) ( ) ( ) ( ) ( ) ( ) ( )
[ ]
( ) ( ) ( )

Hence

( ) ( ) ( ) ( ) ( )
[ ]
( ) ( ) ( )

Dr. Jamil Book Series 2 Page 43


Advanced Mathematical Techniques Chapter # 1 Error Analysis & Difference Operators

EXCERCISE -1
Question # 1:-
Round off the following numbers correct up to 4-significant figures:
a): 2.56235 b): 0.34026 c): 4.50089
d): 0.000243468 e): 2456.782 f): 1.3456
g): 2.5555 h): 8.0008
Answer:-
a): 2.562 b): 0.3403 c): 4.501
d): 0.0002435 e): 2457 f): 1.346
g): 2.226 h): 8.001

Question # 2:-
Round-off the following numbers correct up to 3-decimal places
a): 2.46289 b): 0.46999 c): 0.0035869
d): 0.0015 e): 1.46294 f): 1.35008
Answer:-
a): 2.463 b): 0.470 c): 0.004
d): 0.002 e): 1.463 f): 1.350

Question # 3:-
The following numbers are correct up to last digit. Find the sum
a): 1.3526, 2.00462, 1.532, 28.201, 31.0012
b): 0.532, 7.46571, 1.501, 3.62102
c): 5.2146, 20.12, 11.2356, 1.8948
Answer:-
a): 64.091 b): 13.120 c): 38.46
Question # 4:-
If are true value, approximate value, absolute error, relative error
and percentage error respectively:
a): Given . Find

b): Given Find


c): Given . Find

Answer:-
a): ,
b):
c):

Dr. Jamil Book Series 2 Page 44


Advanced Mathematical Techniques Chapter # 1 Error Analysis & Difference Operators

Question # 5:-
Find the Relative Error in computation of for having
absolute errors respectively.
Answer:-

Question # 6:-
If 5/6 be represented approximately by 0.8333, find
a): Absolute error
b): Relative error
c): Percentage error and also verify theorem-1 and theorem-2.
Answer:-
.00004,

Question # 7:-
Find the Relative Percentage Error in approximate representation of 4/3 by 1.33.
Answer:-

Question # 8:-
Let a number 345.26132 be rounded to 345.261 correct to 3-decimal places, then find
1): Absolute error
2): Relative error
3): Percentage errors and also verify theorem-1 and theorem-2.
Answer:-

Question # 9:-
Write down the approximate representation of 2/3 correct to 4 significant figures and then
find:
1): Absolute error
2): Relative error
3): Percentage error and verify theorem-1 and theorem-2
Answer:-

Dr. Jamil Book Series 2 Page 45


Advanced Mathematical Techniques Chapter # 1 Error Analysis & Difference Operators

Question # 10:-
Find the number of Significant Figures in:
a): given its absolute error as
b): given its relative error as
c): given its absolute error as
Answer:-
a): 4-significant figures
b): 4-significant figures
c): 2-significant figures

Question # 11:-
If be the absolute error in , find the
Relative Error in computation of .
Answer:-

Question # 12:-
Determine the number of correct Significant Digits in the numbers given its absolute
error
a):
b):
c):
Answer:-
a): 2
b): 3
c): 4

Question # 13:-
Determine the number of correct digits in the number given its relative error
a):
b):
c)
Answer:-
a): b): c):

Question # 14:-
Find the number of Significant Figures in , given its relative error as
.

Dr. Jamil Book Series 2 Page 46


Advanced Mathematical Techniques Chapter # 1 Error Analysis & Difference Operators

Answer:-
3-signifcant figures

Question # 15:-
If , find the Percentage Error in at if the error in is
Answer:-

Question # 16:-
If ( ) , find the Relative Percentage Error in ( ) for if error in

Answer:-

Question # 17:-

Find the Percentage Relative Error in , where and

Answer:-

Question # 18:-

The period of a simple pendulum is √ . Find the Maximum Percentage Error in


due to possible error up to in and in .
Answer:-
( )(Note: +ve For Maximum and –ve For Minimum)

Question # 19:-

If the kinetic energy , find the Maximum Error and Percentage Error in as
changes from 49 to 49.5 and changes from to .
Answer:-
Error in

Question # 20:-
The power required to propel a ship of length with a velocity is given by
where is constant. Find the Percentage Error in Power if percentage error in

Dr. Jamil Book Series 2 Page 47


Advanced Mathematical Techniques Chapter # 1 Error Analysis & Difference Operators

velocity is and in length is .

Answer:-
Percentage increase in =

Question # 21:-
Suppose is a function of five different measurable quantities and is given by

Find an upper limit to the Relative Error in measure of


Note:- The upper limit of is if or
Answer:-
The upper limit of the relative error in is: | | | | | | | | | | that is

| | | | | | | | | |

Question # 22:-
The exponential function ( ) can be expanded as

( )
Find , the number of terms, such that their sum yields the value of correct up to 9
decimal places at .
Answer:-
We need to take terms of the exponential series in order that its sum is correct to 9
decimal places.

Question # 23:-
Check whether the function ( ) is well-conditioned or ill-conditioned at and

Answer:-
At :-

Well-conditioned, since the condition= at

At

Ill-conditioned since condition as ⁄

Question # 24:-

Check whether the function ( ) and ( ) are well-conditioned or ill-


conditioned at

Dr. Jamil Book Series 2 Page 48


Advanced Mathematical Techniques Chapter # 1 Error Analysis & Difference Operators

Answer:-
For ( ) :-
Well-conditioned at and
Well-conditioned at

For ( ) :-
Well-conditioned at and
Ill-conditioned at

Question # 25:-
Write:
1): Algorithm
2): Draw flow charts for each of the following:
a): Absolute value computation
b): Input and output flow chart
c): Order processing
Question # 26:-
Draw a flow chart of (Factorial):
Answer:-
“Flow chart of Factorial”

Dr. Jamil Book Series 2 Page 49


Advanced Mathematical Techniques Chapter # 1 Error Analysis & Difference Operators

Question # 27:-
Draw a flow chart of software development.
Answer:-
Flow chart of software development

Dr. Jamil Book Series 2 Page 50


Advanced Mathematical Techniques Chapter # 1 Error Analysis & Difference Operators

Question # 28:
Draw a flow chart of “Accounts Receivable.”
Answer:
“Flow chart of Accounts Receivable”

Question # 29:-
Construct:
a): Forward difference table for:

0 2 4 6 8
( ) 40 51.68 67.04 86.56 110.72

b): Backward difference table for:

10 20 30 40 50
( ) 1 1.3010 1.4771 1.6021 1.6990

Question # 30:-
Construct:

Dr. Jamil Book Series 2 Page 51


Advanced Mathematical Techniques Chapter # 1 Error Analysis & Difference Operators

a): Forward Difference table for:

0.20 0.22 0.24 0.26 0.28 0.30

( ) 1.6596 1.6698 1.6804 1.6912 1.7024 1.7139

b): Central difference table for:

0 1 2 3 4 5 6
( ) 0 7 26 63 124 342 511

c): Backward difference table for:

0.1 0.2 0.3 0.4 0.5


( ) 2.74560 2.82922 2.97427 3.18993 3.49034

Question # 31:-
Construct:
a): Forward difference table for:

2.0 2.1 2.2 2.3 2.4 2.5 2.6


( ) 0.6020 0.6128 0.6232 0.6335 0.6434 0.6532 0.6628

b): Central difference table for:

0.00 0.50 1.00 1.50 2.00 2.50 3.00


( ) 0.000 0.191 0.341 0.433 0.477 0.494 0.499

c): Backward difference table for:

3.1 3.2 3.3 3.4 3.5 3.6 3.7


( ) 0.49136 0.50515 0.51851 0.53148 0.54407 0.55630 0.56820

Question # 32:-
Express and in terms of the values of the function
Answer:-

Question # 33:-
Express and in terms of the values of the function .
Answer:-

Dr. Jamil Book Series 2 Page 52


Advanced Mathematical Techniques Chapter # 1 Error Analysis & Difference Operators

Question # 34:-
Prove that:

a):
)
) ( ) ( )

Question # 35:-
Show that the operator and Commute.

Question # 36:-
Prove that:

a): 1+ ( )

) √

) ( )( )
)

Question # 37:-
Prove that:
)

) √

) ( ) ( )

) +

) ( )

)
)

Dr. Jamil Book Series 2 Page 53


Advanced Mathematical Techniques Chapter # 1 Error Analysis & Difference Operators

Question # 38:-
Prove that:
) ( )

) ( )

Question # 39:-
Evaluate:
) ( )
) ( )
)
) [ ]
Answer:-

) ( )

)
( ) ( )

) ( )

) ( )

Question # 40:-
Prove that:
) ( ) ( ) ( ) ( ) ( ) ( )

( ) ( ) ( ) ( ) ( )
) * +
( ) ( ) ( )

( )
) [ ]
( ) ( ) ( )

) ( ) ( ) taking

Question # 41:-
Prove that:

) √

Dr. Jamil Book Series 2 Page 54


Advanced Mathematical Techniques Chapter # 1 Error Analysis & Difference Operators

) ( )

Question # 42:-
Evaluate (Taking 1 as the interval of differencing):

a): * +

) [ ]

Answer:-

) [ ]
( ) ( ) ( ) ( )
( )
) [ ]
( )( ) ( )

Question # 43:-

Evaluate: ( )

Answer:-

( )

Question # 44:-
Prove that: ( ) ( ) ( ) ( ) ( )

Question # 45:-
Given . Find
Answer:-

Question # 46:-
Given Find
Answer:-

Question # 47:-
Taking 1 as the interval of differencing, Prove that:

( )
Question # 48:-
Given: , , , and .
Show that .

Dr. Jamil Book Series 2 Page 55


Advanced Mathematical Techniques Chapter # 1 Error Analysis & Difference Operators

SOME APPLIED PROBLEMS OF CHPTER # 1


Question # 49:-
The discharge Q over a notch for head H is calculated by the formula ,where
is a given constant. If the head is and an error of possible in its
measurement, estimate the percentage error in computing the discharge.
Answer:-

Question # 50:-
If ( ). Find the Percentage Error in at , if the percentage error in is .

Answer:-

Question # 51:-
The error in the measurement of the Area of the circle is not allowed to exceed . How
accurately should the Diameter be measured?
Answer:-

Question # 52:-
If , find the Maximum Percentage Error in when error in
respectively are equal to .

Answer:-

Question # 53:-
If and error in are respectively, at
. Calculate the Absolute and Percentage Error in the calculation of it.
Answer:-

Question # 54:-
Find the number of terms of the Exponential Series such that their sum gives the value of
correct to decimal places for all the value of in the range .
Answer:-
The number of terms in the exponential series should be

Dr. Jamil Book Series 2 Page 56


Advanced Mathematical Techniques Chapter # 1 Error Analysis & Difference Operators

Question # 55:-
Find the possible Percentage Error in computing the parallel resistance of these resistance
from the formula . If are each in the error by plus .

Answer:-

Question # 56:-
The Deflection at the centre of a road of length and the diameter of supported at its ends
and loaded at the centre with a weight varies as
( ).What is the Percentage
Error in the deflection corresponding to the percentage error in and of and
respectively.
Answer:-

Question # 57:-

The Power dissipated in a resistor is given by . Find the Percentage Error in when
percentage error in is and is .
Answer:-
.

Question # 58:-
The diameter and altitude of a can in the shape of a right circular cylinder are measured as
and respectively. The possible error in each measurement is . Find
approximately the Maximum Percentage Error in the computed value for the Volume and
the Lateral Surface. (Note:- )
Answer:-
.

Question # 59:-
In determining the Specific Gravity by the formula , where is the weight in air and
is the weight in water. can be read within and within . Find the
Maximum Error in if the error are and .
Answer:-
.
Question # 60:-

Dr. Jamil Book Series 2 Page 57


Advanced Mathematical Techniques Chapter # 1 Error Analysis & Difference Operators

The voltage across a resistor is measured with error , and the resistance is measured
with an error . Show that the error in calculating the power generated in the

resistor is ( ). If can be measured to an accuracy of and to an accuracy


of . What is the approximately possible Percentage Error in .
Answer:-
error in

Question # 61:-
Find the possible Percentage Error in computing parallel resistance of two resistance
and from the formula , where and are both in error by each.

Answer:-

Dr. Jamil Book Series 2 Page 58


Advanced Mathematical Techniques Chapter # 1 Error Analysis & Difference Operators

SUMMARY OF THE CHAPTER


Nature or types of errors:-
1): Inherent or propagation error,
2): Truncation errors,
3): Round-off errors.
4): Significant error
The infinite series expansion of in term of Maclaurin’s series is given by

( )

( ) ( )

Absolute, Relative and Percentage Error:-

| |

| |

| |

Theorem - 1:-
If a number be rounded to decimal places, the absolute error is given by

Theorem - 2:-
If a number be rounded to correct significant figures, then the relative error is given by

( )

A General Error Formula:-

( )

Dr. Jamil Book Series 2 Page 59


Advanced Mathematical Techniques Chapter # 1 Error Analysis & Difference Operators

Note:-
1): | | | | | | | |
2): | | | || || |
Numerical stability & ill-condition:-
Well-conditioned or stability:-
A problem is well- conditioned if small errors in the data produce small errors in the
solution.
Ill-conditioned or instability:-
A problem is well- conditioned if small errors in the data produce large errors in the
solution.
The condition:-

( )
| |
( )
Algorithms:-

An algorithm for solving a problem is a finite sequence of simple instruction which, when
followed yields the solution of the problem.

Flow charts:-
Graphical or diagrammatic representation of an algorithm is a flow chart, which its
representation in a programming language is a program.

Difference Operators:-

1): Forward difference Operator :-

= Increase-Copy

2): Backward difference operator :-

= Copy-Decrease

3): Central Difference operator :-

4): Shift operator E (or Translation or Displacement):-

( ) ( ) ( ) ( )

Dr. Jamil Book Series 2 Page 60


Advanced Mathematical Techniques Chapter # 1 Error Analysis & Difference Operators

5): Average Operator :-

( ) [ ( ) ( )]

6): Differential Operator :-


( )
( ) ( )

7): Unit Operator 1:-

( ) ( )

8): Zero Operator 0:-

( )

Properties of Operators:-
1): Linear Property
2): Distributive over addition property

Relation among the operators

( )

[ ]

* +

Dr. Jamil Book Series 2 Page 61


Advanced Mathematical Techniques Chapter # 1 Error Analysis & Difference Operators

HOME WORK PROBLES


FROM CHPTER#1
1): FOR ODD ROLL NUMBERS:-
Solve the following Problems On Your Class Register:-
Week-1: Q#: 3, 7, 13, 19, 21, 27.
Week-2: Q#: 31, 33, 37, 39(b,d), 41.
Week-3: Q#: 43, 45, 51, 55, 57, 59.
2): FOR EVEN ROLL NUMBERS:-
Solve the following Problems On Your Class Register:-
Week-1: Q#: 4, 8, 12, 18, 20, 22, 24
Week-2: Q#: 26, 28, 30, 34, 36, 38(c, d),
Week-3: Q#: 40(b, c), 42(b), 46, 52, 56, 58, 60.

Deadline:
The day completion of CH#1 + 2 DAYS(MAX. SUNDAY 3rd WEEK)

Dr. Jamil Book Series 2 Page 62


Advanced Mathematical Techniques Chapter # 1 Error Analysis & Difference Operators

SEMESTER ASSIGNMET
FROM CHPTER#1
1): FOR GROUP A:-
Solve the following Problems:-
Q#: 1, 5, 9, 13, 17, 21, 25, 29, 33, 37, 41, 45, 49, 53, 57, 61.
2): FOR GROUP B:-
Solve the following Problems:-
Q#: 2, 6, 10, 14, 18, 22, 26, 30, 34, 38, 42, 46, 50, 54, 58.
3): FOR GROUP C:-
Solve the following Problems:-
Q#: 3, 7, 11, 15, 19, 23, 27, 31, 35, 39, 43, 47, 51, 55, 59.
4): FOR GROUP D:-
Solve the following Problems:-
Q#: 4, 8, 12, 16, 20, 24, 28, 32, 36, 40, 44, 48, 52, 56, 60.

OPEN END PROBLEMS(OPE)


EVERY STUDENTS HAS TO SOLVE ON MS-WORD SOME APPLIED PROBLEMS
ON THE TOPIC THAT ASSIGN IN NEXT WEEK FROM THE COURSE MT-441 AMT.
DEADLINE:-
MONDAY OF 9th WEEK FOR CH#1, 2, 3.
TUESDAY OF THE 15TH WEEK FOR CH#4, 5 & OPE.

Dr. Jamil Book Series 2 Page 63


Advanced Mathematical Techniques Chapter#2 Interpolation & Curve Fitting

CHAPTER # 2
INTERPOLATION
&
CURVE FITTING

Dr. Jamil Book Series 2 Page 1


Advanced Mathematical Techniques Chapter#2 Interpolation & Curve Fitting

CHAPTER # 2
PART-I: INTERPOLATION
2.1 Introduction:-
In this chapter we will discuss the problem of approximating a given function by a class of
simpler function especially polynomial. Let us suppose a function ( ) ( ) is not
known explicitly and only the values of ( ) at a set of points are known.
Let ( ) is known at the distinct points , , ,...., i.e.
“Table of values”

2.1.1 Interpolation:-
Interpolation is the process of finding the value of a function for any value of argument or
independent variable in an interval ( )
Thus interpolation is the art of reading between the lines in a given table.

2.1.2 Extrapolation:-
Extrapolation is the process of finding the value of a function outside an interval
( )

2.1.3 Polynomial Interpolation ( ):-


Approximation by the polynomial of least degree satisfying the given data is called
Polynomial Interpolation and the corresponding polynomial is called the interpolating
polynomial.
The following theorem guarantees the existence and uniqueness of such a polynomial.

2.1.4 Theorem:-
Let ( ) ( ) be known at the distinct points . Then there exists a unique
polynomial ( ) of degree less than or equals to and satisfying the condition:

( ) ( )

2.2 Lagrange’s Interpolation Polynomial Formula


Let ( ) are known at the distinct points i.e. there are ( ) points are
given as in the following table:
“Table of values”

( ) ( ) ( ) ( ) ( ) ( )

Dr. Jamil Book Series 2 Page 2


Advanced Mathematical Techniques Chapter#2 Interpolation & Curve Fitting

The Lagrange’s interpolation polynomial formula of degree at most ( ) is given by:

( ) ( ) ( ) ∑ ( )

where ( ) are called Lagrange’s interpolation coefficients and defined as:

( )( ) ( )( ) ( )
( ) [ ( )]
( )( ) ( )( ) ( )

Example # 1:-
Determine the Lagrange’s interpolation polynomial for the data:

( ) 5

and estimate the value of ( ) at .

Solution :-
The Lagrange’s interpolation polynomial formula of degree at most 2 is given by (here
):

( ) ( ) ( ) ∑ ( )

( ) ( ) ( ) ( ) ( ) ( ) ( )
Now
( )( ) ( )( )
( ) = = ( )
( )( ) ( )( )

( )( ) ( )( )
( ) = = ( )
( )( ) ( )( )

( )( ) ( )( )
( ) = ( )
( )( ) ( )( )

Substituting values in Eq. (1), we get

( ) ( )( ) ( )( ) ( )( )

( ) ( )

Dr. Jamil Book Series 2 Page 3


Advanced Mathematical Techniques Chapter#2 Interpolation & Curve Fitting

is the required Lagrange’s interpolation polynomial. Also

( ) ( ) ( ) ( ) ( )

Example # 2:-
Find the missing value from the following data using Lagrange’s interpolation formula:

( ) ( )

Solution:-
Consider the following data:

( ) ( )

The Lagrange’s interpolation polynomial formula of degree at most 3 is given by (here


):

( ) ( ) ( ) ∑ ( )

( ) ( ) ( ) ( ) ( ) ( ) ( ) (1)

Now
( )( )( ) ( )( )( )
( )
( )( )( ) ( )( )( )

( ) ( )

( )( )( ) ( )( )( )
( ) ( )
( )( )( ) ( )( )( )

( )( )( ) ( )( )( )
( ) ( )
( )( )( ) ( )( )( )

Dr. Jamil Book Series 2 Page 4


Advanced Mathematical Techniques Chapter#2 Interpolation & Curve Fitting

( )( )( ) ( )( )( )
( ) ( )
( )( )( ) ( )( )( )

Substituting these values in Eq. (1), we get

( ) ( ) ( )( ) ( )( )

( )( )

( ) ( )

Hence

( ) ( ) ( ) ( ) ( ) ( )

Example # 3:-
Find ( ) using Lagrange’s interpolation polynomial:

( )

Solution:-
The Lagrange’s interpolation polynomial formula of degree at most 3 is given by (here
):

( ) ( ) ( ) ∑ ( )

Here , therefore we have

( ) ( ) ( ) ∑ ( ) ( ) ( ) ( ) ( ) (1)

Now

( )( )( ) ( )( )( )
( ) = =
( )( )( ) ( )( )( )

Dr. Jamil Book Series 2 Page 5


Advanced Mathematical Techniques Chapter#2 Interpolation & Curve Fitting

( )( )( ) [ —( )]( )( )
( ) = =
( )( )( ) , ( )-( )( )

( )( )( ) [ —( )]( )( )
( ) = =
( )( )( ) , ( )-( )( )

( )( )( ) , —( )-( )( )
( ) = =
( )( )( ) , ( )-( )( )

Substituting these values in equ (1), we get

( ) ( ) ( ) ( )( ) ( )( ) ( )( ) ( )( )

2.3 Hermite’s Interpolation Polynomial Formula

Let ( ) and ( ) are known at the distinct points i.e there are ( )
points are given as in the following table:
“Table of values”

( ) ( ) …

( ) ( ) …

The Hermite’s interpolation polynomial formula of degree at most ( ) is given by


(without proof):

( ) ( ) ( ) ∑ ( ) ∑ ( )

where

( ) , ( )( )-, ( )-

( ) ( ), ( )-

where and ( ) are derivatives of ( ) and ( ) respectively.

Dr. Jamil Book Series 2 Page 6


Advanced Mathematical Techniques Chapter#2 Interpolation & Curve Fitting

Example # 1:-
Using Hermite’s interpolation formula find ( ) ( ), and obtain ( ) and
( ) from the following table:

( )

( )

Solution:-
The Hermite’s interpolation polynomial formula of degree at most 5 is given by (here
)

( ) ( ) ( ) ∑ ( ) ∑ ( )

where

( ) , ( )( )-, ( )-

( ) ( ), ( )-

we have:
( ) ( ) ( ) ( ) ( ) ( ) ( ) ( )
( ) (1)

( ) , ( )( )-, ( )-

( ) , ( )( )-, ( )-

( ) , ( )( )-, ( )-

( ) ( ), ( )-

( ) ( ), ( )-

( ) ( ), ( )-

Dr. Jamil Book Series 2 Page 7


Advanced Mathematical Techniques Chapter#2 Interpolation & Curve Fitting

Now we calculate ( ) and ( )

( )( ) ( )( )
( ) ( )
( )( ) ( )( )

( ) ( ) ( ) ( )

( )( ) ( )( )
( )
( )( ) ( )( )

( ) ( ) ( )

( )( ) ( )( )
( ) ( )
( )( ) ( )( )

( ) ( ) ( ) ( )

Substituting these values in . We have

( ) ( )
( ) [ ( )( )] ( )

( )

( ) , ( )( )-( )

( ) ( )
( ) [ ( )( )] ( )

( ) ( )

( )
( ) ( ) ( )

( ) ( )( )

Dr. Jamil Book Series 2 Page 8


Advanced Mathematical Techniques Chapter#2 Interpolation & Curve Fitting

( )
( ) ( ) ( )

Using above values in Eq. (1), we get

( ) ( )( ) ( )( )

( )( ) ( )( )

( )( ) ( )( )

( ) ( )

Hence

( ) ( ) ( ) ( ) ( )

( ) ( ) ( ) ( ) ( )

Example # 2:-
From the following data find ( ) using Hermite’s interpolation formula and hence
determine ( ) ( ).

( ) ( )

Solution:-
The Hermite’s interpolation polynomial formula of degree at most 5 is given by (here
)

( ) ( ) ( ) ∑ ( ) ∑ ( )

where

( ) , ( )( )-, ( )-

( ) ( ), ( )-

we have:

Dr. Jamil Book Series 2 Page 9


Advanced Mathematical Techniques Chapter#2 Interpolation & Curve Fitting

( ) ( ) ( ) ( ) ( ) ( ) ( ) ( )
( ) (1)
Since the values of given are same as in the (Example#1) there is no change in
and . Hence

( ) ( ) ( )
( )( ) ( )( )

( )( ) ( )( )

( )( ) ( )( )

( ) ( ) ( ) ( )

Therefore

( ) ( ) ( ) [ ( ) ( ) ( ) ( ) ( )]

( ) ( ) ( )

Example # 3 (HW):-
Using Hermite’s interpolation, find a cubic polynomial from the data below, also find
( ) ( )
a):-

( )

( )

Answer:-
( ) ( ) ( )

Example # 4 (HW):-
Using Hermite’s interpolation, find ( ) given in the table below and obtain ( )
( ).
b):-

0 1
( ) 1 1
( ) 0 1

Answer:-
( )

Dr. Jamil Book Series 2 Page 10


Advanced Mathematical Techniques Chapter#2 Interpolation & Curve Fitting

2.4 Newton’s Interpolation Polynomial Formulae

2.4.1 Newton’s forward Interpolation Polynomial formula:-


Consider

( ) ( ) ( ) ( ) ( )

( ) ( )

Using binomial formula, we have

( ) ( )( ) ( )( ) ( )
( )

where

This is called Newton’s or Newton’s-Gregory forward interpolation formula.

2.4.2 Newton’s backward Interpolation Polynomial formula:-


Consider

( ) ( ) ( ) ( ) ( )

( ) ( ) ( )

Again using binomial formula for negative power, we have

( ) ( )( ) ( )( ) ( )
( )

where

This is called Newton’s or Newton’s-Gregory backward interpolation formula.

Important note:-
a): Newton’s forward interpolation formula is used for interpolating the values of the
function near the beginning of the table.

b): Newton’s backward interpolation formula is used for interpolating the values of the
function near the end of the table.

Dr. Jamil Book Series 2 Page 11


Advanced Mathematical Techniques Chapter#2 Interpolation & Curve Fitting

Example # 1:-
Estimate ( ) from the following data using Newton’s forward interpolation formula:

3.1 3.2 3.3 3.4 3.5


( ) 0 0.6 1.0 1.2 1.3

Solution:-
“Forward Difference Table”
2 3 4
y y y
3.1 0
0.6
3.2 0.6 -0.2
0.4 0
3.3 1.0 -0.2 0.1
0.2 0.1
3.4 1.2 -0.1
0.1
3.5 1.3

From the above table, we have


, , ,
Here , and

As the point is near the beginning of the table


we use the Newton’s forward interpolation formula.

( ) ( )( ) ( )( )( )
( )

Substituting values, we get

( )
( ) ( )( ) ( )
( )( ) ( )( )( )
( ) ( )

( ) ( ) ( )

Thus
( ) ( ) ( )

Dr. Jamil Book Series 2 Page 12


Advanced Mathematical Techniques Chapter#2 Interpolation & Curve Fitting

Example # 2:-
Find the Newton’s forward interpolation polynomial for the data:

-1 0 1 2
( ) ( ) 4 2 2 4

and estimate ( ).
Solution:-
“Forward Difference Table”

-1 4
-2
0 2 2
0 0
1 2 2
2
2 4

From the above table, we have


, ,

Here , , , then

The Newton’s forward interpolation polynomial formula is given as

( ) ( )( )
( )

( )
( ) ( )( ) ( )

( ) ( )
Hence

( ) ( )

is the required Newton’s forward interpolation polynomial and

( ) ( ) ( ) ( )

Dr. Jamil Book Series 2 Page 13


Advanced Mathematical Techniques Chapter#2 Interpolation & Curve Fitting

Example # 3:-
Estimate ( ) from the following data using Newton’s backward interpolation formula

20 25 30 35 40 45
( ) ( ) 354 332 291 260 231 204

Solution:-
“Backward Difference Table”

204

From the table above, we have

, , , ,

Here , , and , then

As the point is near the end of the table. We use Newton’s backward interpolation
formula

( ) ( )( )
( )

( )( )( ) ( )( )( )( )

Dr. Jamil Book Series 2 Page 14


Advanced Mathematical Techniques Chapter#2 Interpolation & Curve Fitting

Substituting values, we get

( )( ) ( )( )( )
( ) ( )( ) ( ) ( )

( )( )( )( ) ( )( )( )( )( )
( ) ( )

( ) ( ) ( )

Example # 4:-
Given the following table

1 2 3 4 5 6 7 8
( ) 1 8 27 64 125 216 343 512

Find a): ( ) b): ( )


Solution:-
“Forward/Backward Difference Table”

a): For ( ):-


To compute ( ), we use Newton’s-forward interpolation formula, as the point
is near the beginning of the table, therefore we have

Dr. Jamil Book Series 2 Page 15


Advanced Mathematical Techniques Chapter#2 Interpolation & Curve Fitting

( ) ( )( )
( )

Here , and , then

Substituting these values in above formula, we get

( )( ) ( )( )( )
( ) ( )( ) ( ) ( )

( )

Hence

( ) ( ) ( )

b): For ( ):-

To compute ( ), we use Newton’s-backward interpolation formula, as the point


is near the end of the table, therefore we have

( ) ( )( )
( )

Here , and , then

Substituting these values,we get

( )( ) ( )( )( )
( ) ( )( ) ( ) ( )

( )

Hence

( ) ( ) ( )
Dr. Jamil Book Series 2 Page 16
Advanced Mathematical Techniques Chapter#2 Interpolation & Curve Fitting

Example # 5(HW):-
Evaluate ( ) and ( ) from the table:

0 1 2 3 4 5
( ) ( ) 0 3 8 15 24 35

Answer:-
( ) ( )

Example # 6(HW):-
Compute ( ) and ( ) from the table:

0 1 2 3 4 5 6 7
( ) ( ) 0 7 26 63 124 215 342 511

Answer:-
( ) ( )

Dr. Jamil Book Series 2 Page 17


Advanced Mathematical Techniques Chapter#2 Interpolation & Curve Fitting

The DIVIDED DIFFRENCES


Let the values of ( ) ( ) be given at ( ) distinct points . We
define the Newton’s divided differences as:

( ) ( )
, -

( ) ( )
, -

. . .
. . .
. . .

( ) ( )
, -

These are called first divided differences and is devoted by . Higher divided differences
operators are defined as:

, - , -
, -

, - , -
, -

, - , -
, -

. . . . .
. . . . .
. . . . .

, - , -
[ ] for

These differences are denoted by .

Dr. Jamil Book Series 2 Page 18


Advanced Mathematical Techniques Chapter#2 Interpolation & Curve Fitting

“Divided Difference Table”

( )

( ) ( )
, -

, - , -
( ) , -

( ) ( ) , - , -
, - , -

( )
, - , -
, -

( ) ( )
, -
( )

Example:-
Make divided difference table for the data:

-1 0 1 3
( ) ( ) 2 1 0 -1

Solution:-
“Divided Difference Table”

( )

( ) , -

( )
( ) ( ) , -

, - ( ) , -

( )
( )
, -

, -

( )

Dr. Jamil Book Series 2 Page 19


Advanced Mathematical Techniques Chapter#2 Interpolation & Curve Fitting

2.4.3 Newton’s divided difference interpolation formula:-


The Newton’s divided difference interpolation polynominal formula is give by (without
proof):

( ) ( ) ( ) ( ) , - ( )( ) , -

( )( )( ) , - ( ) ( ) , -

Example # 1:-
For the data:
-1 0 2 5
( ) ( ) 7 10 22 235

Find the divided difference polynominal and estimate ( )


Solution:-
“Divided difference table”

-1 7
3
0 10 1
6 2
2 22 13
71
5 235

From the above table, we have


( ) , - , - , -

The Newton’s divided difference interpolation polynominal formula is given by

( ) ( ) ( ) , - ( )( ) , -

( )( )( ) , -

Substituting values, we get

( ) ( )( ) ( )( )( ) ( )( )( )( )

( ) ( ) ( )

Hence

( ) ( ) ( ) ( ) ( )
Dr. Jamil Book Series 2 Page 20
Advanced Mathematical Techniques Chapter#2 Interpolation & Curve Fitting

Example # 2:-
Using Newton’s divided difference formula find the value of ( ), ( ) and ( ) given the
following table:
4 5 7 10 11 13
( ) ( ) 48 100 294 900 1210 2028

Solution:-
“Divided difference table”:-

4 48
52
5 100 15
97 1
7 294 21 0
202 1 0
10 900 27 0
310 1
11 1210 33
409
13 2028
From the above table, we have

( ) , - , - , -
, - , -

The Newton’s divided difference interpolation polynomial formula is given by:

( ) ( ) ( ) ( ) , - ( )( ) , -

( )( )( ) , -

Substituting values we get

( ) ( ) ( )( ) (( )( )( ) ( )( )( )( )

( ) ( ) ( ) ( )( ) ( )( )( )
Then

( ) ( )

( ) ( ) ( ) ( )( ) ( ) ( )( )

Dr. Jamil Book Series 2 Page 21


Advanced Mathematical Techniques Chapter#2 Interpolation & Curve Fitting

( ) ( ) ( ) ( )( ) ( )( )( )

Example # 3(HW):-
Find the Newton’s divided difference polynominal of the following data, and find ( )

0 1 3 4
( ) ( ) 1 4 40 85

Ans:-
( ) ( ) ( )

Example # 4(HW):-
Find the missing values in the following data using Newton’s divided
difference formula:

3 6 7 8 9 10
( ) ( ) 168 - 120 - 72 63

Answer:-
( ) ( )

Dr. Jamil Book Series 2 Page 22


Advanced Mathematical Techniques Chapter#2 Interpolation & Curve Fitting

2.4 Cubic Spline Interpolation


2.4.1 Introduction:-
We are given the ( ) data points ( )

Our aim is to find the value of corresponding to , where


, by using a smooth polynomial curve. We have already seen many methods. A
recent one which is becoming important is the spline-fitting.

2.4.2 Assumption for Cubic Spline Polynomial ( ):-


1): ( ) is a polynomial of degree one for and

2): ( ) is at most a cubic polynomial in each interval ( )

3): ( ) ( ) and ( )are continuous at each point ( )

4): ( )

2.4.3 Cubic Spline Interpolation Polynomial Method:-


The cubic spline interpolation polynomial ( ) in the interval ( ) is given by

( ) ,( ) ( ) - ( )[ ]

( )[ ], (1)

, - (2)

And

(3)

Equations (2) and (3) give ( ) equations in ( ) unknowns, .


Hence, we can solve for , substituting in (1), we get the cubic spline in
each interval.
2.4.4 Note:-
Some authors do not assume linearity of ( ) for and .In that case
will not be zero.

Dr. Jamil Book Series 2 Page 23


Advanced Mathematical Techniques Chapter#2 Interpolation & Curve Fitting

Examples

Points Range of Eq. (1) Range of Eq. (2)

Example #1:-
From the following table:

1 2 3

( ) -8 -1 18

Compute ( )and ( ) ,using cubic spline.


Solution:-
Here , then We have

, -

For , we get:-

, -

, ( ) -

The cubic spline interpolation polynomial is given by

( ) ,( ) ( ) - ( )[ ]

( )[ ] ( )

Dr. Jamil Book Series 2 Page 24


Advanced Mathematical Techniques Chapter#2 Interpolation & Curve Fitting

For , Putting , in Eq.(1), we get:-

( ) ,( ) ( ) - ( )[ ]

( )[ ]

Substituting the values, we get

( ) [ ( )( ) ( ), ( )]

( ) , ( ) - ( )( ) ( )( )

( ) ( ) , for

( ) , for

Hence

( ) ( ) ( ) ( ) ( )

( )

and
( ) ( )

Therefore
( ) ( ) ( ) ( )

Note:-
We can also find ( ) in the interval ( )using equation (1) for Since ( ) is
required, we have not cared to find ( ) in ( ).

Example # 2:-
Using cubic spline, find ( ) and ( ) given that and the table:

0 1 2

( ) -5 -4 3

Dr. Jamil Book Series 2 Page 25


Advanced Mathematical Techniques Chapter#2 Interpolation & Curve Fitting

Solution:-

Here and and

We have

, -

For , we get:-

, -

, ( ) -

The cubic spline interpolation polynomial is given by

( ) ,( ) ( ) - ( )[ ]

( )[ ] ( )

For , Putting , in Eq.(1), we get:-

( ) ,( ) ( ) - ( )[ ]

( )[ ]

Substituting the values, we get

( ) , ( ) ( )- ( )( ) ( )( )

( ) ( ) ( )

( ) ( ) , for

Dr. Jamil Book Series 2 Page 26


Advanced Mathematical Techniques Chapter#2 Interpolation & Curve Fitting

( ) ( ) ( ) ( )( )

and

( ) ( ) ( ) ( )

Therefore

( ) ( ) ( ) ( )

Example#3:-
Find the cubic spline interpolation polynomial for the data given below under the
conditions ( ) ( ) and valid in the sub-interval , -. Hence find ( ):

( )

Answer:-
Here and

We have

, -,

For in above equation, we get:-

, -

, -

Since , and substituting values, we get

( ( ) )

( ( ) )

Solving simultaneously, we get , . The cubic Spline interpolation


Polynomial is given by:

Dr. Jamil Book Series 2 Page 27


Advanced Mathematical Techniques Chapter#2 Interpolation & Curve Fitting

( ) ,( ) ( ) - ( )[ ]

( )[ ] ( )

For , Putting in Eq(1), we get:-

( ) ,( ) ( ) - ( )[ ] ( )[

Substituting values, we have

( ) ,( ) ( ) ( )- ( )[ ] ( )[ ]

( ) ( ) ( ) ( ) ( )

For ( ) we use Eq. (2), therefore

( ) ( ) * ( ) ( ) ( ) +

Example # 4:-
Find the cubic spline polynomial in each interval for the function given below:

0 1 2 3

( ) 1 2 33 244

Assume ( ) ( ) ( )
Solution:-
Here and

We have

, -,

For in above equation, we get:-

, -

Dr. Jamil Book Series 2 Page 28


Advanced Mathematical Techniques Chapter#2 Interpolation & Curve Fitting

, -

Since , and substituting values, we get

( )

( )

Solving simultaneously, we get , . The cubic Spline interpolation


Polynomial is given by:

( ) ,( ) ( ) - ( )[ ]

( )[ ] ( )

1): For , Putting in Eq(1), we get:-

We get

( ) ,( ) ( ) - ( )[ ] ( )[

Substituting values we have,

( )
( ) ,( ) ( ) ( ) ( )+( )[ ( )] ( ), -

( ) ( ) (2)

( )

2): For , Putting in Eq(1), we get:-

( ) ,( ) ( ) - ( )[ ] ( )[

Substituting values, we have

Dr. Jamil Book Series 2 Page 29


Advanced Mathematical Techniques Chapter#2 Interpolation & Curve Fitting

( ) ,( ) ( ) ( )( )- ( ), ( )-
( ), -

( ) (3)

3): For , Putting in Eq(1), we get:-

We get

( ) ,( ) ( ) - ( )[ ] ( )[

Substituting values, we have

( ) ,( ) ( ) ( ) ( )- ( )[ ( )] ( ), ( )-

( ) ( ) ( ) ( )

( ) , -

( ) , (4)

Equations (2), (3) and (4) give the cubic Spline Polynomial in each sub-interval.
For ( ) we use eq (4), therefore

( ) ( ) ( ) ( ) ( )

( ) ( )

Example#5:-
Find the cubic spline polynomial in , - for the following data:

( )
Also find ( ) and ( )

Dr. Jamil Book Series 2 Page 30


Advanced Mathematical Techniques Chapter#2 Interpolation & Curve Fitting

Answer:-

, - ( ) ( )

( ) ( )

Example#6:-
Find the cubic spline polynomial for the data:

( )

Also find ( ) and ( )


Answer:-
, - , ( ) ( )

( ) ( )

Dr. Jamil Book Series 2 Page 31


Advanced Mathematical Techniques Chapter#2 Interpolation & Curve Fitting

CHAPTER # 2
PART-II: CURVE FITTING
2.6 Least squares method for curve fitting
Introduction:-
Let
̅ ( ) ̅ ( )
Least square approximation is the most commonly used method for finding an approximate
curve for the given data dealing with science and engineering. This method was developed
by Legendre. Suppose we have given the following data:
“Table of values”

( )
( )

OR
( )

be the given data. Let ( ) be an approximation to the function . The errors


between of the data and of the approximation are given by:

( )

Since the error can be either or– . We will consider . Then sum of squares of
errors or total error is given by

∑ ∑[ ( )]

The basic least square principle is “to find ̅ ( ) such that the sum of the squares of the
errors is minimum.”

2.6.1 Fitting a straight line by the method of least square:


OR Linear approximation:-
Let
( )

be the given data. Suppose the approximate straight line be given by ( ) ( )


. Now we have to select and so that the straight line is the best fit to the data. The
error is given by

Dr. Jamil Book Series 2 Page 32


Advanced Mathematical Techniques Chapter#2 Interpolation & Curve Fitting

( ) ( )

[ ( )]

The sum of squares of errors is given by

∑ ∑[ ( )]

The principle of least squares, minimum condition provides the following normal
equations.

By solving these equations, we can determine and to obtain the best fit ( ) ( )
.

∑ ∑ ∑

∑ ∑

OR

∑ ∑ ∑

∑ ∑

Example #1:-
By the method of least squares find the best fitting straight line to the data given below:

5 10 15 20 25

( ) 16 19 23 26 30

Solution:-

Let the equation be

The normal equations are

∑ ∑ (1)

Dr. Jamil Book Series 2 Page 33


Advanced Mathematical Techniques Chapter#2 Interpolation & Curve Fitting

∑ ∑ ∑ (2)

To calculate ∑ ∑ ∑ ∑ we form below the table:

“Table of Values”

5 16 25 80

10 19 100 190

15 23 225 345

20 26 400 520

25 30 625 750

∑ ∑ ∑ ∑

Substituting the values in ( ) ( ), we get

Hence the best fitting line is .

Example #2:- (HW)


Fit a straight line to the data, using least squares method. Hence find ( ).

1 2 3 4 5

( ) 16 19 23 26 30

Answer:-

Dr. Jamil Book Series 2 Page 34


Advanced Mathematical Techniques Chapter#2 Interpolation & Curve Fitting

Example # 3:-
Fit a straight line to the data using least squares method. Hence find ( ).

0 1 2 3 4

( ) 1 1.8 3.3 4.5 6.3

Answer:-
( )
Example # 4:-
Fit a straight line to the data using least squares method. Hence find ( ).

0 5 10 15 20

( ) 7 11 16 20 26

Answer:-

( )

2.6.2 Fitting a Parabola or fitting a second degree curve by the method of least
squares: OR Second degree approximation:-
Let
( )
be the given data. Suppose the approximate second degree curve be given by ( )
( ) . Now we have to select and so that the second degree curve is
the best fit to the data. The error is given by

( ) ( )

[ ( )]

Then sum of squares of errors is given by

∑ ∑[ ( )]

By the principle of least squares, minimum condition provides the following normal
equations:

Dr. Jamil Book Series 2 Page 35


Advanced Mathematical Techniques Chapter#2 Interpolation & Curve Fitting

∑ ∑ ∑

∑ ∑ ∑ ∑

∑ ∑ ∑ ∑

By solving these equations, we can determine and to obtain the best fit second degree
polynomial ( ) ( )

Example # 1:-
Fit a parabola, by the method of least squares, to the following data; also estimate ( ).

1 2 3 4 5

( ) 5 12 26 60 97

Solution:-

Let be the best fit parabola. Then the normal equations are:

∑ ∑ ∑ → (1)

∑ ∑ ∑ ∑ → (2)

∑ ∑ ∑ ∑ → (3)

“Tables of values”

1 5 1 1 1 5 5

2 12 4 8 16 24 48

3 26 9 27 81 78 234

4 60 16 64 256 240 960

5 97 25 125 625 485 2425

15 200 55 225 979 832 3672

Hence the equation ( ) ( ) and ( ) become

Dr. Jamil Book Series 2 Page 36


Advanced Mathematical Techniques Chapter#2 Interpolation & Curve Fitting

Solving these equations, we get

Hence the best fitting Parabola is;

( )

and
( ) ( ) ( )

( )

Example#2:- (HW)
Fit a Parabola to the data; and find ( )

Answer:-

( )

Example # 3:-
Fit a second degree Polynomial (Parabola) for the given data

Answer:-

( )

Dr. Jamil Book Series 2 Page 37


Advanced Mathematical Techniques Chapter#2 Interpolation & Curve Fitting

2.6.3 Fitting an exponential curve ( ):-


Let the curve to be fitted be given by .Then applying logarithm (of base 10) on both
sides, we get

, where

This being linear in and , we can find , since and are known. From ,
we can get and hence is found out.

2.6.4 Fitting a curve of the form :-

Let the curve to be fitted be given by .Then applying logarithm (of base 10) on both
sides, we get

, where , ,

Again using this linear fit, we find . Hence are known. Thus is found out.

Example # 1:-
Fit a curve of the form to the data

( )

Solution:-

Since , Applying logarithm on both sides;

, where , ,
The normal equations are: (Here slope=B and y-intercept=A)
∑ ∑ → ( )

∑ ∑ ∑ →( )

Dr. Jamil Book Series 2 Page 38


Advanced Mathematical Techniques Chapter#2 Interpolation & Curve Fitting

We form the table:


“Table of values”

1 2.1790 1 2.1790

2 2.0 4 4.0

3 1.7853 9 5.3559

4 1.6990 16 6.7960

5 1.3010 25 6.5050

6 0.9031 36 5.4186

21 - 9.8674 91 30.2545

Hence the equation ( ) ( ) becomes

Solving these equations, we get

Since

Hence the required equation is:

( ) ( )

Example # 2:-
Fit a curve of the to the data

Dr. Jamil Book Series 2 Page 39


Advanced Mathematical Techniques Chapter#2 Interpolation & Curve Fitting

Solution:-
Since , Applying logarithm on both sides;

, where Y , ,

Then the normal equations are

∑ ∑ →( )

∑ ∑ ∑ →( )

We form the table:

“Tables of values”

1 1200 0.000 3.0792 0.000 0.000

2 900 0.3010 2.9542 0.0906 0.8892

3 600 0.4771 2.7782 0.2276 1.3255

4 200 0.6021 2.3010 0.3625 1.3854

5 110 0.6990 2.0414 0.4886 1.4269

6 50 0.7781 1.6990 0.6054 1.3220

- - 2.8573 14.8530 1.7747 6.3490

Hence the equations ( ) ( ) become

Solving these equations, we get


,
Since

Hence the required equation is

Dr. Jamil Book Series 2 Page 40


Advanced Mathematical Techniques Chapter#2 Interpolation & Curve Fitting

( )

Example # 3:- (HW)


Fit a curve of the form to the data:

Answer:-

( )

Example # 4:- (HW)


Fit a curve of the form to the data by the method of least squares:

Answer:-

( )

Dr. Jamil Book Series 2 Page 41


Advanced Mathematical Techniques Chapter#2 Interpolation & Curve Fitting

EXERCISE-2
Question # 1:-
Determine the Lagrange’s interpolation Polynomial for the data:

( )

and estimate the value at i.e. find ( ).

Answer:-
( ) ( ) and ( )

Question # 2:-
Find the missing value from the following data using Lagrange interpolation Polynomial:

( )

Answer:-
( ) ( ) ( ) and ( )

Question # 3:-
Use Lagrange interpolation Polynomial, find ( ) :

( )

Answer:-
( )

Question # 4:-
Find Lagrange’s interpolation Polynomial fitting the Points ( ) , ( ) , ( )
, ( ) Hence find ( )

Answer:-
( ) ( ) ( )

( ) ( )

Dr. Jamil Book Series 2 Page 42


Advanced Mathematical Techniques Chapter#2 Interpolation & Curve Fitting

Question # 5:-
Given the following data, evaluate ( ) using Lagrange’s interpolation Polynomial:

( )

Answer:-
( )

Question # 6:-
Determine by Lagrange’s method the percentage number of patients over 40 years, using
the following data:

Age over years 30 35 45 55

patients 148 96 68 34

Answer:-
( ) ( )

Question # 7:-
Obtain the Lagrange’s interpolation Polynomial of ( ) from the following data; and
obtain ( ):

( )

Answer:-
( ) ( ) ( )

Question # 8:-
The following data give the percentage of criminals for different age groups:

( )

Using Lagrange’s formula, estimate the percentage of criminals under the age of 35.

Answer:-
( ) ( )

Dr. Jamil Book Series 2 Page 43


Advanced Mathematical Techniques Chapter#2 Interpolation & Curve Fitting

Question # 9:-
Use Lagrange’s interpolation formula to fit a Polynomial to the following data:

( )

Answer:-
( ) ( ) ( )

Question # 10:-
From the following table of values of ( ) and ( ) obtain ( ) and ( ), using
hermit’s interpolation formula:

( )

( )

Answer:-
( ) ( ) and ( ) , ( )

Question # 11:-
From the following data find ( ) using hermit’s interpolation formula and find ( )
( ) ( )

Answer:-
( ) ( ) ( ) and ( )

Question # 12:-
Using hermit’s interpolation, find cubic Polynomial from the data, also find ( )

( )

( )

Answer:-
( ) , ( )

Dr. Jamil Book Series 2 Page 44


Advanced Mathematical Techniques Chapter#2 Interpolation & Curve Fitting

Question # 13:-
Find ( ) from the data below by hermit’s interpolation formula:

( )

’( )

Answer:-
( )

Question # 14:-
Express as a Polynomial in from the following data, using hermit’s interpolating
formula:
0 1

( ) -4 −2

( ) 3 2

Answer:-
( ) ( )

Question # 15:-
Express ( ) as a Polynomial in from the following data, using hermit’s interpolation
formula.

0 1 2

( ) 1 2 33

( ) 0 4 64

Answer:-

( ) ( )

Question # 16:-
Express y as a polynomial in from the following data, using interpolation method.
-2 0 1

( ) 13 -1 1

( ) -31 1 5

Dr. Jamil Book Series 2 Page 45


Advanced Mathematical Techniques Chapter#2 Interpolation & Curve Fitting

Answer:-
( ) ( ) ( )
Question # 17:-
Express y as a polynomial in from the following data, using hermit’s interpolation
method:
1 2 3
( ) -4 5 60
( ) 0 24 96

Answer:-
( ) ( )

Question # 19:-

Estimate ( ) from the following data using newton’s forward interpolation formula:
3.1 3.2 3.3 3.4 3.5

( ) 0 0.6 1.0 1.2 1.3

Answer:-
( )

Question # 20:-

Find the forward interpolation polynomial for the data:


-1 0 1 2
( ) 4 2 2 4

Answer:-
( )
Question # 21:-

Express ( ) from the following data using newton’s backward interpolation formula:
20 25 30 35 40 45
( ) 354 332 291 260 231 204

Answer:

Dr. Jamil Book Series 2 Page 46


Advanced Mathematical Techniques Chapter#2 Interpolation & Curve Fitting

( )

Question # 22:-
Given:
1 2 3 4 5 6 7 8
( ) 1 8 27 64 125 216 343 512

Find
a): ( ) b): ( )

Answer:-
( ) ( )

Question # 23:-
Evaluate ( ) and ( ) from the table:
0 1 2 3 4 5
( ) 0 3 8 15 24 35

Answer:-
( ) ( )
Question # 24:-
Compute ( ) and ( ) using the following data:
0 1 2 3 4 5 6 7
( ) 0 7 26 63 124 215 342 511

Answer:-
( ) ( )

Question # 25:-
Compute ( ) and ( ) using the data:
0.20 0.22 0.24 0.26 0.28 0.30
( ) 1.6596 1.6698 1.6804 1.6912 1.7024 1.7139

Answer:-
( ) ( )

Dr. Jamil Book Series 2 Page 47


Advanced Mathematical Techniques Chapter#2 Interpolation & Curve Fitting

Question # 26:-
For the data find the divided difference polynomial and estimate ( )
-1 0 2 5
( ) 7 10 22 235

Answer:-
( ) ( ) ( ) ( )

Question # 27:-
Using Newton’s divided difference formula, find the value of ( ) ( ) and ( ) given
the following table:
4 5 7 10 11 13
( ) 48 100 294 900 1210 2028

Answer:-
( ) ( ) ( ) ( )( ) ( )( ) ( )
( ) ( ) ( )

Question # 28:-
Use Newton’ s divided difference formula to find ( ) and ( ) from the table:

4 5 7 10 11 13
( ) 48 100 294 900 1210 2028

Answer:-
( ) ( )

Question # 29:-
Find the Newton’s divided difference polynomial of the following data: and find ( )
0 1 3 4
( ) 1 4 40 85

Answer:-
( ) ( ) ( )
Question # 30:-
For the following data, find the cubic polynomial using Newton’s divided difference
formula:

Dr. Jamil Book Series 2 Page 48


Advanced Mathematical Techniques Chapter#2 Interpolation & Curve Fitting

0 1 2 5
( ) 2 3 12 147
Answer:-
( ) ( ) ( ) –

Question # 31:-
From the following data, find the Newton’s divided difference polynomial:
0 2 3 4 7 9
( ) 4 26 58 112 466 922

Answer:-
( ) ( ) ( )

Question # 32:-
Find the missing values in the following data:
45 50 55 60 65
( ) 3.0 2.0 -2.4

Answer:-
( ) ( )

Question # 33:-
The following table gives pressure of a steam at a given temperature. Using Newton’s
formula, compute the pressure for a temperature of
Temperature C. 140 150 160 170 180
Pressure, Kgf/cm2 3.685 4.854 6.302 8.076 10.225

Answer:-
( )

Question # 34:-
Find the missing values in the following data, using Newton’s divided difference formula:
3 6 7 8 9 10
( ) 168 120 72 63

Answer:-
( ) ( )

Dr. Jamil Book Series 2 Page 49


Advanced Mathematical Techniques Chapter#2 Interpolation & Curve Fitting

Question # 35:-
Express y as a polynomial in powers of ( ) from the following data:

0 2 3 6 9 10
( ) 3 7 24 207 714 983

Answer:-
( ) ( ) ( ) ( ) ( ) ( )

Question # 36:-
Find ( ) as a polynomial in powers of ( ) from the following data:
-1 0 2 3 7 10
( ) -11 1 1 1 141 561

Answer:
( ) ( ) ( ) ( )

Question # 37:-
From the following table, compute ( ) and ( ) using cubic spline.
1 2 3
( ) -8 -1 18

Answer:-
( ) ( ) ( )
( ) ( ) ( )
Question # 38:-
Using cubic spline, find ( ) and ( ) given that , and the table:
0 1 2
( ) -5 -4 3

Answer:-
( ) ( ) ( ) ( ) ( )
( ) ( ) ( ) ( )
Question # 39:-

Dr. Jamil Book Series 2 Page 50


Advanced Mathematical Techniques Chapter#2 Interpolation & Curve Fitting

Find the cubic spline polynomial in each interval for the function given below:
Assume that . Also find ( )
0 1 2 3
( ) 1 2 33 244

Answer:-
( ) ( )

( ) ( )

( ) ( )

Question # 40:-
Find the cubic spline given the table:(assume that )
0 2 4 6
( ) 1 9 41 41

Answer:-
( ) ( )

( ) ( )

( ) ( )

Question # 41:-
Obtain the cubic spline approximation for the function ( ) from the following data,
given by . Find ( ).
-1 0 1 2
( ) 0 1 2 18

Answer:-
( ) ( )

( ) ( )

( ) ( )

Question # 42:-
Find the cubic spline interpolation polynomial for the data given below under the condition
and valid in the sub-interval [ ] Hence, find ( )
0 1 2 3
( ) 1 4 10 8

Answer:-

Dr. Jamil Book Series 2 Page 51


Advanced Mathematical Techniques Chapter#2 Interpolation & Curve Fitting

( ) ( ) ( )( )

Question # 43:-
Find the cubic spline polynomial in [ ] for the following data:
Also find ( ) and ( )
0 1 2
( ) 0 2 6
Answer:-
( ) ( ) ( )( )

( ) ( )

Question # 45:-
Find the cubic spline polynomial for the data. Also find ( ) and ( )
0 1 2
( ) 0 1 0

Answer:-
( ) ( ) ( )( )

( ) ( )

Question # 46:-
Find the natural cubic spline in the range [ ] for the following data. Also find ( ) and
( )
0 6 12
( ) 1 3 11

Answer:-
( ) ( ) ( )( )
( ) ( )
Question # 47:-
Find the cubic spline for the following data with and
-1 0 1
( ) 1 0 1

Answer:-
( ) ( )

( ) ( )

Dr. Jamil Book Series 2 Page 52


Advanced Mathematical Techniques Chapter#2 Interpolation & Curve Fitting

Question # 49:-
By the method of least squares find the best fitting straight line to the data given below:
5 10 15 20 25
( ) 16 19 23 26 30

Answer:-
( )
Question # 50:-
Find a straight line to the data, using least square method. Hence find ( )
1 2 3 4 5
( ) 16 19 23 26 30
Answer:-
( )
Question # 51:-
Fit a straight line to the data using least square method. Hence find ( ).
0 1 2 3 4
( ) 1 1.8 3.3 4.5 6.3
Answer:-
( ) ( )
Question # 52:-
Fit a straight line to the data using least square method. Hence find ( )
0 5 10 15 20
( ) 7 11 16 20 26
Answer:-
( ) ( )
Question # 53:-
Fit a straight line of the form to the following data:
1 3 5 7 9
( ) 1.5 2.8 4.0 4.7 6.0
Answer:-
( )
Question # 54:-
Fit a parabola to the data using least square method. Also estimate ( )
1 2 3 4 5
( ) 5 12 26 60 97
Answer:-

Dr. Jamil Book Series 2 Page 53


Advanced Mathematical Techniques Chapter#2 Interpolation & Curve Fitting

( )

( )

Question # 55:-
Fit a parabola to the data and find y(4.5).
1 2 3 4 5
( ) 2 3 5 8 10
Answer:-
( )
Question # 56:-
Fit a second-degree polynomial (parabola) for the given data:
5 10 15 20 25
( ) 16 19 23 26 30
Answer:-
( )

Question # 57:-
Fit a curve of the form y=ax + bx2 to the following data by the method of least square:
1 2 3 4 5
( ) 1.8 5.1 8.9 14.1 19.8
Hint:-
Put (straight line case)
Answer:-
( )

Question # 58:-
Fit a curve of the form to the following data by the method of least square:
1 2 4 6 8
( ) 5.43 6.28 10.32 14.86 19.51
Hint:-
Put (straight line case)
Answer:-

Question # 59:-
Fit a curve of the form to the data:
1 2 3 4 5 6

Dr. Jamil Book Series 2 Page 54


Advanced Mathematical Techniques Chapter#2 Interpolation & Curve Fitting

( ) 151 100 61 50 20 8
Answer:-
( ) ( )
Question # 60:-
Fit a curve of the form to the data:
1 2 3 4 5 6
( ) 1200 900 600 200 110 50
Answer:-
( )
Question # 61:-
Fit a curve of the form to the data:

1 2 3 4 5
( ) 7.1 27.8 62.1 110 161

Answer:-
( )
Question # 62:-
Fit a curve of the form to the data by the method of least squares:
0 5 8 12 20
( ) 3.0 1.5 1.0 0.55 0.18
Answer:-
( )

Question # 63:-
Determine the values of and so that the equation best fit the following data by
the method of least squares:
25 20 12 9 7 5
0.22 0.20 0.15 0.13 0.12 0.10
Answer:-

Question # 64:-
The following table gives the results of measuring the electrical conductivity C of glass at
temperature  Fahrenheit:
 86 148 166 188 202 210
0.004 0.018 0.029 0.051 0.073 0.090
If  find the values of and .

Dr. Jamil Book Series 2 Page 55


Advanced Mathematical Techniques Chapter#2 Interpolation & Curve Fitting

Answer:-

Question # 65:-
Number of bacteria “y” in a culture after hours is given in the following table:
0 1 2 3 4 5 6
( ) 32 47 65 92 132 190 275
Fit a curve of the form y=abx by the method of least square.
Answer:-
( ) ( )
Question # 66:-
An experiment on the life of a cutting tool at different cutting speeds are given below:
Speed V 350 400 500 600
Life T 61 26 7 2.6
Fit a relation of the form
Answer:-
V = 682.3T (-0.1558)
Question # 67:-
Given the following table:
0 1 2 3 4
( ) 1 5 10 22 38
Find the straight line and the parabola of best fit and calculate the sum of the squares of the
residual in both cases. Which curve is more appropriate and why?
Answer:-
( ) ( )
Estraight = 70.7, Eparabola = 2.5, parabola is better

Dr. Jamil Book Series 2 Page 56


Advanced Mathematical Techniques Chapter#2 Interpolation & Curve Fitting

SUMMARY

1. Interpolation:-
Interpolation is the process of finding the value of a function for any value of argument or
independent variable in an interval ( )

2. Extrapolation:-
Extrapolation is the process of finding the value of a function outside an interval
( )

3. Polynomial Interpolation:-
Approximation by the polynomial of least degree satisfying the given data is called
Polynomial Interpolation.

4. Lagrange’s Interpolation formula:-


The Lagrange’s interpolation polynomial formula of degree at most ( ) is given by:
( ) ( ) ( ) ∑ ( )

Where ( ) are called Lagrange’s interpolation coefficients and defined as:


( )( ) ( )( ) ( )
( ) ( )( ) ( )( ) ( ) [ ( )]

5. Hermite’s Interpolation polynomial formula:-

( ) ( ) ( ) ∑ ( ) ∑ ( )

Where

( ) [ ( )( )][ ( )]

( ) ( )[ ( )]

where and ( ) are derivatives of ( ) and ( ) respectively.

6. Newton’s forward Interpolation Polynomial formula:-

Dr. Jamil Book Series 2 Page 57


Advanced Mathematical Techniques Chapter#2 Interpolation & Curve Fitting

( ) ( ) ( ) ( ) ( )

( ) ( )

Using binomial formula, we have

( ) ( )( ) ( )( ) ( )
( )

where

This is called Newton’s or Newton’s-Gregory forward interpolation formula.

7. Newton’s backward Interpolation Polynomial formula:-

( ) ( ) ( ) ( ) ( )

( )

( ) ( )

Again using binomial formula for negative power, we have

( ) ( )( ) ( )( ) ( )
( )

where

This is called Newton’s or Newton’s-Gregory backward interpolation formula.

8.The divided differences:-

( ) ( )
[ ]

These are called first divided differences and is devoted by . Higher divided differences
operators are defined as:

Dr. Jamil Book Series 2 Page 58


Advanced Mathematical Techniques Chapter#2 Interpolation & Curve Fitting

[ ] [ ]
[ ]

for

these differences are denoted by

9. Newton’s divided difference interpolation formula:-

( ) ( ) ( ) [ ] ( )( ) [ ]

( )( )( ) [ ]

( )( ) ( ) [ ]

Cubic Spline Interpolation:-


10. Cubic Spline Interpolation Polynomial Method:-
( ) [( ) ( ) ] ( )[ ] ( )[

and

[ ]

and

Least squares method for curve fitting

11. Fitting a straight line by the method of least square:


OR Linear approximation:-

∑ ∑[ ( )]

Dr. Jamil Book Series 2 Page 59


Advanced Mathematical Techniques Chapter#2 Interpolation & Curve Fitting

12. Fitting a Parabola or fitting a second degree curve by the method of least
squares: OR Second degree approximation:-

Then sum of squares of errors is given by

∑ ∑[ ( )]

By the principle of least squares, minimum condition provides the following normal
equations:

∑ ∑ ∑

∑ ∑ ∑ ∑

∑ ∑ ∑ ∑

By solving these equations, we can determine and to obtain the best fit second-degree
polynomial ( ) ( )

13. Fitting an exponential curve ( ):-

Let the curve to be fitted be given by .Then applying logarithm on both sides, we
get

, where

This being linear in and , we can find , since and are known. From ,
we can get and hence is found out.

14. Fitting a curve of the form :-


Since

, where , ,

Dr. Jamil Book Series 2 Page 60


Advanced Mathematical Techniques Chapter#2 Interpolation & Curve Fitting

Again, using this linear fit, we find . Hence are known. Thus is found out.

Dr. Jamil Book Series 2 Page 61


Advanced Mathematical Techniques Chapter#3 Numerical Differentiation & Integration

CHAPTER # 3
NUMERICAL
DIFFERENTIATION
& INTEGRATION

Dr. Jamil Book Series 2 Page 1


Advanced Mathematical Techniques Chapter#3 Numerical Differentiation & Integration

CHAPTER # 3
PART-I: NUMERICAL DIFFERENTIATION

𝟏): Newton’s Forward differentiation formulae

1a): Newton’s Forward differentiation formulae at general point :-

Let ( ) 𝟏 be the given data points or

“Table of values”

( ) ( ) ( ) 𝟏 ( 𝟏) 𝟏 ( ) ( )

The Newton’s forward interpolation formula is given by

( 𝟏) ( 𝟏)( )
( ) ( )

( 𝟏)( ) ( 𝟏)

where

Differentiating this formula using chain rule, we have

𝟏 𝟏 𝟏
,

𝟏 𝟏
- (F1X)
𝟏

Differentiating once again, we have

𝟏 𝟏 𝟏 𝟏
( ) ( ) ( ) ( )

𝟏 𝟏 𝟏𝟏
, ( 𝟏)
𝟏

Dr. Jamil Book Series 2 Page 2


Advanced Mathematical Techniques Chapter#3 Numerical Differentiation & Integration

𝟏 𝟏 𝟏
- (F2X)
𝟏

and so on.

1b): Newton’s Forward differentiation formulae at Initial point :-

In particular at initial point , we have . Then (F1X) and (F2X) becomes

𝟏 𝟏 𝟏 𝟏 𝟏 𝟏
| 0

1 (F1X0)

𝟏 𝟏𝟏
| 0 – 1 (F2X0)
𝟏

and so on.

2): Newton’s Backward differentiation formulae

2a): Newton’s Backward differentiation formulae at general point :-

Let ( ) 𝟏 be the given data points or

“Table of values”

( ) ( ) ( ) 𝟏 ( 𝟏) 𝟏 ( ) ( )

The Newton’s forward interpolation formula is given by

( 𝟏) ( 𝟏)( )
( ) ( )

( 𝟏)( ) ( 𝟏)

where

Differentiating this formula using chain rule, we have

Dr. Jamil Book Series 2 Page 3


Advanced Mathematical Techniques Chapter#3 Numerical Differentiation & Integration

𝟏 𝟏 𝟏
,

𝟏 𝟏
- (B1X)
𝟏

Differentiating once again, we have

𝟏 𝟏 𝟏 𝟏
( ) ( ) ( ) ( )

𝟏 𝟏 𝟏𝟏
, ( 𝟏)
𝟏

𝟏 𝟏 𝟏
- (B2X)
𝟏

and so on.

2b): Newton’s Backward differentiation formulae at Final point :-

In particular at initial point , we have . Then (B1X) and (B2X) becomes

𝟏 𝟏 𝟏 𝟏 𝟏 𝟏
| 0

1 (B1XN)

𝟏 𝟏𝟏
| 0 1 (B2XN)
𝟏

and so on.

3): Derivative using central difference interpolation formulae

3a): Central differentiation formulae at general point

Let ( ) 𝟏 be the given data points or

“Table of values”

( ) ( ) ( ) 𝟏 ( 𝟏) 𝟏 ( ) ( )

Dr. Jamil Book Series 2 Page 4


Advanced Mathematical Techniques Chapter#3 Numerical Differentiation & Integration

For differentiation in the central part of the table, we can use sterling or Bessel’s

interpolation formulae and differentiate. The Sterling’s formula is given by:

( 𝟏) ( 𝟏) 𝟏
( ) 𝟏 ( )

( 𝟏)

where

and is the central point of the table. Differentiating this formula two times using chain

rule, we get

𝟏 ( 𝟏) 𝟏 𝟏 ( )
0. / 𝟏 . / 1 (C1X)

𝟏 𝟏 (𝟏 )
0 𝟏 . / 1 (C2X)

and so on.

3b): Central differentiation formulae at Central point :-

In particular at initial point , we have . Then (C1X) and (C2X) becomes

𝟏 ( 𝟏) 𝟏 𝟏
| = *( ) ( ) + (C1X0)

𝟏 𝟏
| = 0 𝟏 1 (C2X0)
𝟏

and so on.

Dr. Jamil Book Series 2 Page 5


Advanced Mathematical Techniques Chapter#3 Numerical Differentiation & Integration

Example 1 :-

Find the first two derivative at 𝟏 𝟏 and 𝟏 from the following data :

1.0 1.2 1.4 1.6 1.8 2.0

0.000 0.1280 0.5440 1.2960 2.4320 4.000

Solution :-

“Forward Difference Table ”

1.0 0.000
0.1280
1.2 0.1280 0.2880
0.4160 0.0480
1.4 0.5440 0.3360 0
0.7520 0.0480
0
1.6 1.2960 0.386 0
1.1360 0.0480
1.8 2.4320 0.4320
1.5680
2.0 4.000

𝟏𝟏

We have to use Newton’s forward formula as 𝟏 𝟏 near the starting value.

𝟏𝟏 𝟏

𝟏𝟏 𝟏

𝟏 𝟏
* +

𝟏
, ( 𝟏) -

Substituting the values in above formulae, we have

𝟏 ( ) 𝟏 ( ) ( )
| = 0 𝟏 ( ) ( )1
𝟏𝟏

(𝟏 𝟏)
|
𝟏𝟏

Dr. Jamil Book Series 2 Page 6


Advanced Mathematical Techniques Chapter#3 Numerical Differentiation & Integration

and

𝟏
| = , 𝟏 ( 𝟏)( )-
𝟏𝟏 ( )

(𝟏 𝟏)
|
𝟏𝟏

We have to use Newton’s forward differentiation formula at 𝟏 is exactly the initial


point of the table. Therefore, We have

𝟏 𝟏 𝟏
| = 0 1
𝟏

𝟏
| 0 𝟏 1
𝟏

(𝟏)
| = =0
𝟏

and

𝟏
| = , -
𝟏

𝟏
| ( )
, -
𝟏

(𝟏)
| =
𝟏

Example 2:-

The following table gives the displacement in meters at different times. Find the velocities
and accelerations at 𝟏

0 0.5 1.0 1.5 2.0


0 8.75 30.0 71.25 140.0

Solution :-

We have to use Newton’s back ward differentiation formula at 𝟏 sec is near the end
of the table.

Dr. Jamil Book Series 2 Page 7


Advanced Mathematical Techniques Chapter#3 Numerical Differentiation & Integration

“Backward Difference Table ”

( )

8.75 𝟏
21.25
𝟏
41.25
𝟏 𝟏

From above table, we have

𝟏
𝟏

𝟏
Now the newton’s formulae for backward differentiation are

𝟏 ( 𝟏) ( )
* +

𝟏
, ( 𝟏) -

After substituting we get ,

(𝟏 )
| 𝟏
𝟏

and
𝟏
| , ( 𝟏)( )-
𝟏
( )

(𝟏 )
| 𝟏
𝟏

Dr. Jamil Book Series 2 Page 8


Advanced Mathematical Techniques Chapter#3 Numerical Differentiation & Integration

Now

Therefore we have to use Newton’s backward differentiation formula at sec is


exactly the final point of the table. Therefore, We have
𝟏 𝟏 𝟏 𝟏 𝟏 𝟏
| [ ] [ ( ) ( )]

( )
| 𝟏

Also

𝟏
| , -

𝟏
| , -

( )
| 𝟏

Example 3:-

Find , at for the following data:

0 0.2 0.4 0.6 0.8 1.0 1.2

0 0.122 0.493 1.123 2.022 3.200 4.666

Solution:-
We have to use central difference formula because is the centre point of the given
data.

Dr. Jamil Book Series 2 Page 9


Advanced Mathematical Techniques Chapter#3 Numerical Differentiation & Integration

“Central Difference Table”

0 0

0.2 𝟏

𝟏 𝟏

𝟏 0.4 𝟏

𝟏 𝟏

0.6 𝟏𝟏 𝟏 𝟏

𝟏 𝟏 𝟏

𝟏 0.8 𝟏 𝟏 𝟏

𝟏𝟏 𝟏

1.0 𝟏

1.2

We have
𝟏 , , 𝟏 , 𝟏 , 𝟏 𝟏 ,

, , 𝟏 and 𝟏

Here and , then by central difference formula, we have

𝟏 𝟏 𝟏 𝟏 𝟏
| * ( ) +

𝟏 𝟏 𝟏 𝟏 𝟏 𝟏
| 0 1

| 𝟏

Also

𝟏 𝟏 𝟏
| 0 𝟏 1
𝟏

𝟏 𝟏 𝟏
| [ ( ) ( 𝟏)]
( ) 𝟏

Dr. Jamil Book Series 2 Page 10


Advanced Mathematical Techniques Chapter#3 Numerical Differentiation & Integration

Example 4:-
From the following table:

1.0 1.2 1.4 1.6 1.8 2.0 2.2

2.7183 3.3201 4.0552 4.9530 6.0496 7.3891 9.0250

Calculate at 𝟏 𝟏 .

Answer:-

Example 5:-

Compute , at 𝟏 from the following data:

𝟏
𝟏 𝟏𝟏 𝟏 𝟏 𝟏 𝟏

Answer:-

Dr. Jamil Book Series 2 Page 11


Advanced Mathematical Techniques Chapter#3 Numerical Differentiation & Integration

CHAPTER # 3

Area of Trapezium:-

The area of Trapezium is given by:

𝟏
( ) ( )

Note(Area under the Line or Trapezium):-


Trapezium

( ( ) ( )) ( ( ) ( 𝟏 ))

Dr. Jamil Book Series 2 Page 12


Advanced Mathematical Techniques Chapter#3 Numerical Differentiation & Integration

1): Trapezodal Rule

1a): Simple Trapezoid Rule:-

The numerical value of this definite integral ∫ ( ) the simple trapezoid rule when the
curve ( ) is approximated by lines (linear on first degree approximation and the
number of points are two o and 𝟏 and the number of subinterval 𝟏 and
is given by

Exact area under the curve Area under the Line or Trapezium

∫ ( ) , ( ) ( )- , ( ) ( 𝟏 )- (Area of Trapezium)

1b): Genralized or Composite Trapezoidal Rule:-

Consider the curve ( ) bounded by lines and We are interested to find


∫ ( ) that is the area under the curve ( ) bounded by and , using
Trapezoidal rule

Divide the interval , - into n sub interval , i i- of equal length or step size
as shown in the figure above, Then

Dr. Jamil Book Series 2 Page 13


Advanced Mathematical Techniques Chapter#3 Numerical Differentiation & Integration

𝟏 𝟏

OR

Dr. Jamil Book Series 2 Page 14


Advanced Mathematical Techniques Chapter#3 Numerical Differentiation & Integration

and the step size is

The corresponding value of the function ( ) at the points and

( ) ( 𝟏) ( ) ( )

Let the area under the curve ( ) between these points 𝟏 and be denoted by i,

then

∫ 𝟏
( )

, ( 𝟏) ( )- 𝟏

Thus the area under the curve from to is

∫ ( ) 𝟏

Exact area under the curve Area under the Lines or Trapeziums

Using simple Trapezoid rule for each above integral that is approximate the curve ( )

by line to each of these integral, we have,

∫ ( ) , ( ) ( 𝟏 )- , ( 𝟏) ( )- , ( ) ( )-

, ( 𝟏) ( )-

∫ ( ) , ( ) ( 𝟏) ( ) ( ) ( 𝟏) ( )-

This is called generalized or Composite Trapezoid formula for ( 𝟏) points and sub
intervals.
Note:-
The accuracy of the approximate solutions can be increased n (number of Trapezium or
sub intervals) or by decreasing step size h.

Dr. Jamil Book Series 2 Page 15


Advanced Mathematical Techniques Chapter#3 Numerical Differentiation & Integration

Example#1:-

Evaluate the integral: ∫𝟏 ,using Trapezoid rule for


1): 3-points.
2): 5-points.
3): 11-points.
also calculate the exact value and comment on your result.
Solutions:-
1): For 3-points:-
We have,
𝟏
( ) 𝟏

For these points 𝟏 we have and step size is;

𝟏
𝟏

“Table of values”
1
𝟏

( ) 𝟏 𝟏𝟏
( ) ( 𝟏) ( )

Using trapezoid formula for 3 points, we have

∫ ( ) , ( ) ( 𝟏) ( )-

Substituting value we have,

𝟏 𝟏 𝟏
∫ ,𝟏 ( ) 𝟏𝟏 -
𝟏

2): For 5- points:-


For 5 -points 𝟏 we have and step size is

𝟏 𝟏

“Table of values”

𝟏 𝟏
𝟏
( ) 𝟏 𝟏 𝟏𝟏
( ) ( 𝟏) ( ) ( ) ( )

Using trapezoid formula for 5-point, we have

Dr. Jamil Book Series 2 Page 16


Advanced Mathematical Techniques Chapter#3 Numerical Differentiation & Integration

∫ ( ) , ( ) ( 𝟏) ( ) ( ) ( )-

Substituting value we have,

( 𝟏)
∫ ,𝟏 ( ) ( ) ( 𝟏) 𝟏𝟏 -
𝟏

3): For 11- points:-


For 11 -points 𝟏 𝟏 we have 𝟏 and step size is

𝟏 𝟏
𝟏 𝟏

“Table of values”

1 1.2 1.4 1.6 1.8 2.0

( ) 1 0.6944 0.5102 0.3906 0.3086 0.25


( ) ( 𝟏) ( ) ( ) ( ) ( )

2.2 2.4 2.6 2.8 3.0

0.2066 0.1736 0.1472 0.1275 0.11


( ) ( ) ( ) ( ) ( 𝟏 )

Using trapezoid formula for 11 points, we have

∫ ( ) , ( ) * ( 𝟏) ( ) ( ) ( ) ( ) ( )

( ) ( ) ( )+ ( 𝟏 )]

Substituting value we have,

∫𝟏 ,𝟏 ( ) ( 𝟏 ) ( ) ( ) ( )

( 𝟏) ( 𝟏 𝟏) ( 𝟏 ) ( 𝟏 𝟏) 𝟏𝟏 -

(𝟏 𝟏 𝟏 𝟏)
𝟏

Exact value:-
𝟏 𝟏
∫ | | [ 𝟏]
𝟏 𝟏

Dr. Jamil Book Series 2 Page 17


Advanced Mathematical Techniques Chapter#3 Numerical Differentiation & Integration

Comment:-
It is clear that as the number of trapezium or sub intervals are increased then the
approximate solution become closer to the exact value.

Example#2:-
𝟏
Evaluate the integral: ∫ using trapezoidal rule for
𝟏
1): 5- points
2): 11- points
Also calculate the exact value and comment on your result.
Solutions:-
1): For 5- points:-
We have

𝟏
( )= , 𝟏
𝟏

For 5 -points 𝟏 we have and step size is,

𝟏
h =𝟏

“Table of values”

𝟏
𝟏

( ) 𝟏
( ) ( 𝟏) ( ) ( ) ( )

Using trapezoid formula for 5 points, we have

∫ ( ) , ( ) ( 𝟏) ( ) ( ) ( )-

Substituting value we have,

𝟏
( )
∫ ,𝟏 ( ) ( ) ( ) -

2): For 11- points:-


For 11 -points 𝟏 𝟏 we have 𝟏 and step size is

𝟏 𝟏
𝟏
=𝟏 𝟏

Dr. Jamil Book Series 2 Page 18


Advanced Mathematical Techniques Chapter#3 Numerical Differentiation & Integration

“Table of values”

0 0.1 0.2 0.3 0.4 0.5


𝟏

( ) 1 0.99 0.96 0.92 0.86 0.80


( ) ( 𝟏) ( ) ( ) ( ) ( )

0.6 0.7 0.8 0.9 1.0


𝟏

0.74 0.67 0.61 0.55 0.5


( ) ( ) ( ) ( ) ( 𝟏 )

Using trapezoid formula for 11 points, we have

∫ ( ) , ( ) * ( 𝟏) ( ) ( ) ( ) ( ) ( )

( ) ( ) ( )+ ( 𝟏 )]

Substituting value we have,


𝟏
𝟏
∫ ,𝟏 ( ) ( ) ( ) ( ) ( ) ( )

( ) ( 𝟏) ( ) -

𝟏(𝟏 𝟏 )

Exact value:-
𝟏
𝟏 𝟏 𝟏( 𝟏
∫ | | , 𝟏) ( )-
𝟏

Comment:-
The absolute error on above calculations are

𝟏) | |

) 𝟏𝟏 | |

The error can be futher minimized by increasing sub interval n or decreasing step size by
h.

Dr. Jamil Book Series 2 Page 19


Advanced Mathematical Techniques Chapter#3 Numerical Differentiation & Integration

Note(Area under the Parabola):-

, ( ) ( 𝟏) ( )-

2): Simpson’s 1/3rd Rule

2a): Simple Simpson 1/3rd Rule:-

Dr. Jamil Book Series 2 Page 20


Advanced Mathematical Techniques Chapter#3 Numerical Differentiation & Integration

The numerical value of the definite integral ∫ ( ) by simple simpson’s 3rd rule when
the curve ( ) approximated by parabola (quadratic or second degree approximation)
the number of points are three(odd) 𝟏 and number of interval
(even) and the step size is given by (without proof)

Exact area under the curve Area under the Parabola

∫ ( ) , ( ) ( 𝟏) ( )-

The formula is known as simple simpson’s 1/3rd rule for numerical integration.

2b): Generalized or Composite Simpson 1/ 3rd rule:-


Consider the curve ( ) bounded by the lines and we are interested to
find ∫ ( ) that is the area under the curve ( ) bounded by and and
using simpson 3rd rule.

We shall divide the level of integration , - into an even number of sub –intervals
( ) of equal length o step-size . The number of
points are ( 𝟏) (odd) given by:

𝟏 𝟏

Or

Dr. Jamil Book Series 2 Page 21


Advanced Mathematical Techniques Chapter#3 Numerical Differentiation & Integration

Then the numerical value of definite integral ∫ ( ) or area under the curve ( )
from to from generalized or composite Simpson’s 1/3rd rule is given by:

Exact area under the curve Area under all Parabolas

∫ ( ) ∫ ( )

∫ ( ) ∫ ( ) ∫ ( ) ∫ ( ) ∫ ( )

Using simple Simpson’s 1/3rd rule for each above integrals that is approximate the curve
( ) by parabolas to each of these integrals, we have,

∫ ( ) , ( ) ( 𝟏) ( )- , ( ) ( ) ( )-

, ( ) ( ) ( )- , ( ) ( 𝟏) ( )-

∫ ( ) , ( ) ( 𝟏) ( ) ( ) ( ) ( ) ( 𝟏)

( ) ( )-

∫ ( )

, * +

* + -

This is called generalized or composite Simpson’s 1/3rd formula for ( 𝟏) points and
sub-intervals.
Note:-
In Simpson’s 1/3rd rule, the number of sub-intervals must be even or the number of
points must be odd otherwise it is not applicable.

Dr. Jamil Book Series 2 Page 22


Advanced Mathematical Techniques Chapter#3 Numerical Differentiation & Integration

Example # 3:-
Apply 5-Points Simpson’s 1/3rd rule to evaluate:
𝟏
∫ 𝟏
, compare your answer with the exact value and obtained value by trapezoidal rule.

Solution:-
5- Points trapezoidal rule:-

We have

𝟏
( )= , 𝟏
𝟏

For 5 -points 𝟏 we have and step size is,

(𝟏 ) 𝟏
h =

“Table of values”

𝟏 𝟏

( ) ( )𝟏 ( 𝟏) ( ) ( ) ( )

Using trapezoid formula for 5 points, we have

∫ ( ) , ( ) ( 𝟏) ( ) ( ) ( )-

Substituting value we have,

𝟏
( )
∫ ,𝟏 ( ) ( ) ( ) -

5- Point Simpson’s 1/3rd rule:-


Using Simpson’s 1/3rd formula, for five points we have,

∫ ( ) , ( ) ( 𝟏) ( ) ( ) ( )-

Substituting values, we have


𝟏
∫ ,𝟏 ( ) ( ) ( ) -
𝟏

𝟏
( )

𝟏
Exact value:-
𝟏
, 𝟏 - 𝟏 𝟏( 𝟏( 𝟏(
∫ 𝟏) 𝟏) )
𝟏

Dr. Jamil Book Series 2 Page 23


Advanced Mathematical Techniques Chapter#3 Numerical Differentiation & Integration

𝟏
𝟏 𝟏

𝟏

Comment:-

The solution obtained by Simpson’s 1/3rd rule is better than Trapezoid rule. So we refer
Simpson’s 1/3rd rule.

Example # 4:-
𝟏
Evaluate the integral:∫ , using
1): 5-Points trapezoidal rule:-
2): 5 -Points Simpson’s 1/3rd rule and compare your results with the exact value and
comment on the superiority of the method applied.
Solution:-

1):5-Points trapezoidal rule:-

We have,

( ) 𝟏

For 5 points 𝟏 we have and step size is:


𝟏

“Table of Values”

00 1 2.5 2 5.0 3 7.5 4 10

( ) ( )0 ( 𝟏 )18.75 ( )75 ( )168.75 ( )300

Using Trapezoidal formula, for 5 points we have,

∫ ( ) , ( ) ( 𝟏) ( ) ( ) ( )-

Substituting the values, we have


𝟏
∫ , *𝟏 𝟏 + -

𝟏 ( )
∫ , -

Dr. Jamil Book Series 2 Page 24


Advanced Mathematical Techniques Chapter#3 Numerical Differentiation & Integration

𝟏
∫ 𝟏 𝟏

1): 5 -Points Simpson’s 1/3rd rule:-

Using Simpson’s 1/3rd formula, for five points, we have

∫ ( ) , ( ) ( 𝟏) ( ) ( ) ( )-

Substituting the values, we have


𝟏
∫ , (𝟏 ) ( ) (𝟏 ) -

𝟏
∫ ,𝟏 - ( ) 𝟏

Exact value:-
𝟏
𝟏
∫ , - 𝟏 𝟏

Comment:-
The (absolute) error in above two, methods are:
1): 𝟏 𝟏 𝟏 𝟏
2): 𝟏 𝟏 𝟏
We are thankful to Simpson’s 1/3rd rule because it is far better than Trapezoidal rule.

Note(Area under the Cubic curve):-

Dr. Jamil Book Series 2 Page 25


Advanced Mathematical Techniques Chapter#3 Numerical Differentiation & Integration

, ( ) ( 𝟏) ( ) ( )-

3): Simpson’s 3/8th Rule

3a): Simple Simpson’s 3/8th:-_

The numerical value of the definite integral ∫ ( ) , by simple simpson’s 3/8th rule, when
the curve ( ) approximated by cubic curve( third degree approximation) and number
of points are four 𝟏 and number of sub-intervals by (without proof).

Exact area under the curve Area under Cubic curve

∫ ( ) , ( ) ( 𝟏) ( ) ( )-

This formula is known as simple Simpson’s 3/8th rule for numerical integration.

Dr. Jamil Book Series 2 Page 26


Advanced Mathematical Techniques Chapter#3 Numerical Differentiation & Integration

3b): Generalized or Composite Simpson’s 3/8th rule:-


Consider the curve ( ) bounded by the lines 𝟏 and . we are interested to

find ∫ ( ) that is area under the ( ) bounded by and using


Simpson’s 3/8th rule.

We shall divide the interval of integration , - into( ) number

of sub-intervals of equal length or step-size . The number of points are ( 𝟏)

given by:
𝟏 𝟏

or
𝟏

Then numerical value of definite integral ∫ ( ) or area under the curve ( ) from
from generalized or composite Simpson’s 3/8th rule is given by

Exact area under the curve Area under all Cubic curves

∫ ( ) ∫ ( )

Dr. Jamil Book Series 2 Page 27


Advanced Mathematical Techniques Chapter#3 Numerical Differentiation & Integration

∫ ( ) ∫ ( ) ∫ ( ) ∫ ( ) ∫ ( )

Using simple Simpson’s 3/8th rule for each above integrals that is approximate the curve
( ) by cubic curves to each of these integral, we have

∫ ( ) , ( ) ( 𝟏) ( ) ( )- , ( ) ( ) ( )

( )- , ( ) ( ) ( ) ( )- , ( ) ( )

( 𝟏) ( )

∫ ( ) , ( ) ( 𝟏) ( ) ( ) ( ) ( ) ( )

( ) ( ) ( 𝟏) ( )-

∫ ( ) , ( ) * ( 𝟏) ( ) ( ) ( 𝟏 )+ * ( )

( ) ( )+ ( )-

This is called generalized or composite Simpson’s 3/8th formula for ( 𝟏) points and n
sub-interval.
Note:-
In Simpson’s 3/8th rule, the number of sub-intervals “n” must be multiple of 3, otherwise it
is not applicable.

Example # 5:-
Evaluate: ∫ , when , using
𝟏
1): Trapezoidal rule
2): Simpson’s 1/3rd rule
3): Simpson’s 3/8th rule and compare your result with exact value and comment.

Solution:-
Trapezoidal rule:-
We have,

𝟏
( )
𝟏

For seven points 𝟏 we have and step size is :

Dr. Jamil Book Series 2 Page 28


Advanced Mathematical Techniques Chapter#3 Numerical Differentiation & Integration

“Table of Values”

0 1 2 3 4 5 6
0 1 3 4 5 6 7
( ) 1 ½ 1/3 ¼ 1/5 1/6 1/7
( ) ( 𝟏) ( ) ( ) ( ) ( ) ( )

Using Trapezoidal formulas for seven points, we have

∫ ( ) , ( ) ( 𝟏) ( ) ( ) ( ) ( ) ( )-

Substituting the values, we have


𝟏 𝟏 𝟏 𝟏 𝟏 𝟏 𝟏
∫ [𝟏 { } ]
𝟏

𝟏 𝟏 𝟏 𝟏 𝟏 𝟏 𝟏
∫ [𝟏 { } ]
𝟏

𝟏 𝟏 𝟏
∫ [𝟏 ] , 𝟏 -
𝟏

𝟏
∫ , 𝟏 𝟏-
𝟏

Simpson’s 1/3rd rule:-


Using Simpson’s 1/3rd formula, for seven points, we have

∫ ( ) , ( ) ( 𝟏) ( ) ( ) ( ) ( ) ( )-

Substituting the values, we have


𝟏 𝟏 𝟏 𝟏 𝟏 𝟏 𝟏
∫ [𝟏 { } { } ]
𝟏

𝟏 𝟏𝟏 𝟏 𝟏
∫ [𝟏 ]
𝟏 𝟏

𝟏
∫ , 𝟏 - 𝟏 𝟏
𝟏

Dr. Jamil Book Series 2 Page 29


Advanced Mathematical Techniques Chapter#3 Numerical Differentiation & Integration

Simpson’s 3/8th rule:-


Using Simpson’s 3/8th formula for seven points, we have

∫ ( ) , ( ) ( 𝟏) ( ) ( ) ( ) ( ) ( )-

Substituting the values, we have


𝟏 𝟏 𝟏 𝟏 𝟏 𝟏
∫ [𝟏 { } ( ) ]
𝟏
𝟏 𝟏 𝟏
∫ [𝟏 ]
𝟏

𝟏
∫ , 𝟏 - 𝟏 𝟏
𝟏

Exact value:-

∫ (𝟏 ) 𝟏
𝟏

Comment:-
The (absolute) error in above three, methods are:
1): 𝟏 𝟏 𝟏 𝟏
2): 𝟏 𝟏 𝟏 𝟏
3): 𝟏 𝟏 𝟏 𝟏

From above three methods, we note that the error in Simpson’s rules are less than the error
in Trapezoidal rule, and Simpson’s 1/3rd rule is much better than the Trapezoidal and
Simpson’s 3/8th rules.

Example # 6:-
Evaluate: ∫ , when , using
𝟏
1): Trapezoidal rule
2): Simpson’s 1/3rd rule
3): Simpson’s 3/8th rule and compare your result with exact value and comment.

Solution:-
Trapezoidal rule:-
we have,

Dr. Jamil Book Series 2 Page 30


Advanced Mathematical Techniques Chapter#3 Numerical Differentiation & Integration

𝟏
( )
𝟏
For seven points 𝟏 we have and step size is :

“Table of Values”

0 1 2 3 4 5 6
0 1 2 3 4 5 6
( ) 1 0.5 0.2 0.1 0.05882 0.03846 0.02702
( ) ( 𝟏) ( ) ( ) ( ) ( ) ( )

Using Trapezoidal formulas for seven points, we have

∫ ( ) , ( ) ( 𝟏) ( ) ( ) ( ) ( ) ( )-

Substituting the values, we have

𝟏
∫ ,𝟏 * 𝟏 𝟏 + -
𝟏

𝟏
∫ ,𝟏 𝟏 𝟏 -
𝟏

𝟏
∫ , 𝟏 𝟏 𝟏- ,𝟏 𝟏 𝟏-
𝟏

Simpson’s 1/3rd rule:-

Using Simpson’s 1/3rd formula, for seven points, we have

∫ ( ) , ( ) ( 𝟏) ( ) ( ) ( ) ( ) ( )-

Substituting the values, we have


𝟏
∫ ,𝟏 * 𝟏 𝟏 + * +
𝟏

𝟏
∫ ,𝟏 𝟏 -
𝟏

𝟏
∫ , - 𝟏 𝟏 𝟏
𝟏

Simpson’s 3/8th rule:-


Using Simpson’s 3/8th formula for seven points, we have

Dr. Jamil Book Series 2 Page 31


Advanced Mathematical Techniques Chapter#3 Numerical Differentiation & Integration

∫ ( ) , ( ) ( 𝟏) ( ) ( ) ( ) ( ) ( )-

Substituting the values, we have

∫ ,𝟏 * 𝟏 + ( 𝟏)
𝟏

∫ ,𝟏 𝟏 𝟏 -
𝟏

𝟏
∫ , 𝟏 - 𝟏
𝟏

Exact value:-
, 𝟏 - 𝟏( ) 𝟏( )
∫ 𝟏
𝟏

Comment:-
The (absolute) error in above three, methods are:
1): 𝟏 𝟏 𝟏 𝟏
2): 𝟏 𝟏 𝟏 𝟏 𝟏
3): 𝟏 𝟏 𝟏 𝟏
It is clear that solution obtained by Trapezoidal rule is close to exact value

Dr. Jamil Book Series 2 Page 32


Advanced Mathematical Techniques Chapter#3 Numerical Differentiation & Integration

Comparison of Trapezoidal, Simpson’s 1/3rd and 3/8th rules


S# Characteristics Trapezoidal Rule Simpson’s 1/3rd Simpson’s 3/8th
1 No. of points 𝟏 𝟏 𝟏
2 No. of intervals

3 n is multiple of 1 2 3
4 Function y=f(x) is Lines Parabolas Cubic curves
approximated by

5 Type of First degree Second degree Third degree cubic or


approximation Linear quadratic or parabolic ternary curve

6 Initial coefficients

7 Middle terms 2,2,2,2,… 4,2,4,2,4,2,… 3,3,2,3,3,2,…


coefficients vary

8 Error ( ) ( ) ( )
𝟏 𝟏

Dr. Jamil Book Series 2 Page 33


Advanced Mathematical Techniques Chapter#3 Numerical Differentiation & Integration

EXERCISE- 3
PART-I: NUMERICAL DIFFERENTIATION
Question # 1:-
Find the first two derivative at 𝟏 𝟏 and 𝟏 from the following table:

𝟏 𝟏 𝟏 𝟏 𝟏

( ) 𝟏 𝟏

Answer:-
(1.1) = 0.630 , (1.1) = 6.60, (1) = 0 , (1) = 6.000
Question # 2:-

Find ( ), ( ), (𝟏 ) and (𝟏 ) from the following table:

𝟏 𝟏 𝟏

( ) 𝟏

Answer:-
( ) , ( ) , (𝟏 ) , (𝟏 )
Question # 3:-

Compute ( ), ( ), ( 𝟏), ( ), ( ) and ( ) from the following table:

( ) 𝟏 𝟏

Answer:-
-2, 0, 117, 16, 192, 498, 288, 12

Question # 4:-
Compute the second and third derivatives of the function tabulated below at the point
.

𝟏 𝟏

( ) 𝟏 𝟏

Answer:-

( ) 𝟏 , ( )

Question # 5:-

Compute ( ) from the following table:

Dr. Jamil Book Series 2 Page 34


Advanced Mathematical Techniques Chapter#3 Numerical Differentiation & Integration

( ) 𝟏 𝟏 𝟏 𝟏 𝟏 𝟏𝟏 𝟏𝟏

Answer:-
( )

Question # 6:-

Find ( ) and ( ) at and from the following table:

( )

Answer:-
( ) , ( ) , ( ) , ( )

Question # 7:-
A roc et is launched from the ground and its velocity during first 10 seconds is given below.
Find its acceleration at t = 5 sec.

𝟏 𝟏 𝟏 𝟏 𝟏 𝟏 𝟏

Answer:-

Question # 8:-
The Population of a certain town shown in following table :

( ) 1941 1951 1961 1971 1981

( ) 21.96 40.65 60.81 79.21 96.81

Find the rate of growth of the population in 1941.


Answer:-
1.5881
Question # 9:-

The following table gives corresponding values of Pressure and Specific volume of supper
heated steam:

𝟏 𝟏 𝟏

Find the rate of change of Pressure with respect to volume, when and 𝟏 .

Dr. Jamil Book Series 2 Page 35


Advanced Mathematical Techniques Chapter#3 Numerical Differentiation & Integration

Answer:-
6.2333, -12.6833

Question # 10:-
A road is rotating in a plane about one of its ends. The table given below gives the values of
the angle radians through which the rod has turned. for various values of time seconds.
Find its angular velocity and angular acceleration when .

𝟏 𝟏

Answer:-

Question # 11:-
The specific heats of silica glass at various temperature are as follows:

( ) 𝟏

( ) 𝟏

Find the rate of change of specific heat with respect to temperature at 120°C and 450°C.
Answer:-
0.00004429 ; 0.00004125.
Question #12:-
Find the values of sinx at from the following data using numerical
differentiation.

35 40 45 50 55

0.8192 0.7660 0.7071 0.6428 0.5736

Answer:-
0.5101 ; 0.7066 ; 0.8619
Question # 13:-

In a certain machine a slider moves along a fixed straight rod. Its distance X cms along the
rod is given in the following table for various values of time t seconds. Find the velocity
and acceleration of the slider when t = 0.3 sec.

𝟏 𝟏 𝟏

Answer:-
⁄ ⁄

Dr. Jamil Book Series 2 Page 36


Advanced Mathematical Techniques Chapter#3 Numerical Differentiation & Integration

Question # 14:-
Find the value of x for which f(x) is maximum in the range of x given, using the following
table; Find also the maximum value of f(x).

𝟏 𝟏

( )

Answer:-
92.11 ; 43.27

Question # 15:-
Find the maximum value of ( ) in the given range of , from the following table:

1.2 1.3 1.4 1.5 1.6

( ) 0.9320 0.9636 0.9855 0.9975 0.9996

Answer:-
𝟏 𝟏
Question # 16:-
In the range of ; given in the following data. Find also the minimum value of ( ).

2 3 4 5 6

( ) 31.1875 12.0275 2.8655 3.7052 14.5440

Answer:-
4.416 ; 1.9999
Question # 17 :-
For what value of is the following tabulated function a minimum? Find also the minimum
value of the function.

0 2 4 6

( ) 3 2 11 27

Answer:-
0.58 ; 1.029

Question # 18 :-
A rod is rotating in a plan. The following table given the angle in radian through which
the rod has turned for various values of the time in second.

( ) 0 0.2 0.4 0.6 0.8 1.0 1.2


( ) 0 0.12 0.49 1.12 2.02 3.20 4.67

(i): Calculate the Angular Velocity ( ) and Angular Acceleration


( ) at
(ii): Give final accumulating remarks and interpretation of your findings.
Answer:-

Dr. Jamil Book Series 2 Page 37


Advanced Mathematical Techniques Chapter#3 Numerical Differentiation & Integration

Question # 19 :-
The population of a certain town is given below. Find the rate of growth of the population
in 1941 and 1961.
Year 1931 1941 1951 1961 1971

40.62 60.80 17.95 103.56 132.65

Answer:-

Question # 20 :-
The table below gives the result of an observation. is the observed temperature in degrees
centigrade of a vessel of cooling water, is the time of minutes from the beginning of the
observations.

1 3 5 7 9

85.3 74.5 67.0 60.5 54.3

Find the approximate rate of cooling at and

Answer:-
𝟏𝟏 𝟏

Question # 21 :-
A slider in a machine moves along a fixed straight rod. Its distance along the rod is
given below for various values of the time Find the velocity of the slider and its
acceleration when

0 0.1 0.2 0.3 0.4 0.5 0.6

30.13 31.62 32.87 33.64 39.95 33.81 33.24

Answer:-

Question # 22 :-
Given the following table of values of and

1.00 1.05 1.10 1.15 1.20 1.25 1.30

1.000 1.025 1.049 1.072 1.095 1.118 1.140

Find and at a): 𝟏 b): 𝟏𝟏 c): 𝟏

Answer:-
a): 𝟏𝟏 b): 𝟏 c):

Dr. Jamil Book Series 2 Page 38


Advanced Mathematical Techniques Chapter#3 Numerical Differentiation & Integration

EXERCISE- 3
PART-II: NUMERICAL INTEGRATION
Question # 23:-
Evaluate the integral ∫𝟏 using trapezoidal rule for
1): three points
2): five points
3): eleven points
Also calculate the exact value and comment on your results.

Question # 24:-

Evaluate the integral ∫𝟏 using trapezoid rule with step size and
calculate the exact value and comment on your result.

Question # 25:-
𝟏
Evaluate the integral ∫ using trapezoidal rule for
𝟏
1): five points
2): eleven points
Also calculate the exact value and comment on your results.

Question # 26:-
𝟏
Apply 3-points Simpsons 1/3 rule to evaluate ∫ 𝟏
, compare your answer with exact
value

Question # 27:-
𝟏
Evaluate the integral ∫ , using
1): five points trapezoidal rule
2): five points Simpson’s rule
And compare your result with the exact value and comment on the superiority of the
method applied.

Question # 28:-

Evaluate the integral ∫ , using


1): five points trapezoidal rule
2): five points Simpson’s rule
And compare your result with the exact value and comment on the superiority of the
method applied.

Dr. Jamil Book Series 2 Page 39


Advanced Mathematical Techniques Chapter#3 Numerical Differentiation & Integration

Question # 29:-

Evaluate ∫ 𝟏
, where using
1): trapezoidal rule
2): Simpsons 1/3 rule
3): Simpsons 3/8 rule
and compare your result with exact value and comment.
Question # 30:-

Evaluate ∫ 𝟏
, where using
1): trapezoidal rule
2): Simpsons 1/3 rule
3): Simpsons 3/8 rule
and compare your result with exact value and comment.
Question # 31:-
𝟏
Evaluate ∫ 𝟏
, where using

1): trapezoidal rule


2): Simpsons 1/3 rule
3): Simpsons 3/8 rule
and compare your result with exact value and comment

Question # 32:-
𝟏
Evaluate ∫ , where using
(𝟏 )

1): trapezoidal rule


2): Simpsons 1/3 rule
3): Simpsons 3/8 rule
and compare your result with exact value and comment.

Question # 33:-

Evaluate ∫ by
1): trapezoidal rule
2): Simpsons 1/3 rule
Taking 6 equal interval correct to decimal places.

Question # 34:-
Evaluate ∫𝟏 by
1): trapezoidal rule
2): Simpsons 1/3 rule
Taking 6 equal interval correct to decimal places.

Question # 35:-

Evaluate ∫ 𝟏
where using
1): trapezoidal rule
2): Simpsons 1/3 rule
3): Simpsons 3/8 rule
And compare your result with exact value and comment.

Dr. Jamil Book Series 2 Page 40


Advanced Mathematical Techniques Chapter#3 Numerical Differentiation & Integration

Question # 36:-
𝟏
Evaluate ∫ 𝟏
, using Simpsons 3/8 rule with
Answer:-
0.2310846

Question # 37:-
𝟏
Compute the value of ∫ ( ) taking and using trapezoidal rule
, Simpsons y 3rd rule and 3/8 rule , compare your result by integration.

Question # 38:-
Evaluate ∫ using

Question # 39:-
𝟏
Evaluate ∫ dividing the range into 4 equal parts by

1): trapezoidal rule


2): Simpsons 1/3 rule

Question # 40:-
𝟏
Evaluate ∫ dividing the range into 8 equal parts.
𝟏

Question # 41:-
Evaluate ∫ , taking h= .

Question # 42:-
𝟏
Evaluate ∫ approximately by using a suitable formula and at least five
points.

Question # 43:-
𝟏
Calculate the value of by finding ∫ ,using Simpsons rule by dividing into 4
𝟏
equal parts.

Question # 44:-
𝟏
Evaluate ∫ ( )𝟏 , correct to two decimal places using seven coordinates.

Dr. Jamil Book Series 2 Page 41


Advanced Mathematical Techniques Chapter#3 Numerical Differentiation & Integration

“Miscellaneous Applied Problems”


Question # 44:-
The speed of a train at various times are given in following data:

T (hour) 0 0.5 1 1.5 2.0 2.5 3.0 3.25 3.5

V(Kmph) 0 13 33 39.5 40 40 36 15 0

Find the distance from to


Answer:-
98.5km

Question # 45:-
𝟏
Evaluate: ∫ √𝟏 , using Simpson’s 1/3 rule with 8 subintervals.

Answer:-
0.6479

Question # 46:-
Using Simpson’s 1/3 rule with 6 subintervals to find the Area contained between the
and the curve 4
Answer:-
9.156

Question # 47:-
A reservoir is the form of a surface of revolution and D is the diameter in meters at a depth
of P meters beneath the surface of the water. Find the amounts of water in , that the
reservoir holds when full, from the following data using Simpson’s 1/3rd rule.

P 0 5 10 15 20 25 30

D 36 35 33 29 22 16 9

Hint:- ∫ . /

Answer:-
𝟏

Question # 48:-
𝟏
Compute ∫ . / from the following data using Simpson’s rule:

Dr. Jamil Book Series 2 Page 42


Advanced Mathematical Techniques Chapter#3 Numerical Differentiation & Integration

0 1 2 3 4 5 6

0.146 0.161 0.176 0.190 0.204 0.217 0.230

Answer:-

Question # 49:-
A river is 45m wide, The depth d in meters at a distance meters from one bank is given in
the following data:

I 5 10 15 20 25 30 35 40 45

0 3 6 8 7 7 6 4 3 0

Find the Cross-Section of the river by Simpson’s 3/ rule.


Answer:-
𝟏

Question # 50:-
The velocity of an electric train which starts from

T(minutes) 0 1 2 3 4 5 6 7 8 9 10 11 12

V( ) 0 10 25 40 55 60 62 57 42 30 20 13 0

Find the total distance covered in 12 minutes.


Answer:-

Question # 51:-
Integration provides a means to compute how much mass enters or leaves a reactor over a
specified time period, as in

𝟏

where 𝟏 the initial and final times, respectively. This formula makes intuitive
sense if you recall the analogy between integration and summation. Thus, the integral
represents the summation of the product of flow times concentration to give the total mass
entering or leaving from 𝟏 . If the flow rate is constant, Q can be moved outside the
integral:


𝟏
Use numerical integration to evaluate this equation for the data listed below. Note that
.

( ) 𝟏
( ) 𝟏

Dr. Jamil Book Series 2 Page 43


Advanced Mathematical Techniques Chapter#3 Numerical Differentiation & Integration

Summary Of Ch#3
PART-I: NUMERICAL DIFFERENTIATION
𝟏): Newton’s Forward differentiation formulae:-
1a): Newton’s Forward differentiation formulae at general point :-

𝟏 𝟏 𝟏
,

𝟏 𝟏
- (F1X)
𝟏

𝟏 𝟏 𝟏𝟏
, ( 𝟏)
𝟏

𝟏 𝟏 𝟏
- (F2X)
𝟏

1b): Newton’s Forward differentiation formulae at Initial point :-


𝟏 𝟏 𝟏 𝟏 𝟏 𝟏
| 0 1 (F1X0)

𝟏 𝟏𝟏
| 0 – 1 (F2X0)
𝟏

2): Newton’s Backward differentiation formulae:-


2a): Newton’s Backward differentiation formulae at general point :-

𝟏 𝟏 𝟏
,

𝟏 𝟏
- (B1X)
𝟏

𝟏 𝟏 𝟏𝟏
, ( 𝟏)
𝟏

𝟏 𝟏 𝟏
- (B2X)
𝟏

Dr. Jamil Book Series 2 Page 44


Advanced Mathematical Techniques Chapter#3 Numerical Differentiation & Integration

2b): Newton’s Backward differentiation formulae at Final point :-


𝟏 𝟏 𝟏 𝟏 𝟏 𝟏
| 0 1 (B1XN)

𝟏 𝟏𝟏
| 0 1 (B2XN)
𝟏

3): Derivative using central difference interpolation formulae:-


3a): Central differentiation formulae at general point

𝟏 ( 𝟏) 𝟏 𝟏 ( )
0. / 𝟏 . / 1 (C1X)

𝟏 𝟏 (𝟏 )
0 𝟏 . / 1 (C2X)

3b): Central differentiation formulae at Central point :-

𝟏 ( 𝟏) 𝟏 𝟏
| = *( ) ( ) + (C1X0)

𝟏 𝟏
| = 0 𝟏 1 (C2X0)
𝟏

PART-II: NUMERICAL INTEGRATION


1. Trapezodal rule:-

There are two types of trapezoidal rules.

Simple trapezoidal rule, given by area of trapezium

∫ ( ) , ( ) ( )- , ( ) ( 𝟏 )-

Type of approximation is first degree (linear)


Step size is
Generalized or composite trapezoidal rule, given by

∫ ( ) , ( ) ( 𝟏) ( ) ( ) ( 𝟏) ( )-

Dr. Jamil Book Series 2 Page 45


Advanced Mathematical Techniques Chapter#3 Numerical Differentiation & Integration

2. Simpson’s 1/3rd rule:-

Simple Simpson’s 1/3rd rule, is given by

∫ ( ) , ( ) ( 𝟏) ( )-

Type of approximation is second degree (quadratic).

Step size is

Generalized or composite Simpson’s 1/3rd rule for (2n+1) points and 2n sub-intervals, is
given by

∫ ( ) , ( ) ( 𝟏) ( ) ( ) ( ) ( ) ( )

( 𝟏) ( )-

3. Simpson’s 3/8th rule:-

Simple Simpson’s 3/8th rule, is given by

∫ ( ) , ( ) ( 𝟏) ( ) ( )-

Type of approximation is third degree (cubic)

Step size is

Generalized or composite Simpson’s 3/8th rule for (n+1) points and n sub-intervals, is given
by

∫ ( ) , ( ) * ( 𝟏) ( ) ( ) ( 𝟏 )+ * ( )

( 𝟏 )+ ( )-

Dr. Jamil Book Series 2 Page 46


Advanced Mathematical Techniques CH#4 Numerical Diff. Eqs. & Difference Equations

CHAPTER # 4

Dr. Jamil Book Series 1 Page 1


Advanced Mathematical Techniques CH#4 Numerical Diff. Eqs. & Difference Equations

CHAPTER # 4
PART-I: NUMERICAL DIFFERENTIAL EQUATIONS
1): Euler’s Method:-
Aim:-

To solve first order and first-degree differential equation:

……………………..(1)

Subject to the initial condition

Procedure:-

Suppose we have to find the approximate numerical value of : say ). We

divide the interval into n sub-intervals by the points:

or

and step-size is:

From eq. (1), we have

Integrating from to , we have

∫ ∫

Assuming (constant), then

Dr. Jamil Book Series 1 Page 2


Advanced Mathematical Techniques CH#4 Numerical Diff. Eqs. & Difference Equations

……………… (2)

This is called Euler’s iteration formula for finding numerical solution of first order first
degree differential equation.

For , we have:-

For we have:-

For , we have:-

, and so on.

This Euler’s iteration formula (2) can be used to find , where is known. On
substituting the value of in (2) we get . Similarly putting the value of
in (2), we obtain and so on.

Note

SN For Independent variable For Dependent variable

… ………………………….. ……………………………………….

Dr. Jamil Book Series 1 Page 3


Advanced Mathematical Techniques CH#4 Numerical Diff. Eqs. & Difference Equations

Example# 1:-
Using Euler’s method, find an approximate value of y corresponding to , given that:

Solution:-

We have

The Euler’s iteration formula is given by

“Table of values”

1.0 1.00 3.00 1.30

1.1 1.30 3.70 1.67

1.2 1.67 4.50 2.12

1.3 2.12 5.54 2.67

1.4 2.67 6.74 3.34

1.5 3.34 8.18 4.16

1.6 4.16 9.92 5.15

1.7 5.15 12.00 6.35

1.8 6.35 14.50 7.80

1.9 7.80 17.50 9.55

2.0 9.55

Hence the required approximate value of at is that is

Example# 2(HW):-
Evaluate by Euler’s method from differential equation with
.
Answer:-

Dr. Jamil Book Series 1 Page 4


Advanced Mathematical Techniques CH#4 Numerical Diff. Eqs. & Difference Equations

2): Modified Euler’s Method:-


Aim:-

To solve first order and first-degree differential equation:

……………………..(1)

Subject to the initial condition

Procedure:-

Suppose we have to find the approximate numerical value of y: say ). We

divide the interval into n sub-intervals by the points:

OR

and step-size is:

From eq. (1), we have

Integrating from to , we have

∫ ∫

∫ ∫

Using Trapezoidal rule for numerical integration on right side, we get

………… (2)

Dr. Jamil Book Series 1 Page 5


Advanced Mathematical Techniques CH#4 Numerical Diff. Eqs. & Difference Equations

This is called Modified Euler’s iteration formula for finding numerical solution of first
order first degree differential equation.

Since which occurs on the right hand of eq. (2) can not be calculated since

is unknown. So first we calculate from Euler’s formula:

Thus for each stage we use the following two formula:

Example# 3 :-

Apply modified Euler’s method to solve , subject to and hence find

an approximate value of y when where

Solution :-

The differential equation with initial condition is given by:

we have

The formula for each iteration are given by:

Dr. Jamil Book Series 1 Page 6


Advanced Mathematical Techniques CH#4 Numerical Diff. Eqs. & Difference Equations

“Table of values”

Euler’s formula Modified Euler’s formula

0 0.0 1.0 3.00 1.30 0.1 4.0 1.35

1 0.1 1.35 4.15 1.765 0.2 5.495 1.832

2 0.2 1.832 5.695 2.402 0.3 7.506 2.492

3 0.3 2.492 7.776 3.270 0.4 10.21 3.391

4 0.4 3.391 10.573 4.448 0.5 13.844 4.612

5 0.5 4.612 14.336 6.046 0.6 18.738 6.266

6 0.6 6.266 19.398 8.206 0.7 25.318 8.502

7 0.7 8.502 26.206 11.123 0.8 34.169 11.521

8 0.8 11.521 35.363 15.057 0.9 46.071 15.593

9 0.9 15.593 47.679 20.361 1.0 62.083 21.081

10 1.0 21.081

Hence the required approximate value of at is that is .

Example# 4(HW) :-

Evaluate by Modified Euler’s method from differential equation with


.
Answer:-

Dr. Jamil Book Series 1 Page 7


Advanced Mathematical Techniques CH#4 Numerical Diff. Eqs. & Difference Equations

3): Runge – Kutta method (fouth – Order) (RK – method):-


A fourth order Runge – Kutta formula for

is given by:

where,

( )

( )

This is known as Runge –Kutta iteration formula. The error in this formula is of order .
This method have greater accuracy. No derivatives are required to be tabulated. It requires
only functional values at some selected points on the sub-intervals.

Example #5 :-

Apply Runge – Kutta method to find an approximate value of when , given that;

Solution:-

The differential equation with initial condition is given by :

we have,

For First Iteration: :-

we have

Dr. Jamil Book Series 1 Page 8


Advanced Mathematical Techniques CH#4 Numerical Diff. Eqs. & Difference Equations

( )

( )

The RK – formula for is given by:

Substituting the values we have,

For 2nd Iteration: :-

We have,

( )

( )

Dr. Jamil Book Series 1 Page 9


Advanced Mathematical Techniques CH#4 Numerical Diff. Eqs. & Difference Equations

and

The RK – formula for is given by:

Substituting the values we have

Hence the required approximate value of y at is i.e.

Example# 6(HW) :-

Apply Runge – Kutta method (fourth order), to find an approximate value of when
, given that:

Answer:-

Example# 7(HW) :-

Find

Answer:-

Dr. Jamil Book Series 1 Page 10


Advanced Mathematical Techniques CH#4 Numerical Diff. Eqs. & Difference Equations

CHAPTER # 4
PART-II: DIFFERENCE EQUATIONS
Difference Equation:-

A difference equation is a relation between the values of of a function defined on a

discrete set of argument . In general, a difference equation involves differences of a

function.

Order of Difference Equations:-

The order of a difference equation is the difference between the largest and smallest

arguments appearing in it.

Degree of Difference Equations:-

The degree of the highest differences function appearing in a difference equation free

from radicals and functions is called the degree of the difference equation.

Examples

SN Difference Equations Order Degree

1 1 1

2 2 1

3 3 1

4 2 2

5 3 1

6 4 3

7 2 3

8 5 4

Solution of Difference Equation:-

Solution of a difference equation is an expression for which satisfies the given difference

equation.

Dr. Jamil Book Series 1 Page 11


Advanced Mathematical Techniques CH#4 Numerical Diff. Eqs. & Difference Equations

General Solution of Difference Equation:-

The general solution of a difference equation is that in which the number of arbitrary

constants is equal to the order of the difference equation.

Particular Integral/Solution of Difference Equation:-

A particular solution or particular integral is that solution which is obtained from the

general solution by giving particular values to the constants (using IVP & BVP).

Linear Difference Equation:-

In linear difference equations, occur in the first degree only and are not

multiplied together.

General Linear Difference Equation of Order

A general linear difference equation of order is of the form:

where and ) are given functions of irregular argument .

It can also be written as;

Homogeneous Linear Difference Equation of Order :-

If , the equation is called a homogeneous linear difference equation of order .

Non-homogeneous Linear Difference Equation of Order :-

If , in E , the equations called a non-homogeneous linear


difference equation of order .

Solution of linear homogeneous difference equation with constant coefficients:-


Let us consider the homogeneous difference equation

where are constants. It can be written as;

Dr. Jamil Book Series 1 Page 12


Advanced Mathematical Techniques CH#4 Numerical Diff. Eqs. & Difference Equations

OR

where,

Auxillary Equation :-
Replace by in and equate it to zero, we get

This is called auxiliary equation. Let be the roots of the Eq. The
following cases will arise:

Three Different Types of Solution for Homogeneous DE:-

Case-I (Distinct Real Roots)

If all the roots be real and distinct, the general solution of ) is

given by

Case-II (Repeated Real Roots)

If the roots is repeated “ ” times, then general solution is:

Case-III (Complex Roots)

Let has two imaginary roots . The general solution is given by

where

√ ( )

Dr. Jamil Book Series 1 Page 13


Advanced Mathematical Techniques CH#4 Numerical Diff. Eqs. & Difference Equations

Table for Writing General Solutions of Homogeneous DE


Roots General Solutions
3,4
4,4,4

3±i4

√ ( )

2,3,3,4,4,4,5,6

2,3,3,5,6,3±i4

√ ( )

Example 1:-
Solve: – .
Solution:-
The given equation can be written as

The auxiliary equation is



⇒ – –
⇒ Roots are distinct and real
Hence the general solution is

Example 2:-
Solve: –
Solution:-
The given equation can be written as

The auxiliary equation is



⇒ –

Dr. Jamil Book Series 1 Page 14


Advanced Mathematical Techniques CH#4 Numerical Diff. Eqs. & Difference Equations

Roots are repeated and real

Hence the general solution is

Example 3:-

Solve:

Solution:-

The given equation can be written as

The auxillary equation is

√ √ √

⇒ √

⇒ Roots are complex

Hence the general solution is


where,

√ ( )

Example 4:-

Solve: –
Solution:-

The given equation can be written as;

⇒ – , –

⇒ –

⇒ –

Dr. Jamil Book Series 1 Page 15


Advanced Mathematical Techniques CH#4 Numerical Diff. Eqs. & Difference Equations

The auxillary equation is:

⇒ – –

⇒Roots can distinct and real hence the general solution is

Substituting the initial conditions in eq. (1), we get

Solving simultaneously we get

Hence the particular solution is

⇒ –

Example 5:-

Solve: – –

Solution:-

The given equation can be written as

– –

The auxillary equation is:

– –

Hence the general solution is

Dr. Jamil Book Series 1 Page 16


Advanced Mathematical Techniques CH#4 Numerical Diff. Eqs. & Difference Equations

Substituting the initial and boundary conditions in Equation (1), we get

Solving simultaneously, we get

Hence the particular solution is:

Practice Problems:-

– – –

Dr. Jamil Book Series 1 Page 17


Advanced Mathematical Techniques CH#4 Numerical Diff. Eqs. & Difference Equations

Solution of Non-Homogeneous Linear Difference Equation with Constant


Coefficients
The general form of non-homogeneous linear difference equation is given by

where,

⇒ is a function of

The general or total solution of a non-homogeneous linear difference equation with

constants coefficients is given by

where,

is the homogeneous solution o

and

= is the particular solution which satisfy,

3.11.1. Particular Solution :-

There is no general procedure to find the particular solution of a difference equation. The

method depends on the form of the function i.e. In some simple cases we give the rules

for finding the particular solution.

(Exponential Case):-

1): Case-I: :-

2): Case-II: , then:-

Dr. Jamil Book Series 1 Page 18


Advanced Mathematical Techniques CH#4 Numerical Diff. Eqs. & Difference Equations

1):

2):

3):

4):

5):

and so on.

Three Important Points about the roots of Polynomial:-

If, we have

– –

⇒ 2 and 4 are roots of , which means that

⇒ But

⇒ etc.

Ponit-1: because there are factors of ( – )and ( – )

Ponit-2: These factors – and ( – ) not necessary appear as Linear, May be

Quadratic, May be Cubic, May be BiQuadratic etc. In general power of . That is

– and ( – )

Ponit-3: The Remaining factor w.r.t. root 2 is ( – ) and Remaining factor w.r.t. root 4

( – ) Note the Remaining factor w.r.t. root 2 is ( – ), never be equal to zero at

(actually it is -2) and the Remaining factor w.r.t. root 4 is ( – ), never be equal to

zero at (actually it is 2).

Dr. Jamil Book Series 1 Page 19


Advanced Mathematical Techniques CH#4 Numerical Diff. Eqs. & Difference Equations

Note:-

If “ ” is the root of i.e

This means that have the factor – and can be written as:

where is the remaining factor and In above

1): If , we called Linear Case

2): If , we called Quadratic Case

3): If , we called Cubic Case

4): If , we called BiQuadratic Case and so on.

Examples for calculating Particular solutions for :-

1):

2):

3):

4):

5):

6):

7):

Note:-

1):

Dr. Jamil Book Series 1 Page 20


Advanced Mathematical Techniques CH#4 Numerical Diff. Eqs. & Difference Equations

2):

3):

Example 1:-
Solve: –
Solution:-
The given equation can be written as

For :-
The auxiliary equation is


Roots are distinct and real. Hence the homogeneous solution is

( ) ( )

For :-
We have

⇒ ( ) ( )

Example 2:-
Solve: –
Solution:-
The given equation can be written as

For :-

Dr. Jamil Book Series 1 Page 21


Advanced Mathematical Techniques CH#4 Numerical Diff. Eqs. & Difference Equations

The auxiliary equation is



Roots are repeated and real. Hence the homogeneous solution is

For :-
We have

(Here )

(Quadratic Case)

Example 3:-
Solve:
Solution:-
The given equation can be written as

For :-
The auxiliary equation is



Roots are distinct and real. Hence the homogeneous solution is

Dr. Jamil Book Series 1 Page 22


Advanced Mathematical Techniques CH#4 Numerical Diff. Eqs. & Difference Equations

For :-
We have

⇒ (Here )

We have

⇒ (Linear Case)

Note:-

If

⇒ ( ) and ( )

(Oscillating Case):-

Then

OR

can be written as

and solve as in Case-I with . At the end separate real and imaginary parts

according to the cosine and sine case.

Dr. Jamil Book Series 1 Page 23


Advanced Mathematical Techniques CH#4 Numerical Diff. Eqs. & Difference Equations

Note (For sepration of Real and Imagnary Parts):-

We have

Hence in general

Example 1:-

Solution:-
The given equation can be written as

:-


The homogenous solution is

For :-

( )

Dr. Jamil Book Series 1 Page 24


Advanced Mathematical Techniques CH#4 Numerical Diff. Eqs. & Difference Equations


( )

Example 2:-

Solution:-

The given equation can be written as

:-

⇒ ⇒

⇒ √ ⇒ ( )

⇒ = ( ) 1

⇒ =

For :-
We have

Dr. Jamil Book Series 1 Page 25


Advanced Mathematical Techniques CH#4 Numerical Diff. Eqs. & Difference Equations

( )

⇒ =

Hence the general solution is

Example 3:-
1):

2):

( )

4):

Factorial Polynomial :-

Dr. Jamil Book Series 1 Page 26


Advanced Mathematical Techniques CH#4 Numerical Diff. Eqs. & Difference Equations

Table for differences of factorial polynomial

Important DPZ Formulae

Formulae Name Formulae Expressions

Derivative Formula

Power Formula

Zero Formula

Applications of DPZ Formulae

Evaluate the Following:-


1):

2):
⇒ ( ) [ ]
⇒ ( )

3):
⇒ [ ]

Dr. Jamil Book Series 1 Page 27


Advanced Mathematical Techniques CH#4 Numerical Diff. Eqs. & Difference Equations



(Polynomial Case):-
We have

Then we expand in ascending power of and operate on , by using


DPZ formulae and get the required Particular Solutions

Expansion Formulae of :-

1):

2):

Example-1:-

Solution:-
The given equation can be written as

:-

⇒ ⇒

Roots are distinct and real. Hence the homogeneous solution is

:-

Dr. Jamil Book Series 1 Page 28


Advanced Mathematical Techniques CH#4 Numerical Diff. Eqs. & Difference Equations

⇒ * +

⇒ [ ( ) ]

⇒ * +( )

Ignoring and higher order, we have

⇒ [ ]( )

Using DPZ formulae, we get

⇒ [ ( ) ]

⇒ [ ]

⇒ [ ]

Hence the general solution is


⇒ [ ]

Example-2:-
:
Solution:-
The given equation can be written as

Dr. Jamil Book Series 1 Page 29


Advanced Mathematical Techniques CH#4 Numerical Diff. Eqs. & Difference Equations

:-

⇒ ⇒ ⇒
Roots are distinct and real. Hence the homogeneous solution is

:-
We have

⇒ ( ) ( )

⇒ ( )

⇒ * + ( )

⇒ * ( ) +

⇒ * +( )

Ignoring and higher order, we have

⇒ * +( )

⇒ [ ( )

( )]

Using DPZ formulae, we get

Dr. Jamil Book Series 1 Page 30


Advanced Mathematical Techniques CH#4 Numerical Diff. Eqs. & Difference Equations

⇒ [ ( ) ]

⇒ * +

⇒ ( )

Hence the general solution is

⇒ ( )

, :-
(Mixture/Combine Case)
We have

Using Multiplying Theorem, we have

and then solve as case-I, II OR III according to the form of .

Example-1:-

Solution:-
The given equation can be written as

:-

⇒ ⇒
Roots are distinct and real. Hence the homogeneous solution is

Dr. Jamil Book Series 1 Page 31


Advanced Mathematical Techniques CH#4 Numerical Diff. Eqs. & Difference Equations

We have

Using Multiplying Theorem, we have

Replace by , we get

⇒ * +

⇒ [ ( ) ]

⇒ [ ]

Ignoring and higher order, we have

⇒ [ ]

Using DPZ formulae, we get

⇒ [ ( ) ]

⇒ [ ]

Dr. Jamil Book Series 1 Page 32


Advanced Mathematical Techniques CH#4 Numerical Diff. Eqs. & Difference Equations

⇒ [ ]

Hence the general solution is


⇒ * +

Example-2:-

Solution:-

:-

The auxiliary equation is

⇒ ⇒

Roots are distinct and real. Hence the homogeneous solution is

We have

⇒ =

Using Multiplying Theorem, we have

Replace by , we get

Dr. Jamil Book Series 1 Page 33


Advanced Mathematical Techniques CH#4 Numerical Diff. Eqs. & Difference Equations

⇒ ( )

⇒ ( )

⇒ ( )

Ignoring and higher order, we have

⇒ ( )

⇒ ( ) ( )

Using DPZ formulae, we get

⇒ [ ( ) ]

Hence the general solution is:


Example-3:-

Solution:-

The given equation can be written as

:-

The auxiliary equation is

Dr. Jamil Book Series 1 Page 34


Advanced Mathematical Techniques CH#4 Numerical Diff. Eqs. & Difference Equations

⇒ Roots are repeated and real.

The homogenous solution is

We have

⇒ ( )

⇒ ( ) ( ) ( )

Hence the general solution is:


1):

2): –

– ( )

4): ( – )

Dr. Jamil Book Series 1 Page 35


Advanced Mathematical Techniques CH#4 Numerical Diff. Eqs. & Difference Equations

Simple Example of Mathematical Modeling through Differential Equation


and Difference Equation
For Continuous System (Differential Equation):-

Consider a particle following continuous sinusoidal wave. Then we have

This is the differential equation of this simple dynamical system.

Continuous Dynamical System

For Discrete System (Difference Equation):-


If a particle following discrete sinusoidal wave, then we have




Discrete Dynamical System

Dr. Jamil Book Series 1 Page 36


Advanced Mathematical Techniques CH#4 Numerical Diff. Eqs. & Difference Equations

EXERCISE-4
PART-I: NUMERICAL DIFFERENTIAL EQUATIONS
Question # 1:-
Using Euler’s method find an approximate value of corresponding to , given that
.

Answer:-

Question # 2:-
Using Euler’s method find an approximate value of corresponding to , given that

Answer:-

Question # 3:-
Apply modified Euler’s method to solve and hence find
an approximate value of
Answer:-

Question # 4:-
Using Euler’s modified formula, find an approximate value of
taking the interval 0.02.

Answer:-

Question # 5:-
Apply RK method to find an approximate value of 0.2, given that:
.

Answer:-

Question # 6:-
Apply RK method to find an approximate value of 0.2, given that:
.

Dr. Jamil Book Series 1 Page 37


Advanced Mathematical Techniques CH#4 Numerical Diff. Eqs. & Difference Equations

Answer:-

Question # 7:-
Use Euler’s method to compute from the differential equation
when
Answer:-

Question # 8:-
Given that with initial condition that find by Euler’s
modified method , , correct up to three decimal Places.
Answer:-

Question # 9:-
Use Euler’s method to evaluate correct up to two decimal Places, from
.

Answer:-

Question # 10:-
Given that by
Euler’s method.
Answer:-

Question # 11:-
Evaluate by modified Euler’s method from differential equation with
.
Answer:-

Question # 12:-
Evaluate by modified Euler’s method from differential equation with
.

Dr. Jamil Book Series 1 Page 38


Advanced Mathematical Techniques CH#4 Numerical Diff. Eqs. & Difference Equations

Answer:-

Question # 13:-
Evaluate by modified Euler’s method from differential equation with
.
Answer:-

Question #15:-
Use RK-method to compute from .

Answer:-

Question # 16:-
Compute using RK-method
with step-size
Answer:-

Question # 17:-
Use RK-method to compute from .

Answer:-

Question # 18:-
Use RK-method to compute from .

Answer:-

Question # 19:-
Compute by RK-method correct to five decimal Places, from the equation

Answer:-

Dr. Jamil Book Series 1 Page 39


Advanced Mathematical Techniques CH#4 Numerical Diff. Eqs. & Difference Equations

Question # 20:-
Find the values of using modified Euler’s method with , given
that:

Answer:-

Question # 21:-
Find the values of by using modified Euler’s method
given that:

Answer:-

Question # 22:-
Find the values of by using modified Euler’s method with
given that:

Answer:-

Question # 23:-
Find the values of by using Euler’s and modified Euler’s
methods respectively if:

Answer:-

Question # 24:-
Find

Answer:-

Question # 25:-
Find

Dr. Jamil Book Series 1 Page 40


Advanced Mathematical Techniques CH#4 Numerical Diff. Eqs. & Difference Equations

Answer:-

Question # 26:-
Find

Answer:-

Question # 27:-
Find

Answer:-

Question # 28:-
Find

Answer:-
.

Question # 29:-
Find

Answer:-

Question # 30:-
Find

Answer:-

Dr. Jamil Book Series 1 Page 41


Advanced Mathematical Techniques CH#4 Numerical Diff. Eqs. & Difference Equations

Question # 31:-
Find

Answer:-

Question # 32:-
Find

Answer:-

Question # 33:-
Find
.

Answer:-

Question # 34:-
The following ordinary differential equation(ODE) is used to describe a Population of Fish:

Where and are +ve constants; is known as the intrinsic growth constant. It measures
the differences between the birth and death rates per population unit in the absence of
overcrowding. The ratio is called saturation constant. Take and .
Use initial condition, Find , using Modified Euler’s Formula for step size
. Compare your results with the analytical solution of the above differential equation:

Dr. Jamil Book Series 1 Page 42


Advanced Mathematical Techniques CH#4 Numerical Diff. Eqs. & Difference Equations

EXERCISE-4
PART-II: DIFFERENCE EQUATIONS
Solve the following difference equations

Question # 39:-

Answer:-

Question # 40:-

Answer:-

Question # 41:-

Answer:-

{ ( ) ( )}

Question # 42:-
,
Answer:-

Question # 43:-
, ,
Answer:-

Question # 44:-

Answer:-
√ √
( ) ( )

Dr. Jamil Book Series 1 Page 43


Advanced Mathematical Techniques CH#4 Numerical Diff. Eqs. & Difference Equations

Question # 45:-

Answer:-

{ ( ) ( )}

Question # 46:-

Answer:-

Question # 47:-

Answer:-

Question # 48:-

Answer:-

Question # 49:-

Answer:-

Question # 50:-

Answer:-
(√ )

Question # 51:-

Answer:-

Dr. Jamil Book Series 1 Page 44


Advanced Mathematical Techniques CH#4 Numerical Diff. Eqs. & Difference Equations

Question # 52:-

Answer:-

Question # 53:-

Answer:-

Question # 54:-

Answer:-

( ) ( )

Question # 55:-

Answer:-

Question # 56:-

Answer:-

Question # 57:-

Answer:-

Question # 58:-

Answer:-

( )

Dr. Jamil Book Series 1 Page 45


Advanced Mathematical Techniques CH#4 Numerical Diff. Eqs. & Difference Equations

Question # 59:-
.
Answer:-

( )

Question # 60:-

Answer:-

( )

Question # 61:-

Answer:-

Question # 62:-

Answer:-

Question # 63:-

Answer:-

Question # 64:-

Answer:-

( √ ) ( √ )

Question # 65:-

Dr. Jamil Book Series 1 Page 46


Advanced Mathematical Techniques CH#4 Numerical Diff. Eqs. & Difference Equations

Answer:-

Question # 66:-

Answer:-

Question # 67:-

Answer:-

Question # 68:-
.
Answer:-

Question # 69:-

Answer:-

( )

Question # 70:-

Answer:-

Question # 71:-

Answer:-
( )

Dr. Jamil Book Series 1 Page 47


Advanced Mathematical Techniques CH#4 Numerical Diff. Eqs. & Difference Equations

Question # 72:-
.

Answer:-

Question # 73:-

Answer:-

( )

Question # 74:-
.
Answer:-

Question # 75:-
The Discrete Mathematical Modeling of Forced Vibrating System of particular oscillation
Phenomena leads to the following Nonhomogeneous Difference Equation of the form:

(1): Solve the difference equations in the form


(2): Can you relate this Discrete system to some Continuous Dynamical system?
(3): Write final conclusions on your General Solution.
Answer:-

Question # 76:-
The Discrete Mathematical Modeling of Forced Vibrating System of particular oscillation
Phenomena leads to the following Nonhomogeneous Difference Equation of the form:

(1): Solve the difference equations in the form


(2): Can you relate this Discrete system to some Continuous Dynamical system?
(3): Write final conclusions on your General Solution.

Answer:-

( )

Dr. Jamil Book Series 1 Page 48


Advanced Mathematical Techniques CH#4 Numerical Diff. Eqs. & Difference Equations

Question # 77:-
The Discrete Mathematical Modeling of Forced Vibrating System of particular oscillation
Phenomena leads to the following Nonhomogeneous Difference Equation of the form:

(1): Solve the difference equations in the form


(2): Can you relate this Discrete system to some Continuous Dynamical system?
(3): Write final conclusions on your General Solution.
Answer:-

Question # 78:-
The Discrete Mathematical Modeling of Some Vibrating System of particular oscillation
Phenomena leads to the following Nonhomogeneous Difference Equation of the form:

(1): Solve the difference equations in the form


(2): Can you relate this Discrete system to some Continuous Dynamical system?
(3): Write final conclusions on your General Solution.

Answer:-

( )

Dr. Jamil Book Series 1 Page 49


Advanced Mathematical Techniques CH#4 Numerical Diff. Eqs. & Difference Equations

SUMMARY OF CH#4
PART-I: NUMERICAL DIFFERENTIAL EQUATIONS
1): Euler’s Method:-
Aim:-

To solve first order and first-degree differential equation:

The Euler’s iteration formula for finding numerical solution of first order first degree
differential equation.

2): Modified Euler’s Method:-


The Modified Euler’s iteration formula for finding numerical solution of first order first
degree differential equation.

3): Runge – Kutta method (Fouth – Order) (RK – method):-


A fourth order Runge – Kutta formula for numerical solution of first order first degree
differential equation given by:

where,

Dr. Jamil Book Series 1 Page 50


Advanced Mathematical Techniques CH#4 Numerical Diff. Eqs. & Difference Equations

PART-II: DIFFERENCE EQUATIONS


Difference Equation:-
A difference equation is a relation between the values of of a function defined on a
discrete set of argument . In general, a difference equation involves differences of a
function.
Order of Difference Equation:-
The order of a difference equation is the difference between the largest and smallest
arguments appearing in it.
Degree of a Difference Equation:-
The degree of the highest differences function appearing in a difference equation free
from radicals and functions is called the degree of the difference equation.
Solution of Difference Equation:-
Solution of a difference equation is an expression for which satisfies the given difference
equation.
General Solution of Difference Equation:-
The general solution of a difference equation is that in which the number of arbitrary
constants is equal to the order of the difference equation.
Particular Integral/Solution of Difference Equation:-
A particular solution or particular integral is that solution which is obtained from the
general solution by giving particular values to the constants (using IVP & BVP).
Linear Difference Equation:-
In linear difference equations, occur in the first degree only and are not
multiplied together.
General Linear Difference Equation of Order :-
A general linear difference equation of order is of the form:

where and ) are given functions of irregular argument .


It can also be written as;

Solution of linear homogeneous difference equation with constant coefficients:-


Step-1:-
Let us consider the homogeneous difference equation

where are constants. It can be written as;

Step-2:-
Write auxiliary equation:
Auxillary Equation :-

Dr. Jamil Book Series 1 Page 51


Advanced Mathematical Techniques CH#4 Numerical Diff. Eqs. & Difference Equations

Replace by in and equate it to zero, we get

Step-3:-
Write the solutions according to the nature of the roots of auxiliary equation.
Case-I (Distinct Real Roots)
If all the roots be real and distinct, the general solution of ) is
given by

( )

Case-II (Repeated Real Roots)


If the roots is repeated “ ” times, then general solution is:

Case-III (Complex Roots)


Let has two imaginary roots . The general solution is given by

where
√ ( )

Solution of Non-Homogeneous Linear Difference Equation with Constant


Coefficients:-
The general form of non-homogeneous linear difference equation is given by

The general or total solution of a non-homogeneous linear difference equation with


constants coefficients is given by

where,
is the homogeneous solution o
and
= is the particular solution which satisfy,

Dr. Jamil Book Series 1 Page 52


Advanced Mathematical Techniques CH#4 Numerical Diff. Eqs. & Difference Equations

:-

1): Case-I:

2): Case-II: , then:-

1): (Linear Case)

2): (Quadratic Case)

3): (Cubic Case)

4): (BiQuadratic Case)

Note:-
If

⇒ ( ) and ( )

Note (For sepration of Real and Imagnary Parts):-

OR

can be written as

Dr. Jamil Book Series 1 Page 53


Advanced Mathematical Techniques CH#4 Numerical Diff. Eqs. & Difference Equations

and solve as in Case-I with . At the end separate real and imaginary parts
according to the cosine and sine case.

Factorial Polynomial :-

Important DPZ Formulae

Formulae Name Formulae Expressions

Derivative Formula

Power Formula

Zero Formula

We have

Then we expand in ascending power of and operate on , by using


DPZ formulae and get the required Particular Solutions

Expansion Formulae of :-

1):

2):

,
We have

Dr. Jamil Book Series 1 Page 54


Advanced Mathematical Techniques CH#4 Numerical Diff. Eqs. & Difference Equations

Using Multiplying Theorem, we have

and then solve as case-I, II OR III according to the form of .

Dr. Jamil Book Series 1 Page 55


Advanced Mathematical Techniques Chapter#5 Improper & Elliptic Integrals

CHAPTER # 5
IMPROPER &
ELLIPTIC
INTEGRLAS

Dr. Jamil Book Series 2 Page 1


Advanced Mathematical Techniques Chapter#5 Improper & Elliptic Integrals

CHAPTER # 5
PART-I: IMPROPER INTEGRLAS
Improper Integral of first kind:-

Reason:- Infinite limits of integration:

The improper integral of first kind has three types and is defined or written as:

1st Type: ∫ ( ) ∫ ( ) , -

2nd Type: ∫ ( ) ∫ ( ) , -

3rd Type: ∫ ( ) = ∫ ( ) , -

∫ ( ) ∫ ( ) +∫ ( )

∫ ( ) ∫ ( ) + ∫ ( )

( ) = ( ) ( )

Important Note:-

1): In lower limit , we have “+ve” sign that is

2): In Upper limit , we have “-ve” sign that is

Improper Integral of second kind:-

Reason:- Infinity of the integrant (i.e., f(x)):

The improper integral of second kind has three types and is defined or written as:

Dr. Jamil Book Series 2 Page 2


Advanced Mathematical Techniques Chapter#5 Improper & Elliptic Integrals

1st Type: ∫ ( ) ∫ ( ) 0 ( ) 1

2nd Type: ∫ ( ) ∫ ( ) 0 ( ) 1

3rd Type ∫ ( ) =∫ ( ) ∫ ( ) 0 ( ) 1

∫ ( ) ∫ ( ) ∫ ( )

( ) = ( ) ( )

Exercise #1:-

Evaluate the improper integral: ∫

Solution:-

This improper integral of 1st kind and 1st type and can be written as:

∫ = ∫ = 0 1

[ ]

Hence the integral converges to 1.

Exercise #2:-

Evaluate the improper integral: ∫

Solution:-

This improper integral of 1st kind and 1st type and can be written as:

∫ ∫ [ ]

, -

Hence the integral converges to .

Exercise #3:-

Evaluate the improper integral: ∫


( )

Dr. Jamil Book Series 2 Page 3


Advanced Mathematical Techniques Chapter#5 Improper & Elliptic Integrals

Solution:-

This improper integral of 1st kind and 3rd type and can be written as:

∫ ∫
( ) ( )

∫ [ ]
( )

∫ [ ]
( )

∫ , -
( )

Hence the integral converges to 0.

Exercise #4:-

Evaluate the improper integral: ∫

Solution:-

This improper integral of 1st kind and 3rd type and can be written as:

∫ ∫

Let and , then:

∫ ∫

∫ , - , -

∫ ⌈ ( ) ( )⌉ , -

Hence the integral converges to 0.

Exercise #5 (H.W):-

Evaluate the improper integral:

) ∫ 2): ∫ 3):∫ 4): ∫


( )
( )

Dr. Jamil Book Series 2 Page 4


Advanced Mathematical Techniques Chapter#5 Improper & Elliptic Integrals

Exercise #6:-

Evaluate the improper integral: ∫


Solution:-

This improper integral of 2nd kind and 2nd type and can be written as:

∫ ∫ , -
√ √

∫ , - ( )=

Hence the given integral converges to .

Exercise #7:-

Evaluate the improper integral: ∫


Solution:-

This improper integral of 2nd kind and 1st type and can be written as:

∫ ∫
√ √


∫ ∫ ( ) ( )


( )
∫ [ ]
√ ⁄

∫ 0 √ 1

Hence the given integral converges to -4.

Exercise #8:-

Evaluate the improper integral: ∫

Solution:-

This improper integral of 2nd kind and 3rd type and can be written as:

Dr. Jamil Book Series 2 Page 5


Advanced Mathematical Techniques Chapter#5 Improper & Elliptic Integrals

∫ ∫ ∫

∫ ∫ ∫

∫ [ ] [ ]

∫ [ ] [ ]

∫ , - , -

Hence the given integral is divergent.

Exercise #9:-

Evaluate the improper integral: ∫


( )

Solution:-

This improper integral is of mixed type and can be written as:

∫ ∫ ∫
( ) ( ) ( )

∫ ∫ ∫
( ) ( ) ( )

Now, consider the first integral of above equation we have,

∫ [ ]
( ) (

∫ [ ]* +
( )

∫ , -
( )

Since the first integral is divergent between the two. Hence the given integral is also

divergent.

Dr. Jamil Book Series 2 Page 6


Advanced Mathematical Techniques Chapter#5 Improper & Elliptic Integrals

Sphecial improper integrals of the first kind

1): The Geometric or Exponential integral:-

The integral: ∫ , where is a constant converges if and diverges if .

Example:-

The integrals:

1): ∫ ( ) ) ∫ ( ) are Convergent and

3): ∫ ( ) ) ∫ ( ) are Divergent.

2): The P-integral:-

The integral: ∫ where is a constant and , converges if and diverges if

Example:-

The integrals:

1): ∫ ( ) ) ∫ ( ) are Convergent and

) ∫ ( ) ) ∫ ( ) are Divergent.

Three convergence theorems for improper integrals of the first kind:-

Theorem – 1 (Comparison test) (For integrals with non-negative integrals):-

For convergence (Bring greater function and integral):-

Let ( ) and suppose that ∫ ( ) converges. Then if ( ) ( )

then ∫ ( ) also converges.

Dr. Jamil Book Series 2 Page 7


Advanced Mathematical Techniques Chapter#5 Improper & Elliptic Integrals

For divergence (Bring smaller function and integral):-

Let ( ) and suppose that ∫ ( ) diverges. Then if ( ) ( )

then ∫ ( ) also diverges.

Exercise #1:-

Test the convergence for: ∫

Solution:-

Since for

then for

Dr. Jamil Book Series 2 Page 8


Advanced Mathematical Techniques Chapter#5 Improper & Elliptic Integrals

Therefore ∫ converges by Exponential integral with and Hence by

comparison test ∫ converges.

Exercise #2:-

Test the convergence for: ∫

Solution:-

Since for

Then for .

Therefore ∫ diverges by P-integral with and . Hence by comparison

test ∫ also diverges.

Theorem – 2:-

Let ( ) , then

) ∫ ( ) converges if and “ ” is finite

) ∫ ( ) diverges if and ( may be infinite)

Exercise #3:-

Test the convergence for: ∫

Solution:-

We have,

. /

Here and ⁄ (finite). Hence by Theorem-2, the integral:

∫ converges.

Exercise #4:-

Test the convergence for:∫


Solution:-

Dr. Jamil Book Series 2 Page 9


Advanced Mathematical Techniques Chapter#5 Improper & Elliptic Integrals

We have,


Here and Hence by theorem-2 the integral: ∫ diverges.


Exercise #5: -

Test the convergence for:

) ∫ b):∫

Solution (a):-

We have,

0 1

Hence and ⁄ (finite). Hence by theorem-2, the integral:

∫ converges.

Solution (b):-

We have,

. /

Hence and (finite). Hence by theorem-2, the integral: ∫


diverges.

Exercise #6(HW): -

Test the convergence for:

) ∫ ) ∫ 3) ∫

Answer:-

1): Converges 2): Converges 3): Diverges

Dr. Jamil Book Series 2 Page 10


Advanced Mathematical Techniques Chapter#5 Improper & Elliptic Integrals

Theorem-3:-

If ∫ | ( )| converges, then ∫ ( ) converges.

Exercise#7:-

Test the convergence for: ∫

Solution:-

Consider the absolute value of the function of the given integral.

| |
∫ | | ∫ ∫ ∫
| |

Since | | and ∫ converges by P-integral with and

. Hence by comparison test and theorem-3, the integral ∫ converges.

Exercise #8(HW):-

Test the convergence for: ∫

Sphecial improper integral of the 2nd kind

The P-integral of the 2nd kind:-

) ∫ converges if and diverges if


( )

) ∫ converges if and diverges if


( )

Examples of the P-integral of the 2nd kind:-

) ∫ converges with

) ∫ converges with .

) ∫ diverges with
( )

Three convergence theorems for improper integrals of the second kind:-

Theorem-1 (comparison test) (for integrals with non-negative integrals):-

Dr. Jamil Book Series 2 Page 11


Advanced Mathematical Techniques Chapter#5 Improper & Elliptic Integrals

1a): For convergence (Bring greater function/integral):-

Let ( ) for and suppose that∫ ( ) converges.Then if ( ) ( )

for , then ∫ ( ) also converges.

1b): For divergence (Bring smaller function/integral):-

Let ( ) for and suppose that ∫ ( ) diverges. Then if ( ) ( ).

For , then ∫ ( ) also diverges.

Exercise #1:-

Test the convergence for:∫


Dr. Jamil Book Series 2 Page 12


Advanced Mathematical Techniques Chapter#5 Improper & Elliptic Integrals

Solution:-

This improper integral is of 2nd kind and 1st type.

Since for

for

√ √ for

for
√ √

Then ∫ converges by P-integral of the 2nd kind with and Hence by


comparison test, the integral: ∫ also converges.


Exercise #1:-

Test the convergence for: ∫


( )

Solution:-

This improper integral is of 2nd kind and 1st type.

Since for

for ,
( ) ( )

Then ∫ diverges by P-integral of the 2nd kind with and Hence


( )

by comparison test, the integral ∫ also diverges.


( )

Theorem-2a (For 1st type or lower limit):-

Let ( ) ( ) then

) ∫ ( ) converges if and is finite

) ∫ ( ) diverges if and (A may be infinite).

Theorem-2b (For 2ndtype or upper limit):-

Let ( ) ( ) then

) ∫ ( ) converges if and is finite

) ∫ ( ) diverges if and (A may be infinite).

Dr. Jamil Book Series 2 Page 13


Advanced Mathematical Techniques Chapter#5 Improper & Elliptic Integrals

Theorem-3:-

If ∫ | ( )| converges, then ∫ ( ) converges.

Exercise #3:-

Test the convergence for: ∫


Solution: -

This improper integral is of 2nd kind and 1st type.

We have,


( ) √
√ ( )( )


( ) √( =√
√ )( )( )

Here and (finite). Hence by theorem-2a, the integral: ∫ converges.


Exercise #4:-

Test the convergence for: ∫


( )√

Solution:-

This improper integral is of 2nd kind and 2ndtype.

We have,

( )
( )√ √

Here and (finite). Hence by theorem-2b, the integral:


∫ diverges.
( )√

Exercise #5:-

Test the convergence for: ∫


Dr. Jamil Book Series 2 Page 14


Advanced Mathematical Techniques Chapter#5 Improper & Elliptic Integrals

Solution:-

This improper integral is of 2nd kind and 1st type.

We have,

| |
∫ | | ∫ ∫
√ |√ | √

Since | | and ∫ converges with P-integral with and


. Hence by comparison test and theorem-3, the integral: ∫ converges.


Important Note:-

Convergence/Diverges Criteria for Improper Integral

Convergence criteria for Diverges criteria for

SN For improper integral of the And improper integral of the

1 1st kind. 1st kind.

2 2nd kind 2nd kind

Exercise #6(HW):-

Test the convergence for:

1): ∫ 2): ∫

3): ∫ 4): ∫
√( )( )

Answer:-

1): Diverges 2): Diverges

3): Converges 4): Diverges

Dr. Jamil Book Series 2 Page 15


Advanced Mathematical Techniques Chapter#5 Improper & Elliptic Integrals

CHAPTER # 5
PART-II: ELLIPTIC NTEGRLAS

1): The Incomplete Elliptic Integral of the First Kind:-


It is defined as,

( ) ∫ ,

where is the amplitude of ( ) or , written , and is its modulus,


written . The integral is also called Legendre’s form for the elliptic integral of
the first kind. If the integral is called the complete integral of the first kind and is
denoted by ( )or simply .

2): The Incomplete Elliptic Integral of the Second Kind:-


It is defined as,

( ) ∫ √ ,

Also called Legendre’s form for the elliptic integral of the second kind. If the
integral is called the complete elliptic integral of the second kind and is denoted by ( ) or
simply .

3): The Incomplete Elliptic Integral of the Third Kind:-


It is defined as,

( ) =∫ , .
( )√

Also called Legendre’s from for the elliptic integral of the third kind. Here is a constant
assumed different from zero since if third kind reduces to first kind. If the
integral is called the complete elliptic integral of the third kind.

Writing Integrals Into Elliptic Integrals:-

Examples:-
1): ∫ (√ )

2): ∫ (√ )= (√ ) (√ )

Dr. Jamil Book Series 2 Page 16


Advanced Mathematical Techniques Chapter#5 Improper & Elliptic Integrals

3): ∫ . / = . / . /

4): ∫ √ . /

5): ∫ √ (√ ) = (√ )

6): ∫ √ . /= ( )

7): ∫ = . /
. /√

8): ∫ = (√ ) (√ )
. /√

Example #1 :-

Prove that if :

( ) =∫ = { . / . / . / }.

Solution:-

By the binomial theorem, we have

. /. / . /. /. /
( ) . /( ) ( ) ( ) +...

+…

Putting , we have

+…

Integrating both sides from 0 to , we get

∫ ∫ ∫ ∫ ∫

Using the reduction formula, we have

Dr. Jamil Book Series 2 Page 17


Advanced Mathematical Techniques Chapter#5 Improper & Elliptic Integrals

( )( )( )
( )( )

( )( )( )
{ ( )( )

Then,

∫ | ∫

Substituting these values in above equation, we get

( ) =∫

( ) =∫ = { . / . / . / }.

Example # 2 :-

Evaluate∫ to 3 decimal places by first expressing the integral as an elliptic


integral.

Solution:-

Let, –

when, and, . Since . /

Then,

∫ ∫ ∫
√ √
√ . /

Let

Dr. Jamil Book Series 2 Page 18


Advanced Mathematical Techniques Chapter#5 Improper & Elliptic Integrals

√ √

√ √ √ √

when, and,

∫ √
∫ =∫
√ √ √

∫ ∫ ∫
√ √ √

∫ ∫ √ (√ ) √ (√ )
√ √

∫ √ { ( ) (√ ) ( ) (√ ) ( ) (√ ) }


Example # 3 :-

Evaluate ∫ √ in terms of elliptic integrals.

Solution:-

Let

√ √

=
√ √ √ √

when, and,

Dr. Jamil Book Series 2 Page 19


Advanced Mathematical Techniques Chapter#5 Improper & Elliptic Integrals

∫ √ =∫ √ ∫
√ √

( )
∫ √ =2∫

∫ √ ∫ √ ∫

∫ √ ∫ √ ∫

∫ √ √ ∫ √ √ ∫

∫ √ √ (√ ) √ (√ )

∫ √ √ (√ ) √ (√ )

Example # 4 :-

Evaluate: ∫ √ .

Solution:-

We have,

∫ √ ∫ √( ( )) ∫ √

Let, – when, and,

Since, . /

Dr. Jamil Book Series 2 Page 20


Advanced Mathematical Techniques Chapter#5 Improper & Elliptic Integrals

∫ √ ∫ √ ( ) ∫ √

∫ √ √ ∫ √ √ S (√ )

Example # 5 :-

Express ∫ √ in terms of incomplete elliptic integrals, where .

Solution:-

Let,

√ √

when, and ( )

Then,

∫ √ ∫ √

∫ √ ∫ ∫
√ √

. /
∫ √ ∫

∫ √ ∫ ∫ √

Dr. Jamil Book Series 2 Page 21


Advanced Mathematical Techniques Chapter#5 Improper & Elliptic Integrals

∫ √ ( ) ( ) ( )

Example # 6 :-

Prove that: ∫ (√ ) (√ ).
√ √

Solution:-

We have,

. /

Then,

∫ ∫ ∫
√ √
√ √

Let, –

When, and . Then,

∫ ∫ ∫
√ √ √
√ . – / √

By the property of definite integrals:

Dr. Jamil Book Series 2 Page 22


Advanced Mathematical Techniques Chapter#5 Improper & Elliptic Integrals

∫ ( ) ∫ ( ) ∫ ( )

∫ ∫ ∫
√ √
√ √
[ ]

∫ * (√ ) (√ )+
√ √

Example # 7 :-

Find the length of arc of the curve , .

Solution:-

The formula of arc-length in Cartesian coordinate is,

Arc-length ∫ √ . / ∫ √

Since, ∫ ( ) ∫ ( ) , if ( ) is even

Arc-length ∫ √ ∫ √

Dr. Jamil Book Series 2 Page 23


Advanced Mathematical Techniques Chapter#5 Improper & Elliptic Integrals

Arc-length √ ∫ √ √ (√ )

Example # 8 :-

Find the length of arc of the ellipse .

Solution:-

The arc-length formula in parametric coordinates system is given by:

Arc-length = ∫ √. / . /

Arc-length = ∫ √

Arc-length = ∫ √

Arc-length = ∫ √ ( )

Arc-length = ∫ √

Arc-length = ∫ √

Dr. Jamil Book Series 2 Page 24


Advanced Mathematical Techniques Chapter#5 Improper & Elliptic Integrals

This integral can not be evaluated in the form of the elementary functions (
). It is define a new function, called elliptic functions. This integral is called
the elliptic integral as it is derived from the determination of the Perimeter of the ellipse.
This integral can not be evaluated by standard methods of integration. First the integrand
√ is expanded as power series and integrated term by term

Arc-length ( ) ,

Example # 9 :-

Evaluate in term of Elliptical Integrals: ∫ .


√( )( )

Solution:-

Let,

When,

and,

∫ ∫ ∫
√( )( ) √( )( ) √

∫ ∫ ( ) ( ) ( )
√( )( ) √

Example # 10 :-

Evaluate in term of Elliptical Integrals:∫ .


√( )( )

Solution:-

Let,

When,

and,

Then,

Dr. Jamil Book Series 2 Page 25


Advanced Mathematical Techniques Chapter#5 Improper & Elliptic Integrals

∫ ∫ ∫
√( )( ) √

∫ ∫ ∫ √
√( )( ) √

Let,

When,

and, . /

∫ ∫ ∫
√( )( ) √ √ √

∫ *∫ ∫ +
√( )( ) √ √ √

∫ * (√ ) (√ )+
√( )( ) √

Example # 11 :-

Evaluate in term of Elliptical Integrals:∫ .


√( )( )

Solution :-

Let,

When,

and,

∫ ∫
√( )( ) √( )( )

∫ ∫ ∫

√( )( ) √

Dr. Jamil Book Series 2 Page 26


Advanced Mathematical Techniques Chapter#5 Improper & Elliptic Integrals

∫ ∫ ∫
√ √
√( )( )

√ √
∫ ∫ ( ) ( )
√( )( ) √

Example # 12 :-

Express in term of Elliptical Integrals:∫ .


( )√( )( )

Solution:-

Let,

When,

and,

Then,

∫ ∫
( )√( )( ) ( )√( )( )

∫ ∫
( )√( )( ) √

∫ ∫
( )√( )( ) ( )√ )

( )
∫ ∫
( )√( )( ) ( )√ )

∫ ∫ ∫
( )√( )( ) √ ( )√ )

∫ ∫ ∫
( )√( )( ) √ ) ( )√ )

Dr. Jamil Book Series 2 Page 27


Advanced Mathematical Techniques Chapter#5 Improper & Elliptic Integrals

∫ ∫ ∫
( )√( )( ) √ . /√ )

√ √
∫ ( ) ( )
( )√( )( )

Example # 13 :-

Evaluate ∫ in terms of elliptic integrals.


√( )( )

Solution:-

Let, . Then,


√( )( )


√* ( ) +* ( ) +

Choose so that constant terms of each quadratic are equal

Then,


√( )( )

Let,

Then,


√( )( )

Choose so that the coefficient of in each quadratic is equal to the term i.e = 9

or = 3. Then,


√( )( )

Let ( )
,

Then,

Dr. Jamil Book Series 2 Page 28


Advanced Mathematical Techniques Chapter#5 Improper & Elliptic Integrals

√ ∫
√( )( )

Now let and solve as in previous problem.

Example #14 :-

Express: ∫ in term of elliptic integrals.


Solution:-

We have,


Dr. Jamil Book Series 2 Page 29


Advanced Mathematical Techniques Chapter#5 Improper & Elliptic Integrals

EXERCISE- 5
PART-I: IMPROPER INTEGRALS
“Evaluate the following improper integral”

1): ∫ 2): ∫ ( )

3): ∫ 4): ∫

5): ∫ ⁄
6): ∫
( )

7): ∫ 8): ∫
( )

9): ∫ 10): ∫
( ) ( )

11): ∫ 12): ∫

13): ∫ 14): ∫

15): ∫ 16): ∫
√ √

17): ∫ 18): ∫
√ ( )

⁄ ( )
19): ∫ 20): ∫

21): ∫ 22): ∫

23): ∫ 24): ∫

Answers:-
1): 1 2): 0

3): 4):

Dr. Jamil Book Series 2 Page 30


Advanced Mathematical Techniques Chapter#5 Improper & Elliptic Integrals

5): 6):

7): 8):

9): 10): 0

11): 0 12):

13): 14): 0

15): -4 16): 2

17): 18):

19): 20):

21): 22): 6

23): 24):

Test the convergence of the following improper integrals

25): ∫ 26): ∫

27): ∫ 28): ∫

29): ∫ 30): ∫

31): ∫ 32): ∫

Dr. Jamil Book Series 2 Page 31


Advanced Mathematical Techniques Chapter#5 Improper & Elliptic Integrals

33): ∫ 34): ∫

35): ∫ 36): ∫

37): ∫ 38): ∫

39): ∫ ( ) 40): ∫

41): ∫ 42): ∫
√ ( )

43): ∫ 44): ∫
( )√ √

45): ∫ 46): ∫
√ ( ) ⁄

47): ∫ 48): ∫


49): ∫ 50): ∫
√( )( )

Answers:-

25): Converges 26): Diverges

27): Converges 28): Diverges

29): Converges 30): Diverges

31): Converges 32): Converges

Dr. Jamil Book Series 2 Page 32


Advanced Mathematical Techniques Chapter#5 Improper & Elliptic Integrals

33): Diverges 34): Converges

35): Diverges 36): Converges

37): Converges 38): Converges

39): Converges 40): Diverges

41): Converges 42): Diverges

43): Diverges 44): Converges

45): Converges 46): Converges

47): Diverges 48): Diverges

49): Converges 50): Diverges

Dr. Jamil Book Series 2 Page 33


Advanced Mathematical Techniques Chapter#5 Improper & Elliptic Integrals

EXERCISE- 5
PART-II: ELLIPTIC INTEGRALS
Example # 51 :-

Prove that if :

( ) =∫ = { . / . / . / }.

Example # 52 :-

Evaluate∫ to 3 decimal places by first expressing the integral as an elliptic integral.


Answer:-


Example # 53 :-

Evaluate ∫ √ in terms of elliptic integrals.

Answer:-

∫ √ √ (√ ) √ (√ )

Example # 54 :-

Evaluate: ∫ √ .

Answer:-

∫ √ √ S (√ )

Question # 55:-
Express ∫ √ in terms of incomplete elliptic integrals, where .

Answer:-

∫ √ ( ) ( ) ( )

Dr. Jamil Book Series 2 Page 34


Advanced Mathematical Techniques Chapter#5 Improper & Elliptic Integrals

Example # 56 :-

Prove that: ∫ (√ ) (√ ).
√ √

Example # 57 :-

Find the length of arc of the curve , .

Answer:-

Arc-length √ ∫ √ √ (√ )

Example # 58 :-

Find the length of arc of the ellipse .

Answer:-

Arc-length ( )

Example # 59 :-

Evaluate in term of Elliptical Integrals: ∫ .


√( )( )

Answer:-

∫ ( ) ( ) ( )
√( )( )

Question # 60:-
Evaluate in term of Elliptical Integrals:∫ .
√( )( )

Answer:-

∫ * (√ ) (√ )+
√( )( ) √

Example # 61 :-

Evaluate in term of Elliptical Integrals:∫ .


√( )( )

Answer:-
√ √
∫ ∫ ( ) ( )
√( )( ) √

Dr. Jamil Book Series 2 Page 35


Advanced Mathematical Techniques Chapter#5 Improper & Elliptic Integrals

Example # 62 :-

Express in term of Elliptical Integrals:∫ .


( )√( )( )

Answer:-

√ √
∫ ( ) ( )
( )√( )( )

Example # 63 :-

Evaluate ∫ in terms of elliptic integrals.


√( )( )

Answer:-
√ ∫
√( )( )

Now let and solve as in previous problem.

Question # 64:-

Express: ∫ in term of elliptic integrals.


Question # 65:-
a): Use the binomial theorem to show that if | | .

√ ( ) ( ) ( )

b): If  < 1, prove that:

. /=∫ √

. /= { . / (. / . / }

Question # 66:-
Evaluate:
√ √ √
a): . / b): . / c): S(0.5) d): . /

Answer:-

a): 1.3506 b):1.8541 c): 1.4675 d): 2.1565

Dr. Jamil Book Series 2 Page 36


Advanced Mathematical Techniques Chapter#5 Improper & Elliptic Integrals

Question # 67:-
Show that:

a): ( )

b): ( ) ( ) . /

Question # 68 :-
Find the perimeter of the ellipse .

Hint:- Let be the parametric equations

Answer:-

15.865

Question # 69:-

Evaluate: ∫ in term of elliptic integrals.


Answer:-

∫ ( )
√ √ √

Question # 70:-

Express: ∫ in terms of elliptic integrals.


Answer:-
( ) (√ )
√ √

Question # 71:-


Show that: ∫ ( √ )

Question # 72:-

Evaluate: ∫ .
√ ( )( )
Answer:-
∫ √ ( )
√ ( )( ) √

Dr. Jamil Book Series 2 Page 37


Advanced Mathematical Techniques Chapter#5 Improper & Elliptic Integrals

Question # 73:-

∫ . /.
√ √ √

Question # 74:-

Express each of the following in terms of elliptic integrals:

a): ∫
√( )( )

b): ∫ √

c): ∫
√( )( )

Answer:-
a): . / b): . / c): √ . /

Question # 75:-

Evaluate:

a): ∫
√( )( )( )

b): ∫
√( )( )( )

Answer :-

√ √
a): ( )

√ √
b): . / . /

Question # 76:-


Show that:∫ . /.

Dr. Jamil Book Series 2 Page 38


Advanced Mathematical Techniques Chapter#5 Improper & Elliptic Integrals

Summary Of CH # 5
PART-I: IMPROPER INTEGRLAS
Improper Integral of first kind:-

Reason:- Infinite limits of integration:

1st Type: ∫ ( ) ∫ ( ) , -

2nd Type: ∫ ( ) ∫ ( ) , -

3rd Type: ∫ ( ) = ∫ ( ) , -

∫ ( ) ∫ ( ) +∫ ( )

∫ ( ) ∫ ( ) + ∫ ( )

( ) = ( ) ( )

Improper Integral of second kind:-

Reason:- Infinity of the integrant (i.e., f(x)):

1st Type: ∫ ( ) ∫ ( ) 0 ( ) 1

2nd Type: ∫ ( ) ∫ ( ) 0 ( ) 1

3rd Type ∫ ( ) =∫ ( ) ∫ ( ) 0 ( ) 1

∫ ( ) ∫ ( ) ∫ ( )

( ) = ( ) ( )

Dr. Jamil Book Series 2 Page 39


Advanced Mathematical Techniques Chapter#5 Improper & Elliptic Integrals

Sphecial improper integrals of the first kind

1): The Geometric or Exponential integral:-

The integral: ∫ , where is a constant converges if and diverges if .

2): The P-integral:-

The integral: ∫ where is a constant and , converges if and diverges if

Three convergence theorems for improper integrals of the first kind:-

Theorem – 1 (Comparison test) (For integrals with non-negative integrals):-

For convergence (Bring greater function and integral):-

Let ( ) and suppose that ∫ ( ) converges. Then if ( ) ( )

then ∫ ( ) also converges.

For divergence (Bring smaller function and integral):-

Let ( ) and suppose that ∫ ( ) diverges. Then if ( ) ( )

then ∫ ( ) also diverges.

Theorem – 2:-

Let ( ) , then

) ∫ ( ) converges if and “ ” is finite

) ∫ ( ) diverges if and ( may be infinite)

Theorem-3:-

If ∫ | ( )| converges, then ∫ ( ) converges.

Sphecial improper integral of the 2nd kind

The P-integral of the 2nd kind:-

) ∫ converges if and diverges if


( )

) ∫ converges if and diverges if


( )

Dr. Jamil Book Series 2 Page 40


Advanced Mathematical Techniques Chapter#5 Improper & Elliptic Integrals

Three convergence theorems for improper integrals of the second kind:-

Theorem-1 (comparison test) (for integrals with non-negative integrals):-

1a): For convergence (Bring greater function/integral):-

Let ( ) for and suppose that∫ ( ) converges.Then if ( ) ( )

for , then ∫ ( ) also converges.

1b): For divergence (Bring smaller function/integral):-

Let ( ) for and suppose that ∫ ( ) diverges. Then if ( ) ( ).

For , then ∫ ( ) also diverges.

Theorem-2a (For 1st type or lower limit):-

Let ( ) ( ) then

) ∫ ( ) converges if and is finite

) ∫ ( ) diverges if and (A may be infinite).

Theorem-2b (For 2ndtype or upper limit):-

Let ( ) ( ) then

) ∫ ( ) converges if and is finite

) ∫ ( ) diverges if and (A may be infinite).

Theorem-3:-

If ∫ | ( )| converges, then ∫ ( ) converges.

Important Note:-

Convergence/Diverges Criteria for Improper Integral

Convergence criteria for Diverges criteria for

SN For improper integral of the And improper integral of the

1 1st kind. 1st kind.

2 2nd kind 2nd kind

Dr. Jamil Book Series 2 Page 41


Advanced Mathematical Techniques Chapter#5 Improper & Elliptic Integrals

Summary Of CH # 5
PART-II: ELLIPTIC INTEGRLAS
1): The Incomplete Elliptic Integral of the First Kind:-
It is defined as,

( ) ∫ ,

where is the amplitude of ( ) or , written , and is its modulus,


written . The integral is also called Legendre’s form for the elliptic integral of
the first kind. If the integral is called the complete integral of the first kind and is
denoted by ( )or simply .

2):The Incomplete Elliptic Integral of the Second Kind:-


It is defined as,

( ) ∫ √ ,

Also called Legendre’s form for the elliptic integral of the second kind. If the
integral is called the complete elliptic integral of the second kind and is denoted by ( ) or
simply .

3): The Incomplete Elliptic Integral of the Third Kind:-


It is defined as,

( ) =∫ , .
( )√

Also called Legendre’s from for the elliptic integral of the third kind. Here is a constant
assumed different from zero since if third kind reduces to first kind. If the
integral is called the complete elliptic integral of the third kind.

Dr. Jamil Book Series 2 Page 42

You might also like